Sei sulla pagina 1di 143

Polos Olmpicos de Treinamento

Aula

Curso de lgebra - Nvel 2


Prof. Marcelo Mendes

Produtos Not
aveis

Varios problemas de Algebra


para alunos do Ensino Fundamental utilizam Produtos
Notaveis, que s
ao identidades cl
assicas envolvendo multiplicacao de express
oes.
Vejamos alguns exemplos para diversos produtos not
aveis que auxiliarao na formacao
de ideias para problemas futuros mais difceis.

1 Quadrado da soma ou da diferenca de dois n


umeros
(a + b)2 = a2 + 2ab + b2
(a b)2 = a2 2ab + b2
Problema 1. (OCM) Prove que n
ao existem inteiros positivos a e b tais que

b2 +b
a2 +a

= 4.

Soluc
ao. Suponha que
tais inteiros positivos a e b. A equacao dada e equivalente
 existam
2
2
2
a b + b = 4 a + a = 4a + 4a. Isso lembra o quadrado de 2a + 1, que e 4a2 + 4a + 1.
Assim, seria bom somarmos 1 a cada lado, para obtermos
b2 + b + 1 = 4a2 + 4a + 1.
Por outro lado,
b2 < b2 + b + 1 < b2 + 2b + 1 = (b + 1)2
pois b e um inteiro positivo. Como b2 e (b + 1)2 s
ao quadrados consecutivos, isso mostra
que n
ao seria possvel b2 + b + 1 ser o quadrado de um inteiro.
No pr
oximo exemplo, vamos utilizar um fato u
til de pensar que um n
umero com todos
.
Se
o
n
u
mero
possuir apenas o
os dgitos 1s, como 11...1, pode ser escrito na forma 99...9
9
99...9
dgito 4, por exemplo, como 44...4, entao o escrevemos na forma 4 9 . A vantagem
n
dessas alterac
oes e saber que 99...9
| {z } = 10 1 (verifique esse fato para quantidades pequenas
n

de 9s).


POT 2012 - Algebra
- Nvel 2 - Aula 1 - Prof. Marcelo Mendes

Problema 2. Seja n > 1 um n


umero inteiro. Prove que o n
umero
racional.

ao e
11...1
| {z } 44...4
| {z } n
n

2n

Soluc
ao. Mostrar que 11...144...4 n
ao e racional e equivalente a provar que 11...144...4
n
ao e um quadrado perfeito. Ou seja, este problema tenta mostrar que n
ao h
a outros
quadrados perfeitos com o formato do n
umero 144.
Podemos escrever
102n 1
10n 1
102n + 4
9
9
 10n 1
10n 1
=
(10n + 2)2 .
102n + 4 (10n + 1) =
9
9
Agora, e suficiente mostrarmos que 10n 1 nunca pode ser quadrado perfeito se n > 1.
Isso e verdade pelo fato de 10n 1 deixar resto 3 na divisao por 4 e n
ao existir quadrado
perfeito nessa situac
ao.
11...144...4 = 11...1 102n + 44...4 =

Problema 3. (i) Se n e um inteiro positivo tal que 2n + 1 e um quadrado perfeito, mostre


que n + 1 e a soma de dois quadrados perfeitos sucessivos.
(ii) Se 3n + 1 e um quadrado perfeito, mostre que n + 1 e a soma de tres quadrados.
Problema 4. Suponha que um n
umero inteiro n seja a soma de dois n
umeros triangulares,
b2 +b
a2 +a
ou seja, n = 2 + 2 . Mostre que 4n+1 pode ser escrito como a soma de dois quadrados
em termos de a e b.
Problema 5. Seja x R tal que x +

1
1
= 5. Calcule x2 + 2 .
x
x

Problema 6. (EUA) O n
umero 121b , escrito na base inteira b, e o quadrado de um inteiro
para quais valores de b?
Problema
7. Seja D = a2 + b2 + c2 , sendo a e b inteiros consecutivos e c = ab. Mostre que

D e sempre um inteiro mpar.



2
2
Problema 8. (EUA) Determine a soma dos dgitos na base 10 de 104n +8 + 1 , sendo n
um inteiro positivo.
Problema 9. Mostre que a soma dos quadrados de dois n
umeros mpares consecutivos e
um n
umero par n
ao m
ultiplo de 4.


POT 2012 - Algebra
- Nvel 2 - Aula 1 - Prof. Marcelo Mendes

Problema 10. (IME) Mostre que os n


umeros 49, 4489, 444889, 44448889, ..., obtidos colocandose 48 no meio do n
umero anterior, s
ao quadrados de n
umeros inteiros.
Problema 11. Se x12 + 2x6 (1 2y 2 ) + 1 = 0 e x R , entao mostre que y < 1.
Problema 12. Ache todos os inteiros positivos x, y tais que y 2 x(x + 1)(x + 2)(x + 3) = 1.
Problema 13. Determine todas as triplas de n
umeros reais (x, y, z) que s
ao solucao da
equacao 4x4 x2 4y 4 + 4z 4 1 2xyz + y 8 + 2y 4 z 4 + y 2 z 2 + z 8 = 0.
Problema 14. (OCM) Determine todos os valores reais de x, y e z satisfazendo a igualdade
3x2 + y 2 + z 2 = 2xy + 2xz.
Problema 15. (OCM) Determine todos os pares de inteiros (x, y) que satisfazem a equacao
x2 + x + 1995 = y 2 + y.
Problema 16. (EUA) Encontre x2 + y 2 se x, y Z e xy + x + y = 71, x2 y + xy 2 = 880.

2 Diferenca de quadrados
a2 b2 = (a + b)(a b)
Problema 17. Quantos pares de n
umeros inteiros positivos m e n satisfazem a equacao
2
2
m n = 2011?
Soluc
ao. Suponha que existam inteiros positivos m e n tais que m2 n2 = 2011. Da,
(m + n)(m n) = 2011. Como 2011 e primo e m + n > m n, pois n > 0, segue que
m + n = 2011 e m n = 1 e, portanto, m = 1006 e n = 1005.
Problema 18. Prove que existe exatamente um n
umero natural n tal que 28 + 211 + 2n e
um quadrado perfeito.
Soluc
ao. Vamos buscar
soluc
oes para a equacao 28 + 211 + 2n = k2 , k Z. Ela e

equivalente a 28 1 + 23 + 2n = k2 ou 2n = k2 482 = (k + 48)(k 48). Assim, k + 48 = 2a
e k 48 = 2b , sendo n = a + b. Subtraindo essas equacoes, obtemos 96 = 2a 2b e,
portanto, 25 3 = 2b 2ab 1 . Em cada membro dessa igualdade, temos a fatoracao em
parte par e parte mpar. Igualando, obtemos b = 5 e a = 7. Portanto, a u
nica solucao e
n = a + b = 7 + 5 = 12.




Problema 19. Determine o valor do produto 1 212 1 312 ... 1 912 1 1012 .
3


POT 2012 - Algebra
- Nvel 2 - Aula 1 - Prof. Marcelo Mendes

Problema 20. (EUA) Simplifique a express


ao

( 5 + 6 + 7)( 5 + 6 7)( 5 6 + 7)( 5 + 6 + 7).

Problema 21. (OCM/ITA) Qual e o menor inteior positivo n tal que n n 1 < 0, 01.
Problema 22. Quantos pares de n
umeros inteiros m e n satisfazem a equacao m2 n2 =
2014?
Problema 23. Seja a 6= 1 um n
umero real. Simplifique a express
ao


 


1
1
1
1
1+
1 + 4 ... 1 + 2100 .
1+ 2
a
a
a
a
Problema 24. Racionalize a express
ao
(1 +

64

2)(1 +

32

2)(1 +

16

.
2)(1 + 8 2)(1 + 4 2)(1 + 2 2)

Problema 25. (OCM) Encontre o quociente da divisao de a128 b128 por


(a64 + b64 )(a32 + b32 )(a16 + b16 )(a8 + b8 )(a4 + b4 )(a2 + b2 )(a + b).
Problema 26. A express
ao 2n + 1 e o quadrado de um inteiro para exatamente quantos
n
umeros naturais n?
Problema 27. Determine todas as solucoes inteiras da equacao 32x 52y = 104.

Problema 28. (EUA) Se x + x2 1 + x1x2 1 = 20, entao determine o valor de


x2 +

x4 1 +

x2

.
+ x4 1

Problema 29. Um quadrado e cortado em 49 quadrados menores. Todos esses quadrados


tem as medidas de seus lados, em centmetros, expressas por n
umeros inteiros positivos.
Ha exatamente 48 quadrados com
area igual a 1cm2 . Determine o n
umero de resultados
2
possveis para expressar, em cm , a medida da area do quadrado original.
Problema 30. Seja
umero primo mpar dado. Quantos valores de k inteiro positivo
p p um n
2
existem tais que k pk e tambem um inteiro positivo?
Problema 31. (EUA) Existe um u
nico par de inteiros positivos x e y satisfazendo a equacao
x2 + 84x + 2008 = y 2 . Determine o valor de x + y.

Problema 32. (EUA) Calcule

(104 + 324)(224 + 324)(344 + 324)(464 + 324)(584 + 324)


.
(44 + 324)(164 + 324)(284 + 324)(404 + 324)(524 + 324)
4


POT 2012 - Algebra
- Nvel 2 - Aula 1 - Prof. Marcelo Mendes

3 Produtos not
aveis envolvendo cubos
Soma de dois cubos: a3 + b3 = (a + b)(a2 ab + b2 )
Diferenca de dois cubos: a3 b3 = (a b)(a2 + ab + b2 )
Cubo da soma de dois n
umeros: (a + b)3 = a3 + 3a2 b + 3ab2 + b3
= a3 + b3 + 3ab(a + b)
Cubo da diferenca de dois n
umeros: (a b)3 = a3 3a2 b + 3ab2 b3
= a3 b3 3ab(a b)
Problema 33. (Eslovenia) Sejam a, b R tais que a3 3a2 + 5a = 1 e b3 3b2 + 5b = 5.
Calcule o valor de a + b.
Soluc
ao. As express
oes nessas equac
oes lembram os cubos das diferencas de a e 1 e b e 1,
respectivamente. Assim, podemos reescreve-las como
(a 1)3 + 2(a 1) = 2,
(b 1)3 + 2(b 1) = 2.
Somando-as, obtemos


(a + b 2) (a 1)2 (a 1)(b 1) + (b 1)2 + 2 = 0.

Agora, observe que

(a 1)2 (a 1)(b 1) + (b 1)2 + 2


= (a 1)2 (a 1)(b 1) +


b1
= a1
2
Assim, a + b = 2.

2

(b 1)2 3(b 1)2


+
+2
4
4

3(b 1)2
+ 2 > 0.
4

Problema 34. Prove que se a + b + c = 0, entao a3 + b3 + c3 = 3abc.


Soluc
ao. Se a + b = c, ent
ao (a + b)3 = (c)3 , ou seja,
a3 + b3 + 3ab(a + b) = c3 a3 + b3 + 3ab(c) = c3 .
Logo, a3 + b3 + c3 = 3abc.
Problema 35. (Putnam)
Sejam x, y, z n
umeros reais distintos dois a dois. Prove que

3
x y + 3 y z + 3 z x 6= 0.

Problema 36. Determine o n


umero de solucoes reais distintas da equacao 3 x+ 3 7 x = 3.
5


POT 2012 - Algebra
- Nvel 2 - Aula 1 - Prof. Marcelo Mendes

Problema 37. (EUA/OCM) Mostre que se x e um n


umero satisfazendo 3 x + 9 3 x 9 =
3, entao 75 < x2 < 85.
s
s
r
r
125
125
3
3
3 + 9 +
e um n
umero
Problema 38. (IME 1991) Mostre que 3 + 9 +
27
27
racional.
Problema 39. (EUA) Se x e y s
ao n
umeros inteiros tais que x3 +y 3 +(x+y)3 +30xy = 2000,
determine o valor de x + y.
Problema 40. (Leningrado) Prove que

(23 1)(33 1) . . . (1003 1)


3367
=
.
3
3
3
(2 + 1)(3 + 1) . . . (100 + 1)
5050

4 Outros produtos not


aveis
ab a b + 1 = (a 1)(b 1)
ab + a + b + 1 = (a + 1)(b + 1)
Problema 41. Determine o n
umero de pares ordenados (m, n) de n
umeros inteiros positivos
2
4
que s
ao soluc
oes da equac
ao m + n = 1.
4
Soluc
ao. A equac
ao m
+ n2 = 1 e equivalente a mn 2m 4n + 8 = 8 (m 4)(n 2) = 8,
seguindo os modelos propostos nesta secao.
As possibilidades s
ao m 4 = 1, n 2 = 8; m 4 = 2, n 2 = 4; m 4 = 4, n 2 =
2; m 4 = 8, n 2 = 1, ou seja, os pares ordenados (m, n) s
ao (5, 10); (6, 6); (8, 4); (12, 3).

Problema 42. Determine todos os n


umeros inteiros tais que a soma e o produto s
ao iguais.
Problema 43. (IME) Sejam x1 e x2 as razes da equacao x2 + (m 15)x + m = 0. Sabendo
que x1 e x2 s
ao n
umeros inteiros, determine o conjunto dos possveis valores de m.


POT 2012 - Algebra
- Nvel 2 - Aula 1 - Prof. Marcelo Mendes

Problemas da OBM
Problema 44. (OBM 1a fase/2002) Se xy = 2 e x2 + y 2 = 5, entao
a)

5
2

b)

25
4

c)

5
4

d)

1
2

x2
y2

y2
x2

+ 2 vale:

e) 1

Problema 45. (OBM 3a fase/2003) Mostre que x2 + 4y 2 4xy + 2x 4y + 2 > 0 quaisquer


que sejam os reais x e y.
Problema 46. (OBM 2a fase/2005)
a) Fatore a express
ao x2 9xy + 8y 2 .
b) Determine todos os pares de inteiros (x; y) tais que 9xy x2 8y 2 = 2005.

Problema
47.q(OBM 1a fase/2005) Os inteiros positivos x e y satisfazem a equacao
q

x + 12 y x 21 y = 1. Qual das alternativas apresenta um possvel valor de y?

a) 5

b) 6

c) 7

d) 8

e) 9

Problema 48. (OBM 3a fase/2006) Encontre todos os pares ordenados (x; y) de inteiros
tais que x3 y 3 = 3(x2 y 2 ).

Problema 49. (OBM 2a fase/2006) Sejam a e b n


umeros reais distintos tais que a2 = 6b+5ab
2
e b = 6a + 5ab.
a) Determine o valor de a + b.
b) Determine o valor de ab.
Problema 50. (OBM 2a fase/2008) Sejam x e y n
umeros reais positivos satisfazendo as
1
.
Calcule
o
valor
de xy
.
equacoes x2 + y 2 = 1 e x4 + y 4 = 17
18
Problema 51. (OBM 1a fase/2010) Quantos s
ao os pares (x, y) de inteiros positivos tais
que x2 y 2 = 22010 ?
a) 1000

b) 1001

c) 1002

d) 1003

e) 1004

Problema 52. (OBM 3a fase/2010) Sejam a, b e c reais tais que a 6= b e a2 (b + c) =


b2 (c + a) = 2010. Calcule c2 (a + b).
Problema 53. (OBM 1a fase/2011) Qual e o valor da express
ao 201120112 + 201120032
16 20112007?
a) 2 201120072

b) 2 201120032
c) 2 20112007
d) 2 20112003
e) 2 201120112
7


POT 2012 - Algebra
- Nvel 2 - Aula 1 - Prof. Marcelo Mendes

Dicas

3. Observe que 2n + 1 e o quadrado de um inteiro mpar e que 3n + 1 e o quadrado de


um n
umero n
ao m
ultiplo de 3.
6. N
umeros na base b s
o utilizam dgitos 0, 1, ..., b 1.
10. Escreva 44...4
| {z } 9 = 4
| {z } 88...8
n+1

10n+1 1
9

+8

10n 1
9

+ 9.

12. Agrupe x com x + 3 e x + 1 com x + 2.


13. Comece separando o 1 de dentro dos parenteses (escrevendo x4 depois). Em seguida,
agrupe y 8 , z 8 e 2y 4 z 4 .
14. Se uma soma de quadrados de n
umeros reais e 0, entao todos os n
umeros s
ao iguais
a 0.

15. Veja a resoluc


ao do problema 1.
16. Fatore e faca substituic
oes de vari
aveis x + y = s e xy = p.

a n+
21. Multiplique
a inequac
ao membro a membro por n + n 1. Voce obter

n 1 > 100, cuja menor soluc


ao e 2500.
23. Multiplique e divida tudo por 1 a1 .
27. Primeiramente, descarte os casos em que os n
umeros s
ao negativos. Depois, use que
soma e diferenca de dois n
umeros inteiros tem a mesma paridade. Por fim, lembre-se
que o produto de dois n
umeros negativos e positivo.
30. Escreva k2 pk = n2 e complete o trin
omio quadrado perfeito que comeca com k2 pk,
2
somando e subtraindo p4 .
31. Complete o trin
omio quadrado perfeito que comeca com x2 + 84x.
32. Fatore a express
ao x4 + 324 = x4 + 182 . A dica e somar e subtrair 2 x2 18.
39. Passe 2000 para o lado esquerdo da equacao e fatore fazendo aparecer o fator x + y.


POT 2012 - Algebra
- Nvel 2 - Aula 1 - Prof. Marcelo Mendes

Respostas
5. 23
6. b > 2
8. 4
12. y = x2 + 3x + 1
13. (t2 , t, t) ou (t2 , t, t), t R
14. x = y = z = 0
15. Nao existe par (x, y)
16. 146
19.

11
20

20. 104
21. 2501
22. 0
101

23.
24.

1a2
1a1

64

21

25. a64 b64


26. 1
27. x = 3, y = 2
28. 51, 005
29. 2
30. 1 (para cada primo mpar p)
31. 80
32. 373
36. 2
39. 10
42. (0, 0), (2, 2)
43. 0, 7, 9, 25, 27, 34

Polos Olmpicos de Treinamento


Aula

Curso de lgebra - Nvel 2


Prof. Marcelo Mendes

Equac
oes e Sistemas de Equac
oes

Neste 2o texto de Algebra,


veremos diversos exemplos de equacoes e sistemas de equacoes
em nvel de problemas olmpicos do ensino fundamental.
Eles, possivelmente, servir
ao posteriormente de ideia para problemas mais difceis.

1 Equac
oes
Nossos tres primeiros exemplos s
ao de equacoes em que as solucoes utilizam produtos
not
aveis, como aplicac
ao do u
ltimo assunto.
Problema 1. (EUA) Determine o n
umero de solucoes inteiras da equacao 22x 32y = 55.
Soluc
ao. Inicialmente, observe que o lado esquerdo da equacao e a diferenca dos quadrados
x
de 2 e 3y e, portanto, (2x + 3y ) (2x 3y ) = 55. Veja que x e y s
ao positivos (prove isso!),
alem de (2x + 3y ) e (2x 3y ). Assim, as u
nicas possibilidades s
ao


2x + 3y = 55
e
2x 3y = 1

2x + 3y = 11
.
2x 3y = 5

Apenas o segundo sistema possui solucao, que e (x, y) = (3, 1).


Problema 2. Quantas soluc
oes inteiras possui a equacao x2 4xy + 6y 2 2x 20y = 29?
Soluc
ao. Os dois primeiros termos do lado esquerdo d
ao a pista do comeco pois lembram
o quadrado de x 2y. Assim, vamos reescrever a equacao da seguinte forma
x2 4xy + 4y 2 2x + 4y + 1 + 2y 2 24y + 72 = 102
(x 2y)2 2(x 2y) + 1 + 2(y 2 12y + 36) = 102
(x 2y 1)2 + 2(y 6)2 = 102.


POT 2012 - Algebra
- Nvel 2 - Aula 2 - Prof. Marcelo Mendes

Assim, x 2y 1 e par e n
ao maior que 10. Testanto x 2y 1 = 0, 2, 4, 6, 8, 10,
obtemos (y 6)2 = 51, 49, 43, 33, 19, 1. Logo, as u
nicas solucoes vem de x 2y 1 = 2 e
y 6 = 7 ou x 2y 1 = 10 e y 6 = 1. As solucoes, portanto, s
ao
(29, 13); (25, 13); (1, 1); (3, 1); (25, 7); (5, 7); (21, 5); (1, 5).
Problema 3. (Romenia/2006) Encontre todos os n
umeros reais a e b satisfazendo
2(a2 + 1)(b2 + 1) = (a + 1)(b + 1)(ab + 1).
Soluc
ao. Utilizando produtos not
aveis, a equacao dada fica equivalente a

2 a2 b2 + a2 + b2 + 1 = (ab + a + b + 1) (ab + 1)

2a2 b2 + 2a2 + 2b2 + 2 = a2 b2 + a2 b + ab2 + +2ab + a + b + 1




a2 b2 b + 2 a b2 + 2b + 1 + 2b2 b + 1 = 0,

que pode ser considerada uma equac


ao do 2o grau em a cujo discriminante () e


= (b + 1)4 4 b2 b + 2 2b2 b + 1
= 7b4 + 16b3 18b2 + 16b 7.

Esse polin
omio possui duas caractersticas interessantes. A primeira, que n
os n
ao utio
a
lizaremos, e que ele e um polin
omio recproco de 4 grau e de 1 especie, pois a leitura
de seus coeficientes da esquerda para direita coincide com a leitura feita da direita para a
esquerda. A segunda e que b = 1 e uma raiz ja que o valor 1 zera o . Isso nos leva a
escrever
= 7b4 + 7b3 + 9b3 9b2 9b2 + 9b + 7b 7

= (b 1) 7b3 + 9b2 9b + 7 .

Novamente, o segundo fator desse u


ltimo produto e um polin
omio recproco de 3o grau,
a
mas de 2 especie, ja que as leituras dos coeficientes nos dois sentidos s
ao simetricas. Alem

3
2
disso, b = 1 e novamente uma raiz e, escrevendo 7b +9b 9b+7 = (b1) 7b2 + 2b 7 ,
obtemos

= (b 1)2 7b2 + 2b 7 .

O discriminante de 7b2 + 2b 7 e negativo e, portanto, 7b2 + 2b 7 < 0, b. Como


(b 1)2 0, b, segue que 0, b. Para a R, devemos ter = 0 e, portanto, b = 1 e
a = 1, que e a u
nica soluc
ao.
Problema 4. (Cro
acia) Encontre todas as solucoes inteiras da equacao

4x + y + 4 xy 28 x 14 y + 48 = 0.
Problema 5. Mostre que x2 y 2 = a3 sempre tem solucao inteira (x, y), dado que a Z.
2


POT 2012 - Algebra
- Nvel 2 - Aula 2 - Prof. Marcelo Mendes
Problema 6. Prove que se os coeficientes de uma equacao quadr
atica ax2 + bx + c s
ao
inteiros mpares, ent
ao as razes da equacao n
ao podem ser n
umeros racionais.
Problema 7. Se x e y s
ao reais tais que
x + y = 0.




p

x + x2 + 1 y + y 2 + 1 = 1, prove que

Problema 8. Para quais n


umeros reais a, b, c (a 6= 0, b 6= 0, c 6= 0, a + b + c 6= 0) vale a
1 1 1
1
igualdade + + =
.
a b
c
a+b+c
Problema 9. Sejam a, b, c, d inteiros distintos tais que a equacao
(x a)(x b)(x c)(x d) 4 = 0
possui uma raiz inteira r. Mostre que 4r = a + b + c + d.
Problema 10. (EUA) Se 1

4
4
2
+ 2 = 0, determine o valor de .
x x
x

Problema 11. (EUA) Se ab 6= 0 e |a| =


6 |b|, quantos valores distintos de x satisfazem a
b
a
xa xb
+
=
+
?
equacao
b
a
xa xb

2 Sistemas de equa
c
oes
Vamos iniciar com um problema da 1a fase do nvel 2 da XXI OBM.
Problema 12. (OBM) Rafael tem 23 da idade de Roberto e e 2 anos mais jovem que Reinaldo. A idade de Roberto representa 34 da idade de Reinaldo. Determine a soma em anos
das idades dos tres.
Soluc
ao. Sejam a, o, e as idades de Rafael, Roberto e Reinaldo, respectivamente. Assim,
a = 32 o, a = e 2 e o = 43 e. Da, a = 23 34 e = e 2, o que d
a e = 18. Portanto, a = 16,
o = 24 e a + o + e = 58.
Problema 13. (EUA - Adaptado) Determine todas as triplas ordenadas distintas (x, y, z)
de n
umeros inteiros satisfazendo o sistema de equacoes

x + 2y + 4z = 12
xy + 4yz + 2zx = 22 .

xyz = 6

+
2y
+
4z
= 12
x
Soluc
ao. Podemos reescrever o sistema como x 2y + 2y 4z + x 4z = 44 .

2y 4z = 48
3


POT 2012 - Algebra
- Nvel 2 - Aula 2 - Prof. Marcelo Mendes
Fazendo x = x , 2y = y e 4z = z , chegamos a

+ y +
z
= 12
x

xy + yz + xz
= 44 .

x y z = 48

Assim, x , y , z s
ao razes da equac
ao t3 12t2 + 44t 48 = 0 (verifique!), que possui 2
como raiz. Da, podemos reescreve-la como
t3 2t2 10t2 + 20t + 24t 48 = 0

(t 2) t2 10t + 24 = 0
(t 2)(t 4)(t 6) = 0,

que gera as soluc


oes 2, 4, 6. Assim, 4z = 4 e z = 1. Alem disso, x = 2 e y = 3 ou x = 6 e
y = 1.
Problema 14. (URSS) Encontre todas as solucoes inteiras (x, y, z, t) do sistema

xz 2yt = 3
.
xt + yz = 1
Soluc
ao. Nas duas equac
oes, aparecem as 4 letras exatamente uma vez. Assim, podemos eleva-las ao quadrado e somar o resultado da primeira com o dobro do da segunda,
eliminando o produto xyzt
(xz)2 + 2(xt)2 + 4(yt)2 + 2(yz)2 = 11


x2 z 2 + 2t2 + 2y 2 z 2 + 2t2 = 11


x2 + 2y 2 z 2 + 2t2 = 11.
 2
 2
x + 2y 2 = 1
x + 2y 2 = 11
Temos as seguintes possibilidade
ou
.
z 2 + 2t2 = 11
z 2 + 2t2 = 1
No primeiro, temos x2 = 1, y 2 = 0 e z 2 = 9, t2 = 1. No segundo, x2 = 9, y 2 = 1
e
= 1, t2 = 0. Substituindo nas equacoes iniciais, obtemos as solucoes (x, y, z, t) =
(1, 0, 3, 1), (1, 0, 3, 1), (3, 1, 1, 0), (3, 1, 1, 0).
z2

Problema 15. (Bielorr


ussia) Determine todas as solucoes reais do sistema (n 2):

x1 + x2 + ... + xn1

x + x + ... + x

2
3
n
..

xn + x1 + ... + xn2

=
=
=
=

1
xn
1
x1

..
.

1
xn1

Soluc
ao. Inicialmente, observe que x1 + x2 + ... + xn = xk + x1k (), k {1, 2, ..., n} e que
todos os xk s
ao n
ao-nulos. Tomando duas equacoes quaisquer, obtemos xi + x1i = xj + x1j ,
4


POT 2012 - Algebra
- Nvel 2 - Aula 2 - Prof. Marcelo Mendes

cujas solucoes s
ao xi = xj ou xi =

1
xj .

Supondo a segunda possibilidade e substituindo na equacao do sistema original em que


o lado direito e x1j , chegamos a x1 + ... + x
i + ... + x
j + ... + xn = 0 (a notacao x
i significa
que xi foi suprimido da soma), o que e impossvel ja que (*) garante que os xk s
ao todos
positivos ou todos negativos.
Assim, s
o nos resta a opc
ao em que todos os xk s
ao iguais, digamos a . Substi1
tuindo em qualquer uma das equac
oes, obtemos (n 1) = 1 , ou seja, xk = n1
, k ou

1
, k.
xk = n1

Problema 16. Resolva o sistema de equacoes

x + y + z = 2
x2 y 2 z 2 = 2 .

x 3y 2 + z = 0

Problema 17. (IMTS) O conjunto S e formado por 5 inteiros. Se os elementos de S s


ao somados aos pares, obtemos 1967, 1972, 1973, 1974, 1975, 1980, 1983, 1984, 1989, 1991. Quais
s
ao os elementos de S?
Problema 18. (EUA) Resolva o sitema de equacoes

2x1 + x2 + x3 + x4

x1 + 2x2 + x3 + x4
x1 + x2 + 2x3 + x4

x
+ x2 + x3 + 2x4

1
x1 + x2 + x3 + x4

+
+
+
+
+

x5
x5
x5
x5
2x5

Problema 19. Mostre que o sistema

n
ao possui soluc
oes reais (x, y, z).
Problema 20. Mostre que a u
nica

x1

x2

x3

1
x+ x
1
y+y

z + z1

= y
= z
= x

solucao do sistema
+
+
+

+
+
+

x2
x3
x4
..
.

x3
x4
x5

= 0
= 0
= 0
.
= .. .

x98 + x99 + x100 = 0

x
+ x100 + x1 = 0

99
x100 + x1 + x2 = 0
5

=
=
=
=
=

6
12
24 .
48
96


POT 2012 - Algebra
- Nvel 2 - Aula 2 - Prof. Marcelo Mendes

e x1 = x2 = ... = x99 = x100 = 0.


Problema 21. (EUA) Quatro inteiros positivos a, b, c, d

ab + a + b =
bc + b + c =

cd + c + d =

tem produto igual a 8! e satisfazem


524
146
104

Quanto vale a d?

Problema 22. (EUA) Quantas triplas ordenadas (x, y, z)


de equacoes abaixo?
2
3xy + 2y 2

x
2
x
+ 6yz +
2
x
+ xy
+
Problema 23. (Iberoamericana) Ache todas

+ y
x
2
x
y2
3
x + y 3

de inteiros satisfazem o sistema


z 2 = 31
2z 2 = 44
8z 2 = 100

a triplas de n
umeros reais (x, y, z) tais que
z = 1
+ z2 = 1
+ z 3 = 1

Problema 24. (Romenia) Os n


umeros reais n
ao nulos x, y, z, t verificam as seguintes equacoes

t
x + y + z =
1
1
1
1
+ y + z =
t
x3
x + y 3 + z 3 = 10003

Determine o valor da soma x + y + z + t.

Problema 25. (OCM) Determine a + b + c + d, se

= 3840
6a + 2b
6c + 3d = 4410 .

a + 3b
+ 2d = 3080

Problema 26. (OBM/IME) Sejam a, b, c e k n


umeros reais diferentes de zero satisfazendo
a
b
c
as relacoes k = b+c = c+a = a+b . Qual e o n
umero de possveis valores que k pode assumir?

Problema 27. (OBM) Determine o n


umero de solucoes inteiras e positivas do sistema

a + b
= c2
a + b + c = 30
Problema 28. (OBM) As letras O, B, M representam n
umeros inteiros. Se O B M =
240, O B + M = 46 e O + B M = 64, quanto vale O + B + M ?
6


POT 2012 - Algebra
- Nvel 2 - Aula 2 - Prof. Marcelo Mendes

Problema 29. (OBM) Sejam a, b, c n


umeros reais positivos tais que a(b+c) = 152, b(c+a) =
162 e c(a + b) = 170. Determine o valor de abc.
Problema 30. (OBM) Quantos pares ordenados (x, y) de n
umeros reais satisfazem a equacao
2
x y 2 + (x y 2)2 = 0.

Problema 31. (OBM) Os inteiros 0 < x < y < z < w < t s


ao tais que w = z(x + y) e
t = w(y + z). Sendo w = 9, determine o valor de t.
Problema 32. (EUA) Se x e y s
ao n
umeros reais n
ao-nulos tais que x = 1 +
entao y e igual a:
a) x 1
b) 1 x
c) 1 + x
d) x
e) x

1
1
e y = 1+ ,
y
x


POT 2012 - Algebra
- Nvel 2 - Aula 2 - Prof. Marcelo Mendes

Dicas
4. Fatore o lado esquerdo da equac
ao. Comece escrevendo a soma dos 3 primeiros termos
como o quadrado da soma de dois termos.
5. Observe que o problema n
ao pede todas as solucoes dessa equacao. Assim, fatore o
lado esquerdo e faca x + y = a2 e x y = a.
6. Use a definic
ao: se x Q, ent
ao existem p, q Z, q 6= 0 tais que x = pq . Se for
necessario, acrescente que x e y s
ao primos entre si. Com essa u
ltima observacao, as
paridades de p e q s
o n
ao podem ser ambas pares. Utilize o fato de que 0 e par para
chegar a contradic
oes em todos os casos.
7. Passe o primeiro fator para o lado direito e racionalize (ou entao, racionalize mesmo
na equac
ao inicial). Depois, faca o mesmo com o segundo fator.
8. Para equilibrar a equac
ao, passe 1c para o lado direito. Em seguida, reduza a um
denominador comum em cada lado. Analise, em seguida, as possibilidades de os
n
umeros serem ou n
ao iguais a 0.
9. Use a definic
ao: se r e raiz da equacao em x, entao substituindo x pelo valor r
a equac
ao fica verdadeira. Depois, escreva 4 como produto de 4 n
umeros inteiros
distintos.
16. Combine as equac
oes 1 e 3.
18. Some todas as equac
oes, que nos dar
a a soma de todas os xi s. Depois, subtraia cada
uma desse resultado.
19. Some todas as equac
oes.
20. Subtraia as equac
oes aos pares.
21. Some 1 a cada membro de cada equacao e use a fatoracao xy+x+y+1 = (x+1)(y+1).
22. Some todas as equac
oes e perceba soma de quadrados.
23. Subtraia as equac
oes aos pares.
24. Veja o problema 8.
26. Escreva a = k(b + c), b = k(c + a), c = k(a + b) e some todas as equacoes em seguida.
28. Multiplique a segunda equac
ao por M e a terceira por O.
29. Some todas as equac
oes.
30. Se a soma dos quadrados de dois n
umeros reais e 0, entao os dois n
umeros s
ao iguais
a 0.


POT 2012 - Algebra
- Nvel 2 - Aula 2 - Prof. Marcelo Mendes

Respostas
4. (0, 36), (1, 16), (4, 4), (9, 0), (0, 64), (1, 36), (4, 16), (9, 4), (16, 0)
8. a = b ou b = c ou c = a

9. 1

10. 3
16. (2, 1, 1),

1
19 2
12 , 3 , 4

17. S = {983, 984, 989, 991, 1000}

18. (x1 , x2 , x3 , x4 , x5 ) = (25, 19, 7, 17, 65)

21. 10
22. 0

23. (1, 1, 1), (1, 1, 1)

24. 2000
25. 1985
26. 2
27. 24

1
2 ; 1

28. 20
29. 720
30. 2
31. 45
32. e

Polos Olmpicos de Treinamento


Aula

Curso de lgebra - Nvel 2


Prof. Marcelo Mendes

Recorr
encias - Parte I
Na aula anterior, vimos alguns exemplos de sequencias. Em alguns deles, os termos s
ao
dados em func
ao de termos anteriores, ou seja, eles recorrem a valores de termos anteriores.
Por isso, essas sequencias s
ao chamadas de recorr
encias.
Talvez os exemplos mais cl
assicos de sequencias recorrentes sejam as progressoes aritmetica
e geometrica, que veremos neste texto.

1 Progress
oes Aritm
eticas
O problema 6 da aula anterior e um exemplo de P.A. Por definicao, uma P.A. e uma
sequencia em que a diferenca entre os termos consecutivos e constante. Da, se (a, b, c) e
uma P.A., ent
ao b a = c b, ou ent
ao, 2b = a + c, isto e, b = a+c
2 , ou seja, cada termo de
uma P.A. e a media aritmetica dos termos adjacentes. Essa propriedade, portanto, justifica
o nome desse tipo de sequencia.
Sendo d o valor da diferenca constante (tradicionalmente chamada de raz
ao), temos a
seguinte lei de formac
ao para os termos de uma P.A. {an }
an = an1 + d.
Mas veja que essa e uma formula implcita, recorrente, que necessita de valores anteriores
para se achar o valor de um determinado termo. Somando telescopicamente varias dessas
equacoes
an = an1 + d
an1 = an2 + d
:
a3 = a2 + d
a2 = a1 + d


POT 2012 - Algebra
- Nvel 2 - Aula 4 - Prof. Marcelo Mendes

chegamos a
an = a1 + (n 1)d,
que e a formula cl
assica para o termo geral de uma P.A. Todavia, pode ser mais interessante
em determinados problemas a formula
an = am + (n m)d an am = (n m)d,
que, ao inves de depender do valor do termo a1 , calcula an a partir de qualquer outro termo
am , podendo este, inclusive, ser posterior.
Essa formula nos permite concluir que a1 + an = a2 + an1 = a3 + an2 = .... Da,
somando as duas equac
oes a seguir
S = a1 + a2 + ... + an1 + an
S = an + an1 + ... + a2 + a1
chegamos a
S=

(a1 + an ) n
.
2

Problema 1. (EUA) Os quatro primeiros termos de uma P.A. s


ao a, x, b, 2x. Determine o
a
valor da raz
ao .
b
a+b
2b
a
1
Soluc
ao. Temos 2x = a + b e 2b = x + 2x. Assim,
=
e, portanto, = .
2
3
b
3
Problema 2. (IME) Determine a relac
ao que deve existir entre os n
umeros m, n, p, q para
que se verifique a seguinte igualdade entre os termos de uma mesma progressao aritmetica
n
ao-constante:
am + an = ap + aq .
Problema 3. Encontre o valor de a2 + a4 + a6 + ... + a98 se a1 , a2 , a3 , ... e uma P.A. de raz
ao
1 e a1 + a2 + a3 + ... + a98 = 137.
Soluc
ao. Podemos escrever a1 +a2 +...+a97 +a98 = 137 como (a2 1)+a2 +...+(a98 1)+
a98 = 137. Da, 2 (a2 + a4 + a6 + ... + a98 ) 49 = 137 e, portanto, a2 + a4 + a6 + ... + a98 =
137 + 49
= 93.
2
Problema 4. (EUA) Seja a1 , a2 , ..., ak uma progressao aritmetica finita com a4 + a7 + a10 =
17 e a4 + a5 + a6 + ... + a12 + a13 + a14 = 77. Se ak = 13, determine o valor de k.
Problema 5. Calcule a soma dos 1000 primeiros m
ultiplos positivos de 7.


POT 2012 - Algebra
- Nvel 2 - Aula 4 - Prof. Marcelo Mendes

Problema 6. Um jardineiro tem que regar 60 roseiras plantadas ao longo de uma vereda
retilnea e distando 1m uma da outra. Ele enche seu regador, a 15m da primeira roseira, e,
a cada viagem, rega 3 roseiras. Comecando e terminando na fonte, qual e o percurso total
que ele ter
a que caminhar ate regar todas as roseiras?
Problema 7. Observe a disposic
ao, abaixo, da seq
uencia dos n
umeros naturais mpares.
1a
2a
3a
4a
5a
:

linha
linha
linha
linha
linha
:

1
3, 5
7, 9, 11
13, 15, 17, 19
21, 23, 25, 27, 29
:

Determine o quarto termo da vigesima linha.


Problema 8. (Espanha) Encontre uma P.A. tal que a soma de seus n primeiros termos seja
igual a n2 para qualquer valor de n.
Soluc
ao. Veja que
Sn = a1 + a2 + ... + an = n2 .
Com n = 1, obtemos S1 = a1 = 1 e, com n = 2, S2 = a1 + a2 = 4. Logo, a2 = 3. Assim,
a raz
ao da P.A. e a2 a1 = 3 1 = 2. Portanto, a P.A. procurada e 1, 3, 5, 7, ...
Problema 9. (IME) O quadrado de qualquer n
umero par 2n pode ser expresso como a soma
de n termos, em progress
ao aritmetica. Determine o primeiro termo e a raz
ao da progressao.
Problema 10. (ITA) Provar que se uma P.A. e tal que a soma dos seus n primeiros termos
e igual a n + 1 vezes a metade do n-esimo termo, entao r = a1 .
Soluc
ao. Pelo enunciado, temos
Sn = (n + 1)

(a1 + an ) n
an
an

= (n + 1)
a1 n = an
2
2
2

a1 n = a1 + (n 1)r a1 (n 1) = (n 1)r, n.
Portanto, a1 = r.
m2
Sm
= 2 , sendo Sm e Sn as somas dos m primeiros
Sn
n
termos e dos primeiros n termos, respectivamente, com m 6= n. Prove que a raz
ao da P.A.
e o dobro do primeiro termo.
Problema 11. Numa P.A., tem-se


POT 2012 - Algebra
- Nvel 2 - Aula 4 - Prof. Marcelo Mendes

Problema 12. Se numa P.A. a soma dos m primeiros termos e igual `a soma dos n primeiros
termos, m 6= n, mostre que a soma dos m + n primeiros termos e igual a zero.
Problema 13. (OCM) Mostre que
gressao aritmetica.


2, 3, 5 n
ao podem ser termos de uma mesma pro-

Problema 14. Cada uma das progressoes aritmeticas a seguir tem 80 termos: (an ) =
(9, 13, ...) e (bn ) = (10, 13, ...). Quantos n
umeros s
ao, ao mesmo tempo, termos das duas
progressoes?
Problema 15. Numa P.A., temos ap = q e aq = p, com p 6= q. Determine a1 e ap+q .
Problema 16. (EUA) Se a soma dos 10 primeiros termos e a soma dos 100 primeiros termos de uma progress
ao aritmetica s
ao 100 e 10, respectivamente, determine a soma dos
110 primeiros termos.
Soluc
ao. Vamos escrever os dados do problema da seguinte forma
(a1 + ... + a10 ) + (a11 + ... + a20 ) + ... + (a91 + ... + a100 ) = 10
(a1 + ... + a10 ) + (a1 + ... + a10 ) + ... + (a1 + ... + a10 ) = 100 10
Subtraindo termo a termo, obtemos
0 10 + 10r 10 + ... + 90r 10 = 900
1
100r(1 + ... + 9) = 900 r = .
5
Portanto
a1 + ... + a110 = (a1 + ... + a100 ) + (a101 + ... + a110 )
= 10 + [(a1 + 100r) + ... + (a10 + 100r)]
= 10 + (a1 + ... + a10 ) + 1000r = 10 + 100 200 = 90.

Problema 17. (EUA) Em uma P.A., a soma dos 50 primeiros termos e 200 e a soma dos
50 pr
oximos e 2700. Determine a raz
ao e o primeiro termo dessa seq
uencia.


POT 2012 - Algebra
- Nvel 2 - Aula 4 - Prof. Marcelo Mendes

Problema 18. (EUA) A soma dos n primeiros termos de uma P.A. e 153 e a raz
ao e 2. Se
o primeiro termo e um inteiro e n > 1, determine o n
umero de valores possveis de n.
(a1 + an ) n
= 153, temos [a1 + (n 1)] n = 153. Como a1 + (n 1) e n
2
s
ao inteiros positivos, eles s
ao divisores positivos de 153. Mas 153 = 32 17 e, portanto,
153 possui 6 divisores positivos, sendo 5 deles maiores que 1.

Soluc
ao. Como

Problema 19. (EUA) A soma dos n primeiros termos de uma P.A. e x e a soma dos n
seguintes e y. Calcular a raz
ao.
Problema 20. A sequencia 1, 2, 1, 2, 2, 1, 2, 2, 2, 1, 2, 2, 2, 2, 1, 2, ... consiste de 1s separados
por blocos de 2s, com n 2s no n-esimo bloco. Determine a soma dos 1234 primeiros termos
dessa seq
uencia.
2006

Problema 21. Mostre que 20082007

e um termo da P.A. infinita (6, 13, 20, 27, ...).

Problema 22. (EUA) Os tres primeiros termos de uma progressao aritmetica s


ao 2x
3, 5x 11 e 3x + 1, respectivamente. O n-esimo termo da sequencia e 2009. Quel e o valor
de n?
Problema 23. (EUA) Os quatro primeiros termos de uma progressao aritmetica s
ao p, 9, 3p
q e 3p + q. Qual e o 2010o termo dessa sequencia?

2 Progress
ao Geom
etrica
Semelhante ao que escrevemos para P.A., por definicao, uma P.G. e uma sequencia em
que cada novo termo, a partir do segundo, e o produto do termo anterior por uma constante. Da, se (a, b, c) e uma P.G., ent
ao b2 = ac.
Sendo q o valor da raz
ao constante, temos a seguinte lei de formacao para os termos de
uma P.G. {an }
an = an1 q.
Mas veja que essa tambem e uma formula implcita, recorrente, que necessita de valores
anteriores para se achar o valor de um determinado termo. Multiplicando telescopicamente
varias dessas equac
oes


POT 2012 - Algebra
- Nvel 2 - Aula 4 - Prof. Marcelo Mendes

an = an1 q
an1 = an2 q
:
a3 = a2 q
a2 = a1 q
chegamos a
an = a1 q n1 ,
que e a formula cl
assica para o termo geral de uma P.G.
A formula da soma dos n primeiros termos e
qn 1
,
q1
se q 6= 1 e Sn = a1 n, se q = 1, e a formula do produto dos n primeiros termos pode ser
apresentada de 2 maneiras
S n = a1

Pn = an1 q

n(n1)
2

ou
Pn2 = (a1 an )n .
Problema 24. (EUA) Suponha que x, y, z estejam em P.G. de raz
ao r e x 6= y. Se x, 2y, 3z
est
ao em P.A., determine o valor de r.
Soluc
ao. Temos y = x r e z = x r 2 pela P.G. Pela P.A., segue que 4y = x + 3z. Logo,
4xq = x + 3xq 2 . Se x = 0, ent
ao y = 0 = x, o que n
ao pode ocorrer. Da, 3q 2 4q + 1 = 0,
1
cujas solucoes s
ao q = 1 e q = 3 . Como q = 1 implica x = y, conclumos que q = 31 .
Problema 25. Se (a, b, c) formam, nesta ordem, uma P.A. e uma P.G. simultaneamente,
mostre que a = b = c.
a+c
Soluc
ao. Por ser P.A., temos b =
(*) e, por ser P.G., b2 = ac. Logo,
2
ou seja, (a c)2 = 0. Assim, a = c e, por (*), a = b = c.

a+c
2

2

Problema 26. (OCM) Determine a soma dos n primeiros termos da sequencia:






1, (1 + 2), 1 + 2 + 22 , 1 + 2 + 22 + 23 , ..., 1 + 2 + 22 + 23 + ... + 2k1 .
6

= ac,


POT 2012 - Algebra
- Nvel 2 - Aula 4 - Prof. Marcelo Mendes

Problema 27. 6. Mostre que n


ao existe P.G. de tres termos distintos tal que, ao somarmos
um mesmo n
umero real n
ao-nulo a todos os seus termos, a nova sequencia seja tambem
uma P.G.
Problema 28. (EUA) Numa P.G. de 2n termos, a soma dos termos de ordem par e P e a
soma dos termos de ordem mpar e I. Calcule o 1o termo e a raz
ao.
Soluc
ao. De a2 + ... + a2n = P , segue que q (a1 + ... + a2n1 ) = P ou q I = P . Logo,
n
q2 1
q 2n+1 q
(P I)I 2n
P
= a1
. Logo, a1 = 2n
.
q = . Alem disso, P = a2
I
q1
q1
P I 2n
Problema 29. Prove que, quando os lados de um tri
angulo est
ao em P.G., o mesmo ocorre
para as alturas.

Problema 30. Sejam a, b, c n


umeros reais n
ao-nulos, com a 6= c, tais que
Prove que a, b e c formam uma P.G.

a
a 2 + b2
= 2
.
c
c + b2

Problema 31. (EUA) O 5o e o 8o termos de uma progressao geometrica de n


umeros reais
s
ao 7! e 8!, respectivamente. Qual e o 1o termo?

2 Recorr
encias Lineares de Ordem 2 - Parte I
Por fim, vamos estudar apenas as recorrencias em que a equaca
o caracterstica possui
raiz real dupla. Mas o que e uma equacao caracterstica? Vejamos.
Considere a recorrencia linear de ordem 2 (isto e, s
o depende dos 2 termos imediatamente anteriores)
an = pan1 + qan2 .
A equacao caracterstica dessa recorrencia e a equacao quadr
atica formada repetindo
os mesmos coeficientes da recorrencia, ou seja,
x2 = px + q x2 px q = 0.
Mas como surge essa equac
ao? A resposta ser
a dada no texto da aula seguinte. Por
enquanto, acredite.
Como exemplo, considere uma recorrencia definida por a1 = 1, a2 = 3 e, para n 3,
an = 2an1 an2 . A equac
ao caracterstica associada e x2 2x + 1 = 0, que possui duas
razes iguais a 1. Entrementes, uma olhadinha mais cuidadosa mostra que a recorrencia
em questao e de uma P.A. pois
7


POT 2012 - Algebra
- Nvel 2 - Aula 4 - Prof. Marcelo Mendes

an = 2an1 an2 an an1 = an1 an2 .


Portanto, acabamos de ver que uma P.A. est
a associada a uma equacao caracterstica
com raiz dupla 1.
Agora, vejamos outro exemplo, uma recorrencia em que a1 = 6, a2 = 27 e, para n 3,
an = 6an1 9an2 (*). A equac
ao caracterstica associada e x2 6x + 9 = 0, cujas
razes s
ao iguais a 3. A sada agora e criar uma nova sequencia {bn } dada por an = 3n bn .
Substituindo em (*), chegamos a bn = 2bn1 bn2 , o que mostra que {bn } e uma P.A.!
Assim, sendo bn = A + Bn (o termo geral de uma P.A. e uma funcao polinomial do 1o grau
em funcao de n ou uma func
ao constante no caso em que a P.A. e constante), obtemos
an = 3n (A + Bn).
Para acharmos A e B, fazemos n assumir os valores 1 e 2:

6 = a1 = 3(A + B)
27 = a2 = 9(A + 2B)
cujas solucoes s
ao A = B = 1 e, portanto,
an = 3n (n + 1).
Problema 32. Resolva a recorrencia a1 = 4, a2 = 20 e, para n 3, an = 4an1 4an2 .
Problema 33. Resolva a recorrencia a1 = 8, a2 = 96 e, para n 3, an = 8an1 16an2 .
Problema 34. Considere a sequencia (an ) dada por a1 = 1, a2 = 3 e an = 10an1 25an2 ,
para n > 2. Determine o valor de k, dado por an = kn bn tal que a sequencia (bn ) seja uma
P.A.
Problema 35. (IME) Considere a sequencia {vn }, n = 0, 1, 2, ... definida a partir de seus
dois primeiros termos v0 e v1 e pela formula geral vn = 6vn1 9vn2 , para n 2. Define-se
uma nova sequencia {un }, n = 0, 1, 2, ... pela formula vn = 3n un .
a) Calcule un un1 em func
ao de u0 e u1 .
b) Calcule un e vn em func
ao de n, v1 e v0 .
1
c) Identifique a natureza das sequencias {vn } e {un } quando v1 = 1 e v0 = .
3


POT 2012 - Algebra
- Nvel 2 - Aula 4 - Prof. Marcelo Mendes

Dicas

2. Use ai aj = (i j)r, sendo r a raz


ao.
9. Veja o problema 8.

13. Suponha, sem perda de generalidade que 2, 3, 5 sejam o primeiro, o m-esimo e
o n-esimo termos, respectivamente. Use a formula do termo geral
em
am e an , isole a
raz
a
o
em
cada
uma
e
iguale
essas
express
o
es.
Depois,
utilize
que
2,
3 e, em geral,

k, em que k e um n
umero natural n
ao quadrado perfeito, s
ao n
umeros irracionais.
14. O primeiro termo em comum e 13 e a raz
ao dos termos em comum e mmc(4, 3) = 12,
ja que 3 e 4 s
ao as raz
oes iniciais.
15. Use ai aj = (i j)r, sendo r a raz
ao.
17. Veja a soluc
ao do problema 16 ou use a formula da soma (que dar
a mais trabalho).
19. Veja a sugestao do problema 17.
21. Os termos da P.A. em quest
ao s
ao da forma 7k + 6 ou 7k 1. Assim, basta achar o
2006
na divis
ao por 7.
resto de 20082007
26. Calcule cada uma das somas parciais separadas por vrgulas no enunciado e, em
seguida, calcule a soma dos resultados. Nas duas etapas, use a formula da soma da
P.G.
29. Use que a
area de um tri
angulo e

bh
.
2


POT 2012 - Algebra
- Nvel 2 - Aula 4 - Prof. Marcelo Mendes

Respostas

2. m + n = p + q
4. 18
5. 3503500
6. 1820
7. 387
9. a1 = 4 e r = 8
14. 20
15. a1 = q + p 1, ap+q = 0
17. r = 1 e a1 = 20, 5
19.

yx
n2

20. 2419
22. 502
23. 8041
26. 2n+1 n 2
31. 315
32. 2n (3n 1)
33. 4n (4n 2)
34. 5
35. a) u1 u0 ; b) un = nv3 1 + (1 n)v0 e vn = 3n1 nv1 + 3n (1 n)v0 ; c) un = 13 , sequencia
constante e vn = 3n1 , progress
ao geometrica

10

Polos Olmpicos de Treinamento


Aula

Curso de lgebra - Nvel 2


Prof. Marcelo Mendes

Recorr
encias - Parte II
Na aula 3, falamos de uma sequencia famosa, a Sequencia de Fibonacci, cuja definicao e
a seguinte: F1 = F2 = 1 e, para n 3, Fn = Fn1 + Fn2 . Essa formula e uma recorrencia
linear de ordem 2. Um de nossos objetivos neste 5o texto e mostrar que a formula explcita
para seus termos e
!n
!n
1+ 5
1 5
1
1

.
Fn =
2
2
5
5
Surpreendente, n
ao e mesmo? Imaginar que, substituindo n por 1, 2, 3,
4, 5, 6, ... na
formula acima, acharemos exatamente os termos 1, 1, 2, 3, 5, 8, ..., e nenhum 5 sobra, e
realmente muito belo.
Em geral, nesta aula, trataremos equacoes de recorrencia lineares que dependem somente dos dois termos anteriores. Inicialmente, vamos estudar o caso em que as razes da
equaca
o caracterstica (que definiremos no texto) s
ao distintas.

1 Um Exemplo para Organizar as Ideias


Vamos resolver a recorrencia a1 = 1, a2 = 3 e, para n 3,
an = 3an1 2an2 .
Podemos escrever an an1 = 2 (an1 an2 ) e, em seguida, multiplicar telescopicamente varias delas
an an1 = 2 (an1 an2 )
an1 an2 = 2 (an2 an3 )
:
a3 a2 = 2 (a2 a1 )


POT 2012 - Algebra
- Nvel 2 - Aula 5 - Prof. Marcelo Mendes
obtendo an an1 = 2n2 (a2 a1 ) = 2n1 .
Agora, somamos telescopicamente varias dessa u
ltima equacao
an an1 = 2n1

an1 an2 = 2n2


:
a2 a1 = 2

e chegamos a an a1 = 2 + ... + 2n2 + 2n1 , ou seja, an = 2n 1.


Observe que, na primeira passagem, para transformar an = 3an1 2an2 em an
an1 = 2 (an1 an2 ), pedimos emprestado an1 para o membro esquerdo. Essa operacao
gerou proporc
ao entre os coeficientes dos termos dos dois membros (antes e depois da igualdade), permitiu colocar o fator de proporcao 2 em evidencia e a diferenca que surgiu entre
parenteses no membro direito ficou com o mesmo padrao da diferenca no membro esquerdo,
mas com ndices reduzidos. Essa ser
a nossa ideia para encontrar o termo geral da

2 Sequ
encia de Fibonacci
Como ja definimos anteriormente, seus termos s
ao dados por F1 = F2 = 1 e, para
n 3, Fn = Fn1 + Fn2 . Na verdade, os calculos ficam mais interessantes escrevendo
Fn+1 = Fn +Fn1 . Seria difcil pedir emprestado uma quantidade inteira desta vez pois h
a
somente Fn no membro direito. Assim, vamos chamar de a quantidade que ser
a passada
para o membro esquerdo, ou seja,
Fn+1 Fn = (1 )Fn + Fn1 .
Para repetirmos a ideia bem sucedida do primeiro exemplo, o valor de deve cumprir
a relacao de proporc
ao

ou seja,

1
1
=
,

1
2 1 = 0,

a qual chamaremos de equaca


o caracterstica da sequencia de Fibonacci. Observe desde
ja que os coeficientes dessa equac
ao s
ao os mesmos da recorrencia que define a sequencia.
Sendo 1 e 2 as razes, aqui ser
a mais relevante saber que 1 + 2 = 1 e 1 2 = 1 (mas
veja que ambas s
ao reais e distintas) do que escrever seus valores pela formula de Baskara.
Agora, substituindo por 1 , obtemos
Fn+1 1 Fn = (1 1 )Fn + Fn1 ,
2


POT 2012 - Algebra
- Nvel 2 - Aula 5 - Prof. Marcelo Mendes

ou seja,
Fn+1 1 Fn = 2 (Fn 1 Fn1 ) .
Assim, deixamos a equac
ao pronta para escreve-la varias vezes e fazer o produto telesc
opico
Fn+1 1 Fn = 2 (Fn 1 Fn1 )
Fn 1 Fn1 = 2 (Fn1 1 Fn2 )
:
F3 1 F2 = 2 (F2 1 F1 ) ,
cujo resultado ser
a
Fn+1 1 Fn = 2n1 (F2 1 F1 ) = 2n1 (1 1 ) = n2 .
Analogamente, substituindo por 2 , temos
Fn+1 2 Fn = n1 .
A diferenca entre esses 2 u
ltimos resultados gera
(1 2 ) Fn = n1 n2
e, portanto,
Fn =

n1 n2
1 2

lembrando que 1 6= 2 . Substituindo os valores de 1 e 2 , chegamos ao resultado desejado


!n
!n
1
1+ 5
1 5
1
Fn =

.
2
2
5
5
Mas h
a um pequeno problema. Esse metodo e bastante trabalhoso. A boa notcia e que
podemos deixa-lo como uma quase demonstracao e realizar, na pr
atica, os seguintes passos:
1o passo: Escreva a equac
ao caracterstica.
Basta copiar os mesmos coeficientes da equacao de recorrencia. Em seguida, calcule as
razes dessa equac
ao.
2o passo: Escreva o termo geral da recorrencia.
O termo geral e dado por Fn = An1 + Bn1 (essa formula pode ser encontrada refazendo
os calculos para a recorrencia mais geralmente, ou seja, com a equacao xn = axn1 +bxn2 ).
3


POT 2012 - Algebra
- Nvel 2 - Aula 5 - Prof. Marcelo Mendes
interessante, para reAs constantes A e B s
ao dadas pelos valores dos termos iniciais. E
duzir as contas, calcular o termo de ordem 0, que, no caso da sequencia de Fibonacci, e
F0 = 0.
Vejamos como seria, ent
ao, a resolucao na pr
atica para encontrar o termo geral da
sequencia de Fibonacci.
Passo 1. Equac
ao caracterstica.
De Fn Fn1 Fn2 = 0, obtemos 2 1 = 0, cujas razes s
ao 1 =

1+ 5
2

e 2 =

1 5
2 .

Passo 2. Termos geral.


Fn = An1 + Bn1 . Com os valores 0 e 1 para n, obtemos
0=A+B
1 = A1 + B2
cuja solucao e A = B =

1 .
5

Portanto,
1
Fn =
5

!n
1
1+ 5

2
5

!n
1 5
.
2

Problema 1. Um garoto tem n reais. Todo dia, ele realiza exatamente uma das seguintes
compras: um bolo que custa R$ 1, 00, um sorvete que custa R$ 2, 00 ou um pastel que
tambem custa R$ 2, 00. De quantas maneiras o menino pode gastar seu dinheiro?
Soluc
ao. Seja an o n
umero de maneiras de ele gastar os n reais.
Assim, para gastar os u
ltimos reais, ou ele gasta n 1 reais primeiramente e compra
um bolo no final, ou ele gasta n 2 reais inicialmente e, em seguida, compra um sorvete
ou um pastel. Portanto, podemos escrever
an = an1 + 2an2 ,
com a1 = 1 (s
o d
a pra comprar 1 bolo) e a2 = 3 (comprando 2 bolos ou 1 sorvete ou 1
pastel).
Agora, vamos resolver.
i) Equacao caracterstica: 2 2 = 0, cujas razes s
ao 2 e 1.
4


POT 2012 - Algebra
- Nvel 2 - Aula 5 - Prof. Marcelo Mendes

ii) Termos geral: an = A 2n + B (1)n . Podemos calcular a0 , que n


ao faz sentido para
o gasto do dinheiro, mas existe na sequencia associada: a2 = a1 + 2a0 a0 = 1. Agora,
para n = 0 e n = 1
A+B =1

cuja solucao e A =

2
3

e B = 13 . Assim

2A B = 1,

an =

2n+1 + (1)n
.
3

Problema 2. Determine o termo geral da sequencia definida pela recorrencia a1 = 1, a2 = 4


e an = 4an1 3an1 para n 3.
Problema 3. Determine o termo
geral da sequencia definida recorrentemente por a0 = 0,

a1 = 3 e, para n 3, an = 5an1 + an1 .


Problema 4. Considere um ret
angulo 1 n, que deve ser preenchido por dois tipos de
ret
angulos menores 1 1 e 1 2. De quantas maneiras se pode fazer isso?
Problema 5. (OPM) Uma escada tem n degraus. Para subi-la, em cada passo, pode-se
subir um ou dois degraus de cada vez. De quantos modos diferentes pode-se subir a escada?
Problema 6. Uma sequencia de n
umeros ak e definida por a0 = 0 e ak+1 = 3ak + 1, k 0.
Prove que a155 e divisvel por 11.
Soluc
ao. Inicialmente, veja que essa recorrencia n
ao depende dos dois termos anteriores.
A parcela 1 no membro da direita, na verdade, n
ao e bem-vinda. Assim, de
ak+1 = 3ak + 1
ak = 3ak1 + 1
obtemos ak+1 4ak + 3ak1 = 0. O termo geral dessa recorrencia e an =
monstracao deixamos para o leitor).
Logo, a155 =

3n 1
(a de2

3155 1
. Para finalizar, deixo como sugestao que 35 1 = 242 = 11 22.
2

3
21
e 2an 3an1 = n+1 , n 2.
Problema 7. Seja {an } uma sequencia tal que a1 =
16
2
Encontre o valor de a2 e a lei de recorrencia de cada termo em funcao dos dois termos
imediatamente anteriores.


POT 2012 - Algebra
- Nvel 2 - Aula 5 - Prof. Marcelo Mendes

3 Recorr
encias e Equac
oes do 2o Grau
Como exemplo para organizar as ideias, vamos supor que seja uma raiz da equacao
x2 + x 1 = 0. Assim
2 = + 1.
Da,
3 = 2 + = 2 1

4 = 22 = 3 + 2

5 = 32 + 2 = 5 3.
Ser
a que existe um padrao entre os coeficientes que aparecem no lado direito de cada
potencia de ? Sim, existe! Na pr
oxima aula, que ser
a sobre induca
o finita, estaremos
aptos a provar que
n = (1)n1 Fn + (1)n Fn1 ,
sendo {Fn } a sequencia de Fibonacci.
Problema 8. Se e s
ao as razes da equacao ax2 + bx + c = 0 e Sn = n + n , n N,
entao mostre que aSn+1 + bSn + cSn1 = 0.
Soluc
ao. Como e s
ao as razes de ax2 + bx + c = 0, entao
a2 + b + c = 0
a 2 + b + c = 0.
Da, multiplicando por n1 e n1 , respectivamente, temos
an+1 + bn + cn1 = 0
a n+1 + b n + c n1 = 0.
Somando, obtemos

ou seja,



a n+1 + n+1 + b (n + n ) + c n1 + n1 = 0
aSn+1 + bSn + cSn1 = 0.

Problema 9. Seja a maior raiz de x2 + x 1 = 0. Determine o valor de 5 5.


POT 2012 - Algebra
- Nvel 2 - Aula 5 - Prof. Marcelo Mendes
n n
, n = 1, 2, 3, ....

Determine os dois primeiros termos a1 e a2 dessa sequencia e a lei de recorrencia de cada


termo em func
ao dos dois termos imediatamente anteriores.
Problema 10. Sejam e as razes de x2 + x 1 = 0. Sendo an =


POT 2012 - Algebra
- Nvel 2 - Aula 5 - Prof. Marcelo Mendes

Dicas

2. Use a equac
ao caracterstica e encontre o termo geral seguindo o exemplo e a questao
1.
3. Use a equac
ao caracterstica e encontre o termo geral seguindo o exemplo e a questao
1.
4. Para finalizar, ou ele completa com um quadradinho 1 1 o ret
angulo 1 (n 1),
que pode ser preenchido de an1 maneiras, ou ele completa com um ret
angulo 1 2
o ret
angulo 1 (n 2), que pode ser preenchido de an2 maneiras.
5. Para finalizar, ou ele sobe um degrau a partir do degrau n 1, que pode ser alcancado
de an1 maneiras, ou ele sobe dois degraus a partir do degrau n 2, que pode ser
alcancado de an2 maneiras.
3
7. Multiplique a equac
ao de recorrencia por 2 e subtraia de 2an1 3an2 = n , que e
2
a equac
ao dada substituindo n por n 1.
10. Se a equac
ao caracterstica e x2 + x 1 = 0, entao a equacao de recorrencia e
an = an1 + an2 .

Respostas

3n 1
2
!n

5+3
3. an =

2. an =

53
2

!n

4. Sendo an o n
umero de maneiras, a1 = 1, a2 = 2, an = an1 + an2
5. Sendo an o n
umero de maneiras, a1 = 1, a2 = 2, an = an1 + an2
7. a2 =

69
32

e 4an 8an1 + 3an2

9. 3
10. a1 = 1 e a2 = 1; an = an1 + an2

Polos Olmpicos de Treinamento


Aula

Curso de lgebra - Nvel 2


Prof. Marcelo Mendes

Indu
c
ao - Parte I
O verbo induzir significa gerar. Nesta aula, comecaremos a ver o assunto Inducao Matem
atica (ou Induc
ao Finita ou Princpio da Inducao Finita), que e um metodo de prova
envolvendo n
umeros inteiros que aproveita o trabalho feito na demonstracao de casos anteriores para se provar o fato para um inteiro maior.
Como assim? Vejamos um exemplo.

1 Um Exemplo para Organizar as Ideias


Considere mais uma vez a sequencia de Fibonacci definida por F1 = F2 = 1 e, para
n 3, Fn = Fn1 +Fn2 , ou seja, seus primeiros termos s
ao 1, 1, 2, 3, 5, 8, 13, 21, 34, 55, 89, ...
Vamos mostrar a seguinte identidade
F1 + F2 + ... + Fn = Fn+2 1.
Comecamos verificando para valores pequenos de n:
i. n = 1 : F1 = F3 1 1 = 2 1
ii. n = 2 : F1 + F2 = F4 1 1 + 1 = 3 1
iii. n = 3 : F1 + F2 + F3 = F5 1 1 + 1 + 2 = 5 1
Mas o que o metodo da induc
ao procura fazer e realizar a prova a partir de casos anteriores. Vamos fazer isso com as identidades acima.
Comecando com
F1 = F3 1,
somamos F2 a cada membro da equac
ao e obtemos
F1 + F2 = F2 + F3 1 = F4 1.


POT 2012 - Algebra
- Nvel 2 - Aula 6 - Prof. Marcelo Mendes

Observe que primeiro obtemos o lado esquerdo desejado e, s


o depois, organizamos o
lado direito. Prosseguindo, partimos de
F1 + F2 = F4 1,
somamos F3 a cada membro e chegamos a
F1 + F2 + F3 = F3 + F4 1 = F5 1,
ou seja, repetimos a mesma operac
ao anterior. Isso pode ser feito de maneira geral, supondo
que ja tenhamos chegado a
F1 + F2 + ... + Fk = Fk+2 1.
Em seguida, somamos Fk+1 a cada membro da equacao e conclumos que
F1 + F2 + ... + Fk + Fk+1 = Fk+1 + Fk+2 1 = Fk+3 1.
Portanto, n
ao precisamos fazer as operacoes uma por uma. Podemos descrever um
metodo que sirva para todas elas, desde que tenhamos um caso inicial para servir de ponta
pe inicial. Esse metodo e o que chamamos de induca
o.
Observe que esse processo nos permite, inclusive, supor os resultado para outros momentos distintos do passo imediatamente anterior. Quando isso e necessario, costuma-se
chamar de induca
o forte.
Problema 1. Mostre que 2n > n2 , n > 4.
Soluc
ao. Vejamos um caso inicial. Para n = 5, de fato, 25 > 52 . Se duplicamos ambos os
lados, obtemos 26 > 2 52 > 62 . Agora, vamos mostrar como essa passagem de um n
umero
para o seguinte e geral.
Suponha que seja verdade para k > 4, ou seja, 2k > k2 . Da, duplicamos ambos os
lados
2k+1 > 2k2 ,
e ficar
a faltando apenas mostrar que
2k2 (k + 1)2
o que e equivalente a (k 1)2 2, o que e certamente verdade.
Problema 2. Prove que 3n n3 para todo n inteiro positivo.


POT 2012 - Algebra
- Nvel 2 - Aula 6 - Prof. Marcelo Mendes

Problema 3. Prove a desigualdade de Bernoulli: para todo n


umero real x > 1 e todo
n
umero natural n, ocorre
(1 + x)n 1 + nx.
Soluc
ao. Nesse problema, temos duas letras (x e n). Como inducao e um metodo para
n
umeros inteiros, a variac
ao s
o poder
a ser do valor de n.
No caso inicial, quando n = 1, temos (1 + x)1 1 + x 1, que e verdade.
Em seguida, suponha que a desigualdade seja verdadeira para n = k, isto e, (1 + x)k
1 + kx. Depois, multiplicamos cada lado por 1 + x (que e positivo pois x > 1) e chegamos
a
(1 + x)k+1 (1 + kx)(1 + x).
Agora, veja que (1 + kx)(1 + x) = 1 + (k + 1)x + kx2 1 + (k + 1)x pois kx2 0.
Portanto,
(1 + x)k+1 1 + (k + 1)x.
Problema 4. Se x e y s
ao n
umeros reais quaisquer, entao |x + y| |x| + |y| (desigualdade
triangular). Usando esse fato, prove que se x1 , x2 , ..., xn s
ao n
umeros reais quaisquer, entao
|x1 + x2 + ... + xn | |x1 | + |x2 | + ... + |xn |.
Problema 5. Prove que uma soma arbitraria de n 8 centavos pode ser paga com moedas
de 3 e 5 centavos (tendo essas moedas em quantidade suficiente).
Soluc
ao. Como 8 = 3+ 5, ent
ao a operacao e possvel para 8. Suponha que m 8 centavos
possam ser pagos. Ent
ao, ser
a necessario provar que m + 1 centavos podem ser pagos dessa
maneira.
Se a soma de m centavos foi paga com o uso de moedas de 5 centavos, entao substitua
uma moeda de 5 centavos por duas de 3 centavos e a quantia final ser
a m + 1 centavos.
Caso contr
ario, a soma de m centavos foi paga somente com moedas de 3 centavos e,
como m 8, h
a ao menos tres moedas de 3 centavos. Troque entao tres moedas de 3 centavos por duas de 5 centavos e novamente a quantia final de m+1 centavos desejada foi obtida.
Problema 6. Seja F1 = F2 = 1 e Fn+2 = Fn+1 + Fn , para n 1, a sequencia de Fibonacci.
Prove que quaisquer dois termos consecutivos dessa sequencia s
ao sempre primos entre si,
ou seja, mdc(Fk , Fk+1 ) = 1, k N.
Problema 7. Seja F1 = F2 = 1 e Fn = Fn1 + Fn2 , n 3, a sequencia de Fibonacci.
Mostre que F3n e par.
Problema 8. Seja F1 = F2 = 1 e Fn = Fn1 + Fn2 , n 3, a sequencia de Fibonacci.
Mostre que F5n e m
ultiplo de 5.
3


POT 2012 - Algebra
- Nvel 2 - Aula 6 - Prof. Marcelo Mendes

Problema 9. SejaF1= F2 = 1 e Fn = Fn1 + Fn2 , n 3, a sequencia de Fibonacci.


7 n
.
Mostre que Fn <
4
Problema 10. A sequencia (ai ) e definida por a1 = 0, a2 = 1, an+2 = 3an+1 2an . Encontre
uma formula explcita para o n-esimo termo dessa sequencia.
Soluc
ao. Vamos calcular alguns termos iniciais na busca de algum padrao para a formula
explcita, aquela que depende apenas de n e n
ao mais de outros termos.
a3 = 3a2 2a1 = 3 1 2 0 = 3,
a4 = 3a3 2a2 = 3 3 2 1 = 7,
a5 = 3a4 2a3 = 3 7 2 3 = 15,
a6 = 3a5 2a4 = 3 15 2 7 = 31.
O que os n
umeros 0, 1, 3, 7, 15, 31 tem de especial? Uma olhadinha cuidadosa nos faz
perceber que todos eles s
ao potencias de 2, menos 1. Mais especificamente
a3 = 3 = 22 1,
a4 = 7 = 23 1,
a5 = 15 = 24 1,
a6 = 31 = 25 1.
Portanto, nossa conjectura (sin
onimo formal para chute) ser
a que an = 2n1 1. Somente agora (ap
os a conjectura feita) aplicaremos a ideia de inducao, que s
o conseguira
provar a formula caso ela seja verdadeira (caso fosse falsa, o processo de inducao encontraria um obst
aculo intranspass
avel em algum momento).
Os casos iniciais ja est
ao escritos e validam a conjectura. Em seguida, vamos supor
que tenhamos a formula valida para todo n k (induca
o forte). Em particular, estamos
supondo para k 1 e k
ak1 = 2k2 1
ak = 2k1 1.
Da,




ak+1 = 3ak 2ak1 = 3 2k1 1 2 2k2 1
ak+1 = 2k 1.
1
Problema 11. Sabe-se que a + e um inteiro. Prove que todos os n
umeros da forma
a
1
ao inteiros.
an + n , n = 2, 3, ..., tambem s
a
4


POT 2012 - Algebra
- Nvel 2 - Aula 6 - Prof. Marcelo Mendes

Problema 12. Sejam a e b n


umeros reais distintos. Demonstrar por inducao a proposicao
(a b)| (an bn ) , n N.
Problema 13. A sequencia a1 , a2 , ..., an , ... de n
umeros e tal que a1 = 3, a2 = 5 e an+1 =
n
3an 2an1 , para n > 2. Prove que an = 2 + 1, n N.
Problema 14. Mostre, por induc
ao, que 2 + 4 + ... + 2n = 2n+1 2, n N.
Problema 15. Considere a sequencia definida recorrentemente por an+1 = 3an + 4, n N.
Supondo a0 = 0:
a) Calcule a1 , a2 , a3 , a4 .
b) Conjecture uma formula para an e prove-a por inducao.
Problema 16. Considere a sequencia definida por a1 = 0, a2 = 3, an = 5an1 4an2 , n 3.
Determine a maior potencia de 2 que divide a2012 + 1.
Problema 17. Considere a sequencia {an } definida por an = 32n 1, n N.
a) Para cada n N, mostre que an+1 = an + 8 32n .
b) Demonstre, por induc
ao sobre n, que an e divisvel por 8, para todo n N.
Problema 18. Sendo n um n
umero inteiro positivo qualquer, demonstrar que a express
ao
2n+2
n+1
3
2
e divisvel por 7.
Problema 19. Mostre que
1

1
1
1
1
1
1 1
+ ... +

=
+
+ ... +
,
2 3
2n 1 2n
n+1 n+2
2n

n N.
Problema 20. Prove que
1 1 + 1 2 2 + 1 2 3 3 + ... + 1 2 ... (n 1) n n = 1 2 ... n (n + 1) 1,
n 2 natural.
Problema 21. a) Verifique que a soma dos inversos de 2, 3 e 6 e 1.
b) Prove que p natural, p 3, existem p naturais 2 a 2 distintos n1 , n2 , ..., np tais que
1
1
1
+
+ ... +
= 1.
n1 n2
np
Problema 22. Mostre que o n
umero de diagonais de um polgono convexo de n lados e
n(n 3)
.
dn =
2


POT 2012 - Algebra
- Nvel 2 - Aula 6 - Prof. Marcelo Mendes

Dicas

2. Veja a soluc
ao da quest
ao 1.
6. Suponha que dois n
umeros de Fibonacci consecutivos possuam um fator primo em
comum e conclua que todos os n
umeros de Fibonacci teriam esse fator.



 
1
1
1
1
n
n1
n2
11. Use a identidade a + n = a
+ n1
a+
a
+ n2 .
a
a
a
a


12. Use a identidade (an bn ) = an1 bn1 (a + b) ab an2 bn2 .
16. Resolva a equac
ao de recorrencia.

18. Use os fatos 32n+2 2n+1 = 9n+1 2n+1 , 9 = 7 + 2 e (a + b)k = M a + bk , em que


M a representa um m
ultiplo de a. Ou, entao, chame an+1 = 9n+1 2n+1 e use que
an+1 2an = 7 9n , repetindo a ideia proposta no problema 17.
21. Para o item b, suponha que a igualdade seja verdadeira para k n
umeros, divida a
1
equacao por 2 e some a ambos os lados da nova identidade.
2
22. Observe que as diagonais de um polgono A1 A2 ...An+1 s
ao todas as diagonais de
A1 A2 ...An , alem do lado A1 An e das diagonais que partem de An+1 .

Respostas

15. a) a1 = 4, a2 = 16, a3 = 52, a4 = 212. b) Conjectura: an = 2 3n 2.


16. 4022.

Polos Olmpicos de Treinamento


Aula

Curso de lgebra - Nvel 2


Prof. Marcelo Mendes

Indu
c
ao - Parte II
Vamos iniciar esta aula com a resolucao de alguns problemas propostos na aula anterior.

1 Resoluc
ao de Problemas da Ultima
Aula
Problema 1. a) Verifique que a soma dos inversos de 2, 3 e 6 e 1.
b) Prove que p natural, p 3, existem p naturais 2 a 2 distintos n1 , n2 , ..., np tais que
1
1
1
+
+ ... +
= 1.
n1 n2
np
Soluc
ao. O primeiro item e imediato
1 1 1
3+2+1
+ + =
= 1.
2 3 6
6
A partir dessa soma, seguindo a dica dada na aula passada, obtemos
1
1
1 1
+ +
=
4 6 12
2
1 1 1
1
+ + +
=1
2 4 6 12
1
simplesmente dividindo por 2 e somando a cada lado em seguida.
2

Agora vamos repetir a argumentac


ao para o passo indutivo.
Suponhamos que ja tenhamos k n
umeros naturais 2 n1 < n2 < ... < nk tais que
1
1
1
+
+ ... +
= 1.
n1 n2
nk
Da
1
1
1
1
+
+ ... +
=
2n1 2n2
2nk
2


POT 2012 - Algebra
- Nvel 2 - Aula 7 - Prof. Marcelo Mendes
1
1
1
1
+
= 1,
+
+ ... +
2 2n1 2n2
2nk
que e a soma dos inversos de k + 1 n
umeros naturais distintos pois 2 < 4 2n1 < 2n2 <
... < 2nk .

1
1
Problema 2. Sabe-se que a+ e um inteiro. Prove que todos os n
umeros da forma an + n ,
a
a
n = 2, 3, ..., tambem s
ao inteiros.
Soluc
ao. Vejamos os casos iniciais para perceber como o padrao novamente e mantido para
o passo indutivo. Para n = 2,



1
1
1
2
a+
2
a + 2 = a+
a
a
a





1
1
1 2
que e claramente inteiro. A multiplicac
ao a +
e para
a+
no lugar de a +
a
a
a
preparar melhor para o passo indutivo. Para n = 3,


 

1
1
1
1
3
2
a + 3 = a + 2
a+
a+
a
a
a
a
que tambem e inteiro. Pronto, agora estamos bem preparados para o passo indutivo, basta
1
repetir o argumento para os casos pequenos. Suponha que an + n seja inteiro para todo
a
1
1
k1
k
+ k1 s
ao inteiros. Logo
k n (induc
ao forte). Em particular, a + k e a
a
a



 
1
1
1
1
k+1
k
k1
a
+ k+1 = a + k
a+
a
+ k1
a
a
a
a
tambem e inteiro.
Problema 3. Prove que
1 1 + 1 2 2 + 1 2 3 3 + ... + 1 2 ... (n 1) n n = 1 2 ... n (n + 1) 1,
n 2 natural.
Soluc
ao. Observe que podemos reescrever a soma da seguinte forma
1! 1 + 2! 2 + 3! 3 + ... + n! n = (n + 1)! 1,
sendo k! = 1 2 ... k o fatorial do n
umero inteiro n
ao-negativo k (0! = 1).
i) Para n = 1, 1! 1 = 2! 1.
ii) Suponha que a identidade seja verdadeira para n = k, ou seja,
1! 1 + 2! 2 + 3! 3 + ... + k! k = (k + 1)! 1.
2


POT 2012 - Algebra
- Nvel 2 - Aula 7 - Prof. Marcelo Mendes

iii) Agora vejamos para n = k + 1. Vamos somar (k + 1)! (k + 1) a ambos os membros da


suposta identidade do item ii):
1! 1 + 2! 2 + 3! 3 + ... + k! k + (k + 1)! (k + 1)
= (k + 1)! + (k + 1)! (k + 1) 1 = (k + 1)! (k + 2) 1 = (k + 2)! 1.

2 Mais Problemas
Problema 4. (China) Ha pelo menos quatro barras de chocolate em n(n 4) caixas.
Camila pode, por vez, escolher 2 caixas, pegar uma barra de cada uma dessas caixas e
coloca-las em uma terceira caixa. Determine se sempre e possvel por todas as barras em
uma mesma caixa.
Soluc
ao. Vejamos o caso inicial com 4 barras. As possibilidades iniciais de quantidades
nas caixas s
ao
1, 1, 1, 1, 0, 0, ...
1, 1, 2, 0, 0, 0, ...
1, 3, 0, 0, 0, 0, ...
2, 2, 0, 0, 0, 0, ...
4, 0, 0, 0, 0, 0, ...
Se ocorrer o caso na 1a linha, ent
ao pegamos as barras nas caixas 3 e 4 e as passamos
para a 5a caixa. Assim, chegamos no caso da 2a linha (n
ao faz diferenca se o 2 aparece na
a
a
3 ou na 5 posic
ao).
Se ocorrer o caso da 2a linha, s
o precisamos deslocar as barras das caixas 1 e 2 para a
a
3 caixa e o objetivo est
a feito.
Se ocorrer o caso da 3a linha, tomamos uma barra de cada caixa e as colocamos em
uma outra caixa, gerando a configurac
ao da 4a linha.
Se ocorrer o caso da 4a linha, tomamos uma barra de cada caixa e as colocamos em
uma outra caixa, obtendo o caso da 2a linha, que ja mostrado como se finaliza.
Finalmente, se ocorrer o caso da 5a linha, n
ao h
a nada a fazer pois todas as barras ja
est
ao na mesma caixa.
Agora, vamos supor que seja possvel deslocar k barras, dispostas de maneira aleatoria
nas n caixas, para uma u
nica caixa.


POT 2012 - Algebra
- Nvel 2 - Aula 7 - Prof. Marcelo Mendes

Numa configurac
ao com k +1 barras, separamos uma delas e realizamos o procedimento
indutivo com as demais k barras. Ficamos, assim, com
k, 1, 0, 0, 0, 0, ...
Podemos deslocar todas as barras para uma u
nica caixa a partir dos seguintes movimentos:
k, 1, 0, 0, ... k 1, 0, 2, 0, ...
k 2, 2, 1, 0, ... k 3, 2, 0, 2, ...
k 1, 1, 0, 1, ... k + 1, 0, 0, 0, ...
e a inducao foi finalizada.
Problema 5. Determine, com prova, se e possvel arranjar os n
umeros 1, 2, 3, ..., 1000 em
uma fila de tal forma que a media de qualquer par de n
umeros distintos n
ao esteja localizada
entre esses dois n
umeros.
Soluc
ao. Vamos provar um resultado mais geral para os n
umeros 1, 2, 3, ..., 2n .

i) Vejamos alguns casos iniciais.


Se n = 1, os n
umeros s
ao 1 e 2 e n
ao h
a nada a fazer.
Se n = 2, podemos dispor os n
umeros na ordem 1, 3; 2, 4.
Para n = 3, dispomos os n
umeros na ordem 2, 6, 4, 8; 1, 5, 3, 7. Essa ordenacao
foi obtida a partir do caso n = 2 separando os pares dos mpares da seguinte forma:
duplicamos os n
umeros do caso n = 2 e obtemos 2, 6, 4, 8. Se o resultado era valido
para n = 2, ent
ao duplicando permanece valendo. Em seguida, subtramos 1 de cada
um desses pares, obtendo 1, 5, 3, 7, que tambem possui a propriedade desejada. Alem
disso, tomando um n
umero par e um n
umero mpar, claramente a media n
ao aparece
no conjunto.
ii) Suponha que seja possvel arranjar os n
umeros 1, 2, ..., 2k na forma
a1 , a2 , ..., a2k
com a propriedade desejada.
iii) Para 2k+1 , a ideia e a mesma mostrada para se passar de 4 para 8. Separamos os pares
e os mpares de 1 a 2k+1 da seguinte forma
2a1 , 2a2 , ..., 2a2k ; 2a1 1, 2a2 1, ..., 2a2k 1.
4


POT 2012 - Algebra
- Nvel 2 - Aula 7 - Prof. Marcelo Mendes

Mostrado que o resultado e valido para potencias de 2, temos que e verdadeiro, em


particular, para 1024. Apos chegar `
a configuracao valida para 1024, e s
o apagar os n
umeros
1001 ate 1024.
Problema 6. Prove por induc
ao que
13 + 23 + ... + n3 >

n4
,
4

n N.
Problema 7. Se A e um conjunto finito com n elementos, mostre que A possui 2n subconjuntos.
Problema 8. Prove que dentre quaisquer 2m + 1 inteiros distintos, cujos valores absolutos
n
ao excedem 2m 1, e possvel encontrar 3 deles cuja soma e igual a 0.
Soluc
ao. Provemos o resultado por inducao. Para m = 1, nossos n
umeros ser
ao {1, 0, 1}
e sua soma e zero. Suponha que o resultado seja verdadeiro para m = k 1, k 2.
Considere um conjunto arbitrario A formado por 2k + 1 n
umeros cujos valores absolutos n
ao excedem 2k 1. Se dentre eles houver 2k 1 n
umeros cujos valores absolutos n
ao
excedem 2k 3, o resultado e verdadeiro pela hip
otese de inducao.
Caso contr
ario, sem perda de generalidade, podemos considerar que A contem os
n
umeros 2k 1, 2k 2, 2k + 1(2k + 1 est
a) ou 2k 1, 2k 2, 2k + 2(2k + 1 n
ao
est
a e os demais est
ao). Primeiro, considere os seguintes 2k 2 pares de n
umeros, supondo
2k 1 e 2k + 1 no conjunto:
(1, 2k 2)
(2, 2k 3)
(3, 2k 4)
:
(k, k 1)
e
(1, 2k + 2)
(2, 2k + 3)
:
(k + 1, k)
Se 0 est
a dentre os n
umeros escolhidos, entao juntamente com 2k 1 ou 2k + 1
teramos um tripla com soma zero. Sen
ao, teramos 2k 1 inteiros de A distribudos dentre
os n
umeros desses 2k 2 pares e dois deles estariam no mesmo par, que juntamente com
2k 1 ou 2k + 1 formariam um tripla com soma zero.


POT 2012 - Algebra
- Nvel 2 - Aula 7 - Prof. Marcelo Mendes

Agora, considere que 2k + 1 e os 2k 4 pares


(1, 2k 3)
(2, 2k 4)
(3, 2k 5)
:
(k 2, k)
e
(2, 2k + 3)
:
(k + 1, k)
Se 0 ou 1 estiver e trivial. Sen
ao, ja escolhemos 3 n
umeros (0, 1, 2k + 1). Assim
teramos que escolher ainda 2k 3 n
umeros dentre os 2k 4 pares, e teramos dois deles
no mesmo par.
Problema 9. Prove que a sequencia
22 3, 23 3, ..., 2n 3, ...
contem um n
umero infinito de inteiros tais que cada dois deles sejam relativamente primos.
Soluc
ao. Provemos por induc
ao.
Suponha que ja tenhamos k n
umeros a1 = 2n1 3, a2 = 2n2 3, ..., ak = 2nk 3, que
sejam primos entre si.
Construamos agora ak+1 relativamente primo com os k demais. Seja N = a1 a2 ...ak .
Pelo Princpio da Casa dos Pombos, dentre os N +1 n
umeros 20 , 21 , ..., 2N haver
a dois deles,
r
s
digamos 2 e 2 (r > s), com o mesmo resto na divisao por N . Assim, 2r 2s = 2s (2rs 1)
e divisvel por N , assim como 2rs 1, pois N e mpar (ja que e produto de primos). Portanto, ak+1 = 2rs 3 e relativamente primo com N e, consequentemente, com cada um
dos a1 , a2 , ..., ak .
Problema 10. Tres inteiros foram escritos em um quadro-negro. Entao um deles foi apagado e a soma dos outros dois, menos 1, foi escrito em seu lugar. Esse procedimento foi

repetido varias vezes ate que os n


umeros 17, 1983, 1989 aparecessem eventualmente. E
possvel que os n
umeros iniciais fossem 2, 2, 2?


POT 2012 - Algebra
- Nvel 2 - Aula 7 - Prof. Marcelo Mendes

Problema 11. Uma quantidade finita de cart


oes e colocada em duas torres, com mais
cart
oes na torre esquerda que na direita. Cada cart
ao tem um ou mais nomes distintos
escrito nele. Alem disso, diferentes cart
oes podem compartilhar alguns nomes. Para cada
nome, definimos uma ac
ao pelo movimento de todo cart
ao que tem esse mesmo nome escrito nele para a torre oposta. Prove que e sempre possvel finalizar com mais cart
oes na
torre da direita atraves de varias ac
oes para diferentes nomes.
Problema 12. Sejam F1 = F2 = 1 e, para n 3, Fn = Fn1 + Fn2 os n
umeros de Fibonacci. Prove o Teorema de Zeckendorff: todo n
umero natural pode ser escrito de maneira
u
nica como soma de n
umeros de Fibonacci com ndices maiores que 1 e n
ao consecutivos.


POT 2012 - Algebra
- Nvel 2 - Aula 7 - Prof. Marcelo Mendes

Dicas

7. Use que, ao acrescentarmos um novo elemento a um conjunto ja existente, os subconjuntos do conjunto inicial permanecem e todos elementos, unidos com o novo
elemento, formam os novos subconjuntos.
10. Tente mostrar que h
a sempre 2 n
umeros pares em cada tripla.
11. Inducao sobre a quantidade de nomes.


POT 2012 - Algebra
- Nvel 2 - Aula 7 - Prof. Marcelo Mendes

Respostas

10. Sempre h
a a presenca de no mnimo 2 n
umeros pares. Como no incio temos 3
n
umeros pares, em seguida teremos 2 pares e um mpar; e dois pares e um mpar e
dois pares e um mpar ... Pode-se provar por inducao que em qualquer passo teremos
sempre 2 n
umeros pares e um mpar. Mas a tripla (17, 1983, 1989) s
o tem n
umeros
mpares e nunca poder
a ser obtida de (2, 2, 2).
11. Vamos utilizar induc
ao sobre a quantidade de nomes.
i. n = 1 e trivial.
ii. Sejam a1 , a2 , ..., an os n primeiros nomes e a, o nome n + 1.
iii. Para n + 1, h
a 2 casos, sendo aE e aD as quantidades de cart
oes com apenas o
nome a nas pilhas esquerdas e direita, respectivamente.
1o caso: aE aD .
Realize as operac
oes com os nomes a1 , a2 , ..., an .
2o caso: aE > aD .
Realize a troca apenas com os cart
oes em que est
a escrito apenas o nome a e repita
as operac
oes do 1o caso.
12. F2 = 1. Se Fk e o maior n
umero de Fibonacci menor que ou igual a n, devemos ter
n Fk < Fk1 pois, caso contr
ario, n Fk Fk1 e, portanto, n Fk + Fk1 = Fk+1 ,
absurdo. Assim, e s
o escrever n = Fk + (n Fk ) e utilizar hip
otese de inducao para
n Fk .

Polos Olmpicos de Treinamento


Curso de lgebra - Nvel 2
Prof. Marcelo Mendes

Aula

Desigualdades - Parte I
1 Fatos Elementares
i) Nenhum quadrado de n
umero real e negativo.
ii) Desigualdade de Cauchy (Medias Aritmetica e Geometrica)
Se a1 , a2 , ..., an s
ao n
umeros reais positivos, entao

a1 + a2 + ... + an
n a1 a2 ...an ,
n
com igualdade ocorrendo se, e somente se, a1 = a2 = ... = an .
Para mostrar essa u
ltima desigualdade, vamos utilizar um tipo diferente de inducao
(que n
ao serve para qualquer problema).

1. Se n = 2, ent
ao

a1 + a2

2
a1 a2 0.
a1 a2 pois
2

2. Para n = 4, ent
ao utilizando o caso ja mostrado para 2 n
umeros, temos
r
a1 +a2
4
+ a3 +a
a1 + a2 + a3 + a4
a1 + a2 a3 + a4
2
2
=

4
2
2
2
q

a1 a2 a3 a4 = 4 a1 a2 a3 a4 ,
quaisquer que sejam a1 , a2 , a3 , a4 reais positivos.
a1 + a2 + a3
, obter
3
r
a1 + a2 + a3 + a1 +a32 +a3
a1 + a2 + a3
4 a1 a2 a3
4
3

3. Assim, podemos escolher a4 =


POT 2012 - Algebra
- Nvel 2 - Aula 8 - Prof. Marcelo Mendes

a1 + a2 + a3
3

4

a1 a2 a3

a1 + a2 + a3
3

a1 + a2 + a3
3 a1 a2 a3 ,
3

e concluir que o resultado tambem e verdadeiro para n = 3.


A demonstrac
ao segue copiando as ideias acima. J
a temos os casos iniciais. Em seguida,
supondo o resultado verdadeiro para k, obtemos o resultado para 2k e para k 1 repetindo
os procedimentos realizados nos itens 2 e 3 acima. Assim, provamos a desigualdade para
qualquer quantidade natural maior que ou igual a 2 de n
umeros reais positivos.

2 Problemas
Problema 1. Determine o valor m
aximo da funcao f (x) = x(1 x), sendo x (0; 1).
Soluc
ao. Essa e uma func
ao quadr
atica. Poderamos encontrar o seu valor m
aximo atraves
da ordenada do vertice da par
abola (desde que a abscissa do vertice esteja em (0; 1), o que,
de fato, e verdade).
Mas se resolvermos utilizando a Desigualdade de Cauchy, poderemos aplicar a ideia
para funcoes de grau maior que 2:
x + (1 x) p
x(1 x)
2
1
x(1 x) ,
4
com igualdade ocorrendo se, e somente se, x = 1x, ou seja, x = 12 . Assim, o valor m
aximo
de f e 14 .
Observa
c
ao. Existe uma diferenca entre descobrir que f (x) 41 e concluir que 41 e seu
valor m
aximo. Por exemplo, podemos afirmar que sen x 3, porem o valor m
aximo de
sen x e 1, pois a igualdade em sen x 3 n
ao ocorre.
Problema 2. Determine o valor m
aximo da funcao f (x) = x3 (1 x), sendo x (0; 1).
Soluc
ao. Uma ideia possvel seria aplicar a Desigualdade de Cauchy com os n
umeros reais
3
positivos x e 1 x:
x3 + (1 x) p 3
x (1 x).
2
2


POT 2012 - Algebra
- Nvel 2 - Aula 8 - Prof. Marcelo Mendes

Apesar de verdadeiro, esse fato n


ao nos d
a um valor (n
ao poder ser vari
avel) m
aximo
para f .
Outra tentativa seria com x, x, x, 1 x, todos positivos:

ou seja,

p
x + x + x + (1 x)
4 x3 (1 x),
4

x (1 x)

2x + 1
4

4

e, novamente, n
ao achamos um valor m
aximo. Todavia, chegamos bem perto. Basta
substituir 1 x por 3(1 x):

e da

p
x + x + x + 3(1 x)
4 x3 3(1 x),
4

x3 3(1 x)

 4
3
4

x3 (1 x)

27
.
81

27
3
aximo de f e
.
Como a igualdade ocorre com x = 3(1 x) x = , o valor m
4
81
Problema 3. Determine o valor m
aximo da funcao f (x) = x(1 x)3 , sendo x (0; 1).
Problema 4. (Treinamento Cone Sul) Sejam a e b n
umeros reais positivos tais que a+b = 1.
4
Prove que ab2 .
27
Problema 5. Sejam A, B, C os vertices de um tri
angulo inscrito em um crculo unit
ario (ou
seja, cujo raio mede 1) e seja P um ponto no permetro do tri
angulo. Mostre que
PA PB PC

32
.
27

Problema 6. Dados n
umeros positivos arbitrarios a, b, c, prove que ao menos uma das
seguintes desigualdades e falsa:
1
1
1
a(1 b) > , b(1 c) > , c(1 a) > .
4
4
4


POT 2012 - Algebra
- Nvel 2 - Aula 8 - Prof. Marcelo Mendes

Problema 7. (IMO) Sendo K, L, M pontos sobre os lados BC, CA, AB do ABC, mostre
1
que a area de ao menos um dos tri
angulos AM L, BKM, CLK e menor que ou igual da
4
area do tri
angulo ABC.
Soluc
ao. Sendo k, l, m [0; 1], podemos escrever
BK = ka, KC = (1 k)a
CL = lb, LA = (1 l)b
AM = mc, M B = (1 m)c.
Assim,
1
mc (1 l)b senA
2
[AM L] = m(1 l) [ABC].

[AM L] =

Analogamente,
[BKM ] = k(1 m) [ABC],
[CLK] = l(1 k) [ABC].

1
Supondo que as tres
areas em quest
ao sejam maiores que da area de ABC, o resultado
4
segue pelo problema 6.
Problema 8. (Treinamento Cone Sul) Sejam ha , hb , hc as alturas do ABC. Prove que
ABC e equilatero ahb + bhc + cha e igual a 6 vezes a area do ABC.
Problema 9. (Treinamento Cone Sul) Seja P um polgono convexo com 2012 lados e com
todos os angulos internos iguais. Sejam l1 , l2 , ..., l2012 os comprimentos dos lados consecutivos. Prove que se
l2011 l2012
l1 l2
+ + ... +
+
= 2012,
l2 l3
l2012
l1
entao P e um polgono regular.
Problema 10. Mostre que, se x, y, z s
ao n
umeros reais positivos, entao
1
1
1
(1 + xy) + (1 + yz) + (1 + zx) 6.
x
y
z
Problema 11. Prove a desigualdade entre as medias geometrica e harmonica para 2 n
umeros
a e b reais positivos, ou seja,

ab
4

1
a

2
.
+ 1b


POT 2012 - Algebra
- Nvel 2 - Aula 8 - Prof. Marcelo Mendes

Problema 12. Prove a desigualdade entre as medias quadr


atica e aritmetica para 2 n
umeros
reais positivos.
Soluc
ao. Devemos mostrar que
r

a 2 + b2
a+b

,
2
2

que e equivalente a (a b)2 0.


Problema 13. Prove que se a, b, c s
ao as medidas dos lados de um tri
angulo e a2 + b2 = kc2 ,
1
entao k > .
2
Problema 14. a) Prove que se a, b s
ao inteiros positivos com a 6= b, entao
1 1
4
+
.
a b
a+b
permitido apagar qualquer par deles a e b,
b) Em uma lousa, escrevemos n n
umeros. E
a+b
no lugar. Repetindo tal procedimento n 1 vezes, obtemos o n
umero
escrevendo
4
2012
k. Se os n n
umeros iniciais eram 2012, prove que k
.
n
Problema 15. Seja x um n
umero real e m, um natural. Prove que
1
1
1
x(x + 1)(x + 2)...(x + m 1)
x1+ 2 + 3 +...+ m .
m(m 1)(m 2)...1


POT 2012 - Algebra
- Nvel 2 - Aula 8 - Prof. Marcelo Mendes

Dicas
3. Repita a ideia da soluc
ao do problema 2.
4. Repita a ideia da soluc
ao do problema 2.
5. Repita a ideia da soluc
ao do problema 2. Use tambem potencia do ponto P e que,
supondo P sobre o lado BC, a corda contendo P A tem medida menor que ou igual
`a medida 2 do di
amentro.
6. Suponha a possibilidade de ocorrerem as 3 desigualdades e multiplique-as.
8. Use a Desigualdade de Cauchy com ahb , bhc , cha .
9. Use a Desigualdade de Cauchy com

l2011 l2012
l1 l2
, , ...,
,
.
l2 l3
l2012 l1

13. Use o problema 12 e a desigualdade triangular.


14. Compare a soma dos inversos dos n
umeros antes e depois de cada substituicao de
n
umeros.
x+1+1
x+2
=
e aplique a Desigualdade de Cauchy. Faca
15. Escreva, por exemplo,
3
3
o mesmo para os demais fatores do numerador e do denominador aos pares.

Respostas
13. Pelo enunciado, pelo problema 12 e pela desigualdade triangular, temos
a 2 + b2
kc2
=

2
2

a+b
2
1
k> .
2

2

>

 c 2
2

4
1 1
+
(a + b)2 4ab (a b)2 0.
a b
a+b
b) Por a), segue que a soma dos inversos dos n
umeros envolvidos nunca aumenta.
Assim, comparando o incio e o final dos procedimentos, temos

14. a)

1
1
1
+ ... +

2012
2012
k
1
1
n

2012
k
2012
.
k
n
6

Polos Olmpicos de Treinamento


Aula

Curso de lgebra - Nvel 2


Prof. Marcelo Mendes

Desigualdades - Parte II
1 A Desigualdade de Cauchy-Schwarz
Sejam a1 , a2 , ..., an , b1 , b2 , ..., bn n
umeros reais. Entao:
a21 + a22 + ... + a2n


b21 + b22 + ... + b2n (a1 b1 + a2 b2 + ... + an bn )2 .

Perceba o padrao que h


a na aplicac
ao desta desigualdade: multiplicamos os termos de
mesma ordem de cada soma do lado esquerdo e, em seguida, extramos a raiz quadrada;
o resultado e colocado na posic
ao correspondente no somat
orio do lado direito, que ser
a
elevado ao quadrado.
Vejamos agora alguns exemplos antes de apresentarmos a demonstracao da desigualdade.
Exemplo 1 Prove a desigualdade entre as medias quadr
atica e aritmetica para n n
umeros
reais positivos.
Soluc
ao. O caso particular para n = 2 dessa desigualdade apareceu na aula anterior e a
solucao apresentada apenas utilizou o fato de um quadrado de n
umero real ser n
ao-negativo.
Vejamos agora a soluc
ao para uma quantidade qualquer.
Por Cauchy-Schwarz, temos
a21 + a22 + ... + a2n
r


12 + 12 + ... + 12 (a1 + a2 + ... + an )2

a1 + a2 + ... + an
a21 + a22 + ... + a2n

,
n
n
como desejavamos (a express
ao no lado esquerdo e a media quadr
atica dos n n
umeros
a1 , a2 , ..., an ).


POT 2012 - Algebra
- Nvel 2 - Aula 9 - Prof. Marcelo Mendes
Exemplo 2 Sejam a, b, c n
umeros reais positivos. Prove que
a2 + b2 + c2 ab + bc + ca.
Soluc
ao. Por Cauchy-Schwarz:
a2 + b2 + c2


b2 + c2 + a2 (ab + bc + ca)2 .

Extraindo a raiz quadrada, temos o resultado.

Demonstra
c
ao (da Desigualdade de Cauchy-Schwarz)
n
X
Considere a func
ao f (x) =
(ak x bk )2 . Desenvolvendo, teremos uma funcao do 2o grau

em x:

k=1

f (x) = x2

n
X
k=1

cujo descriminante e dado por


=4

n
X

a k bk

k=1

a2k 2x

!2

n
X

ak bk +

k=1

n
X
k=1

n
X

b2k ,

k=1

a2k

n
X
k=1

b2k

Como f e uma soma de quadrados, f (x) 0, x R. Da, 0 e segue o resultado.


J
a a igualdade ocorre quando = 0. Nesse caso, f possui uma raiz (dupla) x0 . Isso
implica que
n
X
k=1

(ak x0 bk )2 = 0.

Nessa soma, todos os n


umeros envolvidos s
ao reais e, portanto, seus quadrados s
ao
n
ao-negativos. Assim, ak x0 bk = 0, k, o que indica que, para a igualdade, ak e bk s
ao
proporcionais para todo k.

2 Problemas
Problema 1. Seja c o comprimento
angulo ret
angulo cujos catetos
da hipotenusa de um tri
medem a e b. Prove que a + b c 2.
Soluc
ao. Do primeiro exemplo, com n = 2, temos
a 2 + b2

2
Da

a+b
2

2

(a + b)2
c2

a + b c 2.
2
4
2


POT 2012 - Algebra
- Nvel 2 - Aula 9 - Prof. Marcelo Mendes

Problema 2. Sendo a, b, c n
umeros reais positivos, mostre que


1
1
9
1
+
+
.

2
a+b b+c c+a
a+b+c

Problema 3. Sejam x1 , x2 , ..., xn n


umeros reais positivos e y1 , y2 , ..., yn uma permutacao
dos xi , 1 i n. Prove a desigualdade
x21 x22
x2
+
+ ... + n x1 + x2 + ... + xn .
y1
y2
yn
Soluc
ao. Pela desigualdade de Cauchy-Schwarz, segue
 2

x1 x22
x2n
+
+ ... +
(y1 + y2 + ... + yn ) (x1 + x2 + ... + xn )2 .
y1
y2
yn
O resultado segue do fato de x1 + x2 + ... + xn = y1 + y2 + ... + yn pois os yi s
ao os xi
em alguma ordem.
Problema 4. (Baltic-Way) Prove que para quaisquer reais positivos x1 , x2 , ..., xn , y1 , y2 , ..., yn
ocorre
n
X
4n2
1
n
.
X
xi y i
2
k=1
(xi + yi )
k=1

Soluc
ao. Por Cauchy-Schwarz, garantimos que

n
n
X
1 X
(xi + yi )2
xi y i
k=1

k=1

n
X
xi + y i

xi y i
k=1

!2

Pela desigualdade entre as medias aritmetica e geometrica, obtemos


xi + y i
2.

xi y i
Assim,
n
X

n
n
X
1 X
(xi + yi )2
xi y i
k=1

k=1

k=1

!2

= (2n)2 = 4n2 .


POT 2012 - Algebra
- Nvel 2 - Aula 9 - Prof. Marcelo Mendes

Problema 5. Se a, b, c s
ao n
umeros positivos, prove que
a2 b + b2 c + c2 a


a2 c + b2 a + c2 b 9a2 b2 c2 .

Obs: Resolva esse problema de duas maneiras, utilizando a desigualdade entre as medias
aritmetica e geometrica e atraves da desigualdade de Cauchy-Schwarz.
Problema 6. Sejam a1 , a2 , ..., an , b1 , b2 , ..., bn n
umeros reais positivos tais que

n
X

ak =

k=1

n
X
k=1

bk . Mostre que

n
X
k=1

a2k
1X

ak .
ak + bk
2
k=1

Problema 7. Se a1 , a2 , ..., an , b1 , b2 , ..., bn s


ao 2n n
umeros reais positivos, mostre que ou
a1 a2
an
+
+ ... +
n
b1
b2
bn
ou
b2
bn
b1
+
+ ... +
n.
a1 a2
an

Problema 8. Sejam a, b, c os lados de um tri


angulo. Mostre que
a
b
c
3
+
+
.
b+c c+a a+b
2
Soluc
ao. Somando 1 a cada frac
ao do lado esquerdo, obtemos
9
a+b+c a+b+c a+b+c
+
+
.
b+c
c+a
a+b
2
Para mostrar essa u
ltima desigualdade, vamos utilizar a desigualdade de Cauchy-Schwarz

 h
i2

a+b+c a+b+c a+b+c


[(b + c) + (c + a) + (a + b)]
+
+
3 a+b+c
b+c
c+a
a+b



a+b+c a+b+c a+b+c
[2(a + b + c)]
+
+
9(a + b + c),
b+c
c+a
a+b
de onde segue o resultado.

Problema 9. Se a > 0, b > 0, c > 0, ent


ao prove que

bc ca ab
+
+
a + b + c.
a
b
c


POT 2012 - Algebra
- Nvel 2 - Aula 9 - Prof. Marcelo Mendes
b3
c3
a3
+
+
a + b + c, quaisquer que sejam a, b, c
Problema 10. (Romenia) Prove que
bc ca ab
reais positivos.
Problema 11. Sejam a, b, c, d n
umeros reais positivos. Mostre que
1 1 4 16
64
+ + +

.
a b
c
d
a+b+c+d
Soluc
ao. Pela desigualdade de Cauchy-Schwarz, obtemos


1 1 4 16
+ + +
(1 + 1 + 2 + 4)2 = 64,
(a + b + c + d)
a b
c
d
de onde segue o resultado.


POT 2012 - Algebra
- Nvel 2 - Aula 9 - Prof. Marcelo Mendes

Dicas

2. Passe a + b + c para a esquerda e escreva 2(a + b + c) = (a + b) + (b + c) + (c + a).


5. Faca a primeira soluc
ao atraves da desigualdade entre as medias aritmetica e geometrica
aplicada a cada soma em parenteses do lado esquerdo. A segunda pode ser obtida atraves da desigualdade de Cauchy-Schwarz reescrevendo a2 c + b2 a + c2 b como
c2 b + a2 c + b2 a.
n
X
a2k
por
(ak + bk ), aplique a desigualdade de Cauchy-Schwarz
ak + bk
k=1
k=1
n
n
X
X
e lembre-se de que
(ak + bk ) = 2
ak .

6. Multiplique

n
X

k=1

k=1

an
b1
b2
bn
a1 a2
+
+ ... +
< n ou
+
+ ... +
< n e aplique a desigualdade
b1
b2
bn
a1 a2
an
de Cauchy-Schwarz com as somas nos lados esquerdos dessas duas desigualdades.
Conclua, assim, um absurdo.
r 2
bc
bc
(repita o mesmo para as demais parcelas do lado esquerdo) e
=
9. Escreva
a
a
utilize o exemplo 2.

7. Suponha

10. Repita a ideia do problema 9.


POT 2012 - Algebra
- Nvel 2 - Aula 9 - Prof. Marcelo Mendes

Soluc
oes

2. Por Cauchy-Schwarz
((a + b) + (b + c) + (c + a))

1
1
1
+
+
a+b b+c c+a

(1 + 1 + 1)2 = 9.

5. 1a Soluc
ao. Pela desigualdade entre as medias aritmetica e geometrica, temos
a2 b + b2 c + c2 a
3
a2 b b2 c c2 a
3
a2 b + b2 c + c2 a 3abc.
Analogamente,
a2 c + b2 a + c2 b 3abc.
Multiplicando essas duas desigualdades, segue o resultado.
2a Soluc
ao. Por Cauchy-Schwarz, obtemos


a2 b + b2 c + c2 a c2 b + a2 c + b2 a (abc + abc + abc)2 = 9a2 b2 c2 .

6. Por Cauchy-Schwarz, obtemos


n
X
k=1

n
a2k X
(ak + bk )
ak + bk
k=1

n
X
k=1

k=1

n
X
k=1

ak

k=1

n
X

X
a2k
2
ak
a k + bk

n
X

k=1

ak

!2

!2

a2k
1X

ak .
ak + bk
2
k=1

a2
an
b1
b2
bn
a1
+
+ ... +
< n ou
+
+ ... +
< n. Pela desigualdade de
b1
b2
bn
a1
a2
an
Cauchy-Schwarz e pela u
ltima hip
otese, temos

7. Supondo

an
a1 a2
+
+ ... +
b1
b2
bn
7



b1
b2
bn
+
+ ... +
a1 a2
an

< n2 ,


POT 2012 - Algebra
- Nvel 2 - Aula 9 - Prof. Marcelo Mendes

um absurdo. Logo, ao menos uma das desigualdades


a1 a2
an
+
+ ... +
n,
b1
b2
bn
b1
b2
bn
+
+ ... +
n
a1 a2
an
e verdadeira.
9. Utilizando o resultado do exemplo 2, obtemos
r

bc
+
a

ca
+
b

ab

bc
a

ca
+
b

ca
b

ab
+
c

ab
c

bc
= c + a + b.
a

Polos Olmpicos de Treinamento


Aula

Curso de lgebra - Nvel 2


Prof. Marcelo Mendes

10

Problemas Envolvendo M
aximos e Mnimos
Vamos iniciar esta aula aplicando desigualdades aprendidas nas u
ltimas duas aulas focando mais em exemplos envolvendo m
aximos e mnimos de funcoes.
Problema 1. Determine o valor m
aximo da funcao f (x) = x(1 x)3 , sendo x (0; 1).
Soluc
ao. A ideia da soluc
ao desse problema ja foi aprendida na aula 8. Vamos rever
como resolve-lo e, mais uma vez, chamar a atencao para a diferenca existente entre obter
f (x) k e garantir que k e o valor m
aximo de f .
Atraves da desigualdade entre as medias aritmetica e geometrica, ja que x e 1 x s
ao
positivos, obtemos
3x + (1 x) + (1 x) + (1 x) p
4 3x(1 x)3
4
 4
3

3x(1 x)3
4
27
.
256
27
Nesse momento, a expectativa
obvia e de que
deva, de fato, ser o valor m
aximo de
256
f , mas ainda precisamos garantir esse fato.
x(1 x)3

E como conseguiremos essa garantia? Da mesma forma que procedemos na aula 8.


27
Mostrar que o m
aximo de f e
e equivalente a achar um valor de x (0; 1) que de a
256
igualdade na desigualdade, e isso ocorre (gracas `a condicao de igualdade em MA MG)
1
quando 3x = 1 x x = , que e um valor no intervalo (0; 1).
4
27
Portanto,
realmente e o valor m
aximo de f .
256


POT 2012 - Algebra
- Nvel 2 - Aula 10 - Prof. Marcelo Mendes
Problema 2. Determine o valor m
aximo da funcao f (x) = x4 (2 x), sendo x (0; 2).
Problema 3. Seja a um n
umero real positivo dado. Determine o valor de x [0; a] que
maximiza o valor de f (x) = x5 (a x).
Problema 4. Seja x > 0, x R. Determine o valor mnimo de x2 +

2
.
x

Problema 5. (EUA) Considere a equacao 3x2 4x + k = 0 com razes reais. Determine o


valor de k para o qual o produto das razes da equacao seja m
aximo.
Problema 6. Se x, y, z s
ao reais e satisfazem x + y + z = 5 e yz + zx + xy = 3, prove que
13
1 z
e determine o valor mnimo de z.
3
Soluc
ao. De (x + y + z)2 = x2 + y 2 + z 2 + 2(xy + yz + zx), obtemos x2 + y 2 + z 2 = 19.
Assim, por Caychy-Schwarz, chegamos a

(x2 + y 2 )(1 + 1) (x + y)2 19 z 2 2 (5 z)2
38 2z 2 25 10z + z 2
3z 2 10z 13 0,
cuja solucao e 1 z

13
. O valor mnimo de z, de fato, e -1, quando x = y = 3.
3

Problema 7. Seja k uma constante real positiva. Dentre todos os tri


angulos tendo base a
e altura relativa a essa base h, sendo a + h = k, determine aquele(s) cuja area e m
axima.
Problema 8. Sejam A, B, C os vertices de um tri
angulo inscrito em um crculo unit
ario (ou
seja, cujo raio mede 1) e seja P um ponto no permetro do tri
angulo. Mostre que
PA PB PC

32
.
27

Soluc
ao. Nesse problema, n
ao foi pedido o valor m
aximo de P A P B P C. Mesmo assim,
32
vamos mostrar que P AP B P C
e, em seguida, examinar se a igualdade pode ocorrer,
27
32
ou seja, se o valor m
aximo de P A P B P C e
.
27


POT 2012 - Algebra
- Nvel 2 - Aula 10 - Prof. Marcelo Mendes

P
X

Pela potencia do ponto P em relacao a (ABC) (ABC entre parenteses representa o a


circunferencia que passa pelos pontos A, B e C), temos
P B P C = P A P X.
Da, utilizando a desigualdade entre as medias aritmetica e geometrica, obtemos


P A + P A + 2P X
2P A P B P C = P A 2P X
3
 3


4
2AX 3

=
3
3
2

3

32
,
27
utilizando que AX e uma corda e, portanto, tem medida menor que ou igual `a medida do
di
ametro, que e 2.
PA PB PC

J
a conseguimos chegar ao resultado pedido no enunciado. Agora, vamos verificar se e
32
possvel obtermos o valor m
aximo
.
27
Essa igualdade ocorre se, e somente se, P est
a sobre o di
ametro que passa por A e
4
P A = 2P X = , que depende do tri
angulo ABC inicial. Portanto, nem sempre a igual3
dade ocorre.


POT 2012 - Algebra
- Nvel 2 - Aula 10 - Prof. Marcelo Mendes

Problema 9. Sejam a e b n
umeros reais positivos. Ache o valor m
aximo da funcao real e
de vari
avel real
x
.
y= 2
ax + b
Soluc
ao. Podemos escrever a equac
ao acima da seguinte forma
ayx2 x + by = 0,
cujo discriminante e
= 1 4aby.
Como x R, temos 0, ou seja, y

1
.
4ab

1
e o nosso candidato a valor m
aximo da funcao. Para esse valor ser atingido,
Assim,
4ab
devemos ter = 0 e, portanto
x=

1
= 2b.
2ay

Pense tambem em uma soluc


ao comecando com MA MG entre ax2 e b.
Problema 10. Seja P um ponto no interior de um tri
angulo A1 A2 A3 e P1 , P2 , P3 , os pes
das perpendiculares de P a A2 A3 , A3 A1 , A1 A2 . Localize o ponto P tal que
A1 A2 A2 A3 A3 A1
+
+
.
P P3
P P1
P P2
seja mnimo.
Soluc
ao. Vamos pensar um pouquinho. As fracoes envolvidas nessa soma relacionam bases e alturas (pense sempre que dist
ancias lembram alturas e que alturas lembram area)
dos tri
angulos A2 P A3 , A3 P A1 , A1 P A2 e, portanto, nos fazem pensar nas areas desses
tri
angulos, e a
area do tri
angulo A1 A2 A3 ser
a a soma dessas areas.
Pela desigualdade de Cauchy-Schwarz, temos


A1 A2 A2 A3 A3 A1
+
+
(A1 A2 P P3 + A2 A3 P P1 + A3 A1 P P2 )
P P3
P P1
P P2
(A1 A2 + A2 A3 + A3 A1 )2 .
Como
A1 A2 P P3 + A2 A3 P P1 + A3 A1 P P2 = 2S
e
A1 A2 + A2 A3 + A3 A1 = p,
sendo S a area e p o permetro do tri
angulo A1 A2 A3 , chegamos a
4


POT 2012 - Algebra
- Nvel 2 - Aula 10 - Prof. Marcelo Mendes

p2
A1 A2 A2 A3 A3 A1
+
+

.
P P3
P P1
P P2
2S
Portanto, o candidato a valor mnimo de

A1 A2 A2 A3 A3 A1 p2
+
+
e
.
P P3
P P1
P P2
2S

Esse valor mnimo ser


a atingido se a igualdade ocorrer na desigualdade. A igualdade
na desigualdade de Cauchy-Schwarz ocorre com a proporcao entre as respectivas parcelas
das somas envolvidas, ou seja,
A1 A2
A2 A3
A3 A1
P P3
P P1
P P2
=
=
A1 A2 P P3
A2 A3 P P1
A3 A1 P P2
P P1 = P P2 = P P3 ,
de onde segue que o valor mnimo e atingido e e quando P e o incentro do tri
angulo A1 A2 A3 .


POT 2012 - Algebra
- Nvel 2 - Aula 10 - Prof. Marcelo Mendes

Dicas

2. Repita as ideias da soluc


ao do problema 1.
3. Repita as ideias da soluc
ao do problema 1.
4. Repita as ideias da soluc
ao do problema 1.
k
4
4
5. Denote por x1 e x2 as razes. Assim, x1 + x2 = e x1 x2 = . Escreva x2 = x1
3
3
3
e repita as ideias da soluc
ao do problema 1.
7. Use h = k a e repita as ideias da solucao do problema 1.


POT 2012 - Algebra
- Nvel 2 - Aula 10 - Prof. Marcelo Mendes

Soluc
oes

2. Por MA MG, temos


x + x + x + x + 4(2 x) p
5 4x4 (2 x)
5
1
x (2 x)
4
4

 5
8
49
= 5.
5
5

3. Por MA MG, temos


p
x + x + x + x + x + 5(a x)
6 5x5 (a x)
6
x5 (a x)

5a
6

6

= 55

 a 6
6

7. Por MA MG, temos


a + (k a) p
a(k a)
2

a(k a)
k2
,
2
8

k
k
com igualdade se, e somente se, a = k a, ou seja, a = . Nesse caso, h = tambem,
2
2
o que determina os tri
angulos com area m
axima.

Polos Olmpicos de Treinamento


Aula

Curso de lgebra - Nvel 2


Prof. Marcelo Mendes

11

Fun
c
oes Definidas Implicitamente - Parte I
Talvez a experiencia de alguns de voces diga que as solucoes de uma equacao devam
ser necessariamente n
umeros. Mas isso n
ao e verdade. Em matem
atica, podemos ter, por
exemplo, matrizes ou func
oes como solucoes para equacoes matriciais ou funcionais, respectivamente.
Nesta aula, vamos aprender resolver algumas equacoes funcionais, que tem funcoes como
solucoes e, por isso, dizemos que essas funcoes foram definidas implicitamente (implcito
significa escondido).
Problema 1. Determine todas as funcoes f : Z+ Z+ tais que f (1) = c e
f (x + y) = f (x) + f (y), x, y Z+ .
Soluc
ao. Nesse problema, a equac
ao funcional e f (x + y) = f (x) + f (y). Precisamos
resolve-la.
Observe que o problema nos permite utilizarmos quaisquer valores inteiros n
ao-negativos
para x e y. Assim, vamos iniciar com x = y = 0:
f (0 + 0) = f (0) + f (0) f (0) = 0.
Agora, facamos a escolha de deixar x vari
avel e y = 1:
f (x + 1) = f (x) + f (1) = f (x) + c.
Acabamos de gerar uma equac
ao de diferenca, tipo explorado na aula de somas telesc
opicas. Vamos escrever varias dessas equacoes, variando n 2:
f (n) = f (n 1) + c
f (n 1) = f (n 2) + c
:


POT 2012 - Algebra
- Nvel 2 - Aula 11 - Prof. Marcelo Mendes

f (2) = f (1) + c
e som
a-las, obtendo
f (n) = cn.
Assim, a (
unica) soluc
ao e f (x) = cx, sendo c = f (1). (Poderamos, tambem, ter aplicado
a formula do termo geral da P.A.)
Problema 2. Seja f : R R uma funcao tal que f (0) = 1 e, para quaisquer x, y R,
f (xy + 1) = f (x)f (y) f (y) x + 2.
Determine o valor de f(2012).
Problema 3. Seja f : R+ R uma funcao satisfazendo a equacao funcional
f (a) + f (b) = f (ab), a, b R+ .
Mostre que:
a) f (1) = 0.
b) f (an ) = n f (a), a R+ , n N.
 
1
= f (a), a R+ .
c) f
a
Soluc
ao.
a) Com a = b = 1, obtemos f (1) + f (1) = f (1 1), ou seja, f (1) = 0.
b) Observe que a equac
ao funcional dada nos d
a permicao para operar apenas com 2
n
umeros (a e b). Podemos mostrar, utilizando inducao, que tambem sera possvel operar
com qualquer quantidade finita (no mnimo 2) de n
umeros. Supondo ser possvel para
k n
umeros, ou seja, que
f (a1 ) + ... + f (ak ) = f (a1 ... ak ) ,
podemos garantir que

f (a1 ) + ... + f (ak ) + f (ak+1 ) = f (a1 ... ak ) + f (ak+1 ) = f (a1 ... ak ak+1 ) .
Assim, podemos escrever
f (an ) = f (a
... a}) = f (a) + ... + f (a) = n f (a).
| {z
{z
}
|
n


POT 2012 - Algebra
- Nvel 2 - Aula 11 - Prof. Marcelo Mendes

c) f (a) + f



 
 
1
1
1
=f a
= f (1) = 0, ou seja, f
= f (a).
a
a
a

Problema 4. Seja f : Z Z uma funcao satisfazendo


f (n2 ) = f (n + m)f (n m) + m2 , m, n Z.
Determine o conjunto de todos os possveis valores de f (0).
Problema 5. Seja f uma func
ao com duas propriedades:
i. f (x + y) = x + f (y), x, y R;
ii. f (0) = 2.
Determine o valor de f (2012).
Problema 6. (EUA) Seja f uma func
ao satisfazendo f (xy) =

f (x)
para todos os n
umeros
y

reais positivos x e y. Se f (500) = 3, qual e o valor de f (600)?


Problema 7. Para todos os inteiros x, a funcao f satisfaz f (x+1) =
calcule f (2012).

1 + f (x)
. Se f (1) = 2,
1 f (x)

Soluc
ao. Com:
x = 1, obtemos f (2) =

1+3
= 2;
13

x = 2, obtemos f (3) =

12
1
= ;
1+2
3

x = 3, obtemos f (4) =

1
1+

1
3
1
3

x = 4, obtemos f (5) =

1+
1

1
2
1
2

= 3.

1
;
2

Como o valor 3 apareceu novamente e a regra que calcula cada novo valor e sempre a
1 1
mesma, os valores vao sempre se repetir de 4 em 4 na sequencia 3, 2, , . Por 2012 ser
3 2
1
m
ultiplo de 4, segue que f (2012) = f (4) = .
2
Problema 8. Determine todas as funcoes f : R R satisfazendo
f (x)f (y) f (xy) = x + y, x, y.
Soluc
ao. Atribuindo o valor 0 a x e a y, temos (f (0))2 f (0) = 0. Assim, f (0) = 0 ou
f (0) = 1.


POT 2012 - Algebra
- Nvel 2 - Aula 11 - Prof. Marcelo Mendes

Suponha f (0) = 0 e atribua 0 apenas `a vari


avel y. Da, f (x) 0 0 = x, x R,
absurdo. Portanto, f (0) = 1.
Fazendo y = 0, obtemos f (x)f (0) f (0) = x + 0, ou seja, f (x) = x + 1, que e a u
nica
solucao. (Substitua essa soluc
ao na equacao funcional para verificar a igualdade!)
Problema 9. Considere uma func
ao f definida no conjunto dos n
umeros naturais tal que
f (n + 2) = 3 + f (n), n N, f (0) = 10, f (1) = 5.
Qual o valor de

f (81) f (70)?

Problema 10. Seja f : Q+ Q+ uma funcao tal que


f (xf (y)) =
Mostre que f (f (x)) =

f (x)
, x, y Q+ .
y

1
, x Q+ .
x

Soluc
ao. Temos
y f (xf (y)) = f (x) f (f (x)) = f (y f (xf (y))) =

1
f (y)
= .
xf (y)
x

Problema 11. A func


ao f e definida para todos os pares ordenados (x, y) de inteiros positivos e tem as seguintes propriedades:
i. f (x, x) = x,
ii. f (x, y) = f (y, x),
iii. (x + y)f (x, y) = yf (x, x + y).
Qual e o valor de f (22, 55)?
Soluc
ao. De iii, obtemos
55f (22, 33) = 33f (22, 55)
33f (22, 11) = 11f (22, 33)
22f (11, 11) = 11f (11, 22),
utilizando ii no lado direito da u
ltima equacao. Multiplicando-as, obtemos:
55 33 22f (11, 11) = 33 11 11f (22, 55)
f (22, 55) = 110.


POT 2012 - Algebra
- Nvel 2 - Aula 11 - Prof. Marcelo Mendes

Problema 12. A func


ao f , definida sobre o conjunto de todos os pares ordenados de inteiros
positivos, satisfaz as seguintes propriedades:
i. f (x, x) = x;
ii. f (x, y) = f (y, x);
iii. (x + y)f (x, y) = yf (x, x + y).
Calcule f (14, 52).
Problema 13. Se
f (n + 1) = (1)n+1 n 2f (n)

para os inteiros n 1 e f (1) = f (2013), determine valor da soma


f (1) + f (2) + f (3) + ... + f (2012).
Problema 14. Determine todas as funcoes f : Z+ Z+ , injetoras, satisfazendo
f (m + f (n)) = f (f (m)) + f (n), m, n Z+ .
Soluc
ao. Atribuindo o valor 0 `
as vari
aveis m e n, temos
f (f (0)) = f (f (0)) + f (0) f (0) = 0.

Agora, atribuindo o valor 0 apenas `a vari


avel n, obtemos
f (m) = f (f (m)).

Como f e injetora, segue que f (m) = m, m. (Se f e uma funcao injetora, entao
f (x) = f (y) implica x = y.)
Problema 15. Considere a equac
ao funcional
f (xy) = f (x) + f (y), x, y Dom(f ).
Mostre que se 0 Dom(f ), ent
ao existe uma u
nica solucao para a equacao dada.

Problema 16. Seja f : N N uma funcao estritamente crescente (isto e, x < y f (x) <
f (y)) tal que f (2) = 2 e f (mn) = f (m)f (n) para todo par de inteiros positivos m e n
primos entre si. Determine o valor de f (3).
Soluc
ao. Inicialmente, observe que o problema n
ao permite escrever f (4) = f (2)f (2) = 4,
pois o m
aximo divisor comum entre 2 e 2 n
ao e 1.
i. 2 e 5 s
ao primos entre si: f (10) = f (2)f (5) = 2f (5).
ii. 2 e 9 s
ao primos entre si: f (18) = f (2)f (9) = 2f (9).
iii. 3 e 5 s
ao primos entre si: f (15) = f (3)f (5).
iv. f e estritamente crescente: f (9) < f (10), f (15) < f (18) e f (3) > f (2) = 2.
Logo, 4f (5) = 2f (10) > 2f (9) = f (18) > f (15) = f (3)f (5) e, portanto, f (3) < 4.
Assim, 2 < f (3) < 4, ou seja, f (3) = 3.
5


POT 2012 - Algebra
- Nvel 2 - Aula 11 - Prof. Marcelo Mendes

Dicas

2. Calcule f (1). Depois faca y = 1.


4. Faca m = n = 0. Em seguida, faca m = n = 2.
5. Faca x = 1, y = 0. Depois faca y = 1 e deixe x vari
avel.
6. Faca x = 100 e y = 5.
9. Calcule as imagens de n
umeros pares e mpares separadamente.
12. Repita a soluc
ao do problema 11.
13. Faca n = 1, 2, ..., 2011.
15. Faca y = 0.


POT 2012 - Algebra
- Nvel 2 - Aula 11 - Prof. Marcelo Mendes

Respostas e Soluc
oes

2. Com x = y = 0, obtemos f (1) = 2. Com y = 1, segue


f (x + 1) = f (x)f (1) f (1) x + 2 = 2f (x) x.
Com x = 1 e x = 2, obtemos, respectivamente:
f (2) = 2 2 1 = 3,
f (3) = 2 3 2 = 4.
Vamos mostrar por induc
ao que f (x) = x + 1, x Z+ . Os casos iniciais ja foram
verificados. Alem disso, f (x+1) = 2(x+1)x = x+2. Em particular, f (2012) = 2013.
4. Inicialmente, com m = n = 0 temos f (0) = (f (0))2 f (0) = 0 ou f (0) = 1. Por
outro lado, com m = n = 2 obtemos f (4) = f (4)f (0) + 4, que mostra que f (0) = 1.
5. Com x = 1, y = 0, achamos f (1) = f (1 + 0) = 1 + f (0) = 3. Em seguida, escolhemos
apenas y = 1, obtemos f (x + 1) = x + f (1) = x + 3, que e uma P.A. se x Z+ .
Assim, f (2012) = f (0) + 2012 3 = 6038.
6. f (500) =

f (100)
5
f (100)
f (100) = 15. Logo, f (600) =
= .
5
6
2

9. Somando
f (2) = 3 + f (0)
:
f (70) = 3 + f (68)
obtemos f (70) = 3 35 + 10 = 115. E pela soma de
f (3) = 3 + f (1)
:
f (81) = 3 + f (79)
p

obtemos f (81) = 3 40 + 5 = 125. Logo, f (81) f (70) = 10.

12. 364.


POT 2012 - Algebra
- Nvel 2 - Aula 11 - Prof. Marcelo Mendes

13. Com n = 1, 2, ..., 2012, temos:


f (2) = 1 2f (1)
f (3) = 2 2f (2)
:
f (2013) = 2012 2f (2012).
Sendo f (1) = f (2013) e S = f (1) + ... + f (2012), obtemos:
S = (1 2) + ... + (2011 2012) 2S S =
15. Com y = 0, obtemos f (x) = 0, x.

1006
.
3

Polos Olmpicos de Treinamento


Aula

Curso de lgebra - Nvel 2


Prof. Marcelo Mendes

Sequ
encias
Uma sequencia nada mais e do que um conjunto de n
umeros ordenados. Assim, podemos estabelecer um primeiro termo (a1 ), um segundo termo (a2 ), ... e o termo geral de
uma sequencia e escrito na forma an . Os problemas costumam informar qual e o valor de
alguns termos e uma lei de formac
ao para os demais termos. Se necessario, faremos uso de
termos, que na sequencia, s
ao anteriores aos termos dados ou posteriores (que ser
a mais
raro).
Algumas vezes, essa lei de formac
ao ser
a implcita, ou seja, n
ao poderemos calcular os
termos diretamente a partir da posic
ao que eles ocupam na sequencia. Por exemplo, se
cada termo e a soma dos dois termos imediatamente anteriores e os primeiro e segundo
termos s
ao iguais a 1. Possivelmente, precisaremos de uma lei explcita, que calcula um
termo da sequencia apenas a partir da posicao que ele ocupa.
No par
agrafo anterior, a sequencia em questao e a famosa Sequencia de Fibonacci. Na
pr
oxima aula, vamos aprender como encontrar seu termo geral.

1 Sequ
encias simples
Problema 1. Mostre que a sequencia definida por an = n2 + n + 2 para n 1, entao na
sequencia a1 , a2 , a3 , ... contem an quadrado perfeito, mas apenas em quantidade finita.
Soluc
ao. Inicialmente, veja que a1 = 4, que e quadrado perfeito. Mas para n > 1, ocorre
n2 < n2 + n + 2 < n2 + 2n + 1,
ou seja, an est
a situado entre 2 quadrados perfeitos consecutivos e, portanto, n
ao pode ser
um quadrado.
Problema 2. Uma sequencia {an } e definida por a1 = 2 e, para n 2, an e o maior divisor
primo de a1 a2 ... an1 + 1. Mostre que an nunca e igual a 5.


POT 2012 - Algebra
- Nvel 2 - Aula 3 - Prof. Marcelo Mendes

Soluc
ao. O m
aximo divisor primo de a1 +1 = 3 e a2 = 3. Logo, se n > 2, a1 a2 ...an1 +1
n
ao possui fatores 2 nem 3, ou seja, se an = 5, entao a1 a2 ... an1 + 1 = 5k ou
a1 a2 ... an1 = 5k 1, que e m
ultiplo de 4, uma contradicao pois o u
nico fator par do
membro esquerdo dessa u
ltima equac
ao e a1 = 2.
Problema 3. (OBM) Considere a sequencia oscilante:
1, 2, 3, 4, 5, 4, 3, 2, 1, 2, 3, 4, 5, 4, 3, 2, 1, 2, 3, 4, ...
Determine o 2003o termo desta sequencia.
Soluc
ao. Uma parte da sequencia, com 8 algarismos, se repete: 1, 2, 3, 4, 5, 4, 3, 2. Dividindo 2003 por 8, obtemos 3 como resto, e deste modo, o 2003o termo corresponde ao
terceiro elemento da parte da sequencia que se repete, isto e, 3.
Problema 4. (OBM-Adaptado) A sequencia de algarismos 1, 2, 3, 4, 0, 9, 6, 9, 4, 8, 7, ... e
construda da seguinte maneira: cada elemento, a partir do quinto, e igual ao u
ltimo
algarismo da soma dos quatro anteriores. Os algarismos 2, 0, 0, 4, juntos e nesta ordem,
aparecem na sequencia?
Problema 5. Calcule a soma 1 2 + 3 4 + . . . 98 + 99 100.

2 Somas Telesc
opicas
Vamos entender o que e uma soma telesc
opica atraves do nosso primeiro exemplo.

Problema 6. (EUA) Se F (n + 1) =
mine o valor de F (101).

2F (n) + 1
para n = 1, 2, ..., e F (1) = 2, entao deter2

Soluc
ao. Podemos reescrever a equac
ao que define os termos dessa sequencia recursivamente (isto e, em func
ao de termos anteriores) da seguinte forma:
1
F (n + 1) F (n) = .
2
Assim, podemos escrever essas equacoes variando n de 100 a 1:
1
2
1
F (100) F (99) =
2
:
1
F (3) F (2) =
2

F (101) F (100) =


POT 2012 - Algebra
- Nvel 2 - Aula 3 - Prof. Marcelo Mendes
1
2
Somando telescopicamente todas essas equacoes, obtemos F (101) F (1) = 50, ou seja,
F (101) = 52 pois F (1) = 2.
F (2) F (1) =

Sequencias como essa que acabamos de ver em que a diferenca entre os valores dos
termos consecutivos e constante s
ao chamadas de Progress
ao Aritmetica (P.A.).
Acho que deu pra entender o que e uma soma telesc
opica: s
ao somas em que os termos
intermediarios s
ao cancelados e, no final, s
o restam o primeiro e o u
ltimo.
Pode ate mesmo ser interessante escrever coisas do tipo
1 n = (1 2) + (2 3) + ... + [(n 1) n] .
Vejamos agora mais um exemplo.
Problema 7. Encontre o valor da soma
S=

1
1
1
1
+
+
+ ... +
.
12 23 34
999 1000

Soluc
ao. Essa e uma aplicac
ao cl
assica para somas telesc
opicas. Observe que os denomi1
=
nadores s
ao produtos de n
umeros consecutivos. Com o auxlio da identidade
k (k + 1)
1
1

, conclumos que
k k+1
S=

1 1 1 1 1 1
1
1
1
999
+ + + ... +

S =1
=
.
1 2 2 3 3 4
999 1000
1000
1000

Problema 8. (EUA) Encontre a soma

1
1
1
1
+
+
+ ... +
.
13 35 57
255 257

Problema 9. (OBM) Encontre a soma

1
1
1
1
+
+
+ ... +
.
1 4 4 7 7 10
2998 3001

Problema 10. (Hungria) Prove que para todos os inteiros positivos n,


1
1
1
1
1
1
+
+ ... +
=
+
+ ... +
.
12 34
(2n 1) 2n
n+1 n+2
2n
Soluc
ao. Veja
1
1
1
1 1 1
1
1
+
+ ... +
= 1 + + ... +

12 34
(2n 1) 2n
2 3 4
2n 1 2n


1 1
1
1
1
1 1 1
+
2
+ + ... +
= 1 + + + + ... +
2 3 4
2n 1 2n
2 4
2n
3


POT 2012 - Algebra
- Nvel 2 - Aula 3 - Prof. Marcelo Mendes

=1+



1
1
1
1
1 1 1
+ + + ... +
+
1 + + ... +
2 3 4
2n 1 2n
2
n

1
1
1
+
+ ... +
.
n+1 n+2
2n
Observe que, apesar de muito semelhante aos problemas anteriores, este n
ao utiliza
soma telesc
opica.
=

Problema 11. O pagamento de um certo pintor aumenta de acordo com o dias em que
ele trabalha. No primeiro dia ele recebeu 1 real. no segundo dia ele recebeu o que tinha ganho no primeiro dia mais 2 reais. No terceiro dia ele recebeu o que tinha recebido
no segundo dia mais 3 reais. Desse modo, quanto o marceneiro ir
a receber no centesimo dia?
Soluc
ao. Seja Ln o valor pago no n-esimo dia. O problema no diz que Ln+1 = Ln + (n + 1).
Vamos escrever varias equac
oes seguidas:
Ln+1
Ln
Ln1
L2

=
=
=
...
=

Ln + (n + 1)
Ln1 + n
Ln2 + (n 1)
L1 + 2

Somando tudo, obtemos um cancelamento de varios termos (soma telesc


opica), sobrando:
(n + 1)(n + 2)
Ln+1 = (n + 1) + n + (n 1) + . . . + 2 + 1 =
.
2
Problema 12. Prove que S =

1
1
1
+
+. . .+

e um n
umero inteiro.
1+ 2
2+ 3
99 + 100

Soluc
ao. A dica e racionalizac
ao dos denominadores:



1 2
1
1

=

=
1 2 .
1+ 2
1+ 2
1 2
Repetindo o procedimento para as demais parcelas, chegamos a:
S =

1 2 + 2 3 + ... + 99 100 = 1 100 = 99


S = 99,

que e um n
umero inteiro.
Problema 13. Determine o valor da express
ao


2002
2002
2002
2002
E=
+
+
+ ... +
26
6 10 10 14
1998 2002
4

1
+ 1 + 1 +...+ 1
1+ 2
2+ 3
3+ 4
99+ 100


POT 2012 - Algebra
- Nvel 2 - Aula 3 - Prof. Marcelo Mendes

Problema 14. (EUA) A Sequ


encia de Fibonacci 1, 1, 2, 3, 5, 8, 13, 21, ... comeca com dois
1s e cada termo seguinte e a soma de seus dois antecessores. Qual dos dez dgitos (do sistema de numerac
ao decimal) e o u
ltimo a aparecer na posicao das unidades na seq
uencia
de Fibonacci?
Problema 15. (OBM) Determine o m
aximo divisor comum de todos os termos da sequencia
cujos termos s
ao definidos por an = n3 n.
Problema 16. (EUA) Considere uma sequencia un definida por u1 = 5 e a relacao
un+1 un = 3 + 4(n 1), n = 1, 2, 3, ...
Se un e expresso como um polin
omio em n, determine a soma algebrica de seus coeficientes.
Soluc
ao. Podemos escrever
un un1 = 3 + 4(n 2)
un1 un2 = 3 + 4(n 3)
:
u2 u1 = 3 + 4 1
Somando todas essas equac
oes, obtemos
un u1 = 3(n 1) + 4 (1 + 2 + ... + (n 2)) = 3(n 1) + 2(n 1)(n 2)
un = 2n2 3n + 6,
cuja soma dos coeficientes e 5.
Problema 17. (Estonia) Prove a desigualdade
2010 <

22 + 1 32 + 1
20102 + 1
1
+
+
...
+
< 2010 .
2
2
2
2 1 3 1
2010 1
2

Problema 18. Calcule a soma

n
X

(k
+
1)
k
+
k
k
+
1
k=1


POT 2012 - Algebra
- Nvel 2 - Aula 3 - Prof. Marcelo Mendes

Problema 19. Considere a seq


uencia definida por a1 = 1 e an+1 =

an
1+nan .

Calcule a2012 .

Soluc
ao. Comecaremos com um artifcio algebrico bastante u
til que e observar que, na
formula de an+1 , a frac
ao do membro direito pode ser melhor desenvolvida se for invertida,
porque poderemos desmembrar o resultado. De fato, temos
1
an+1

1 + n an
1
=
+n
an
an
1
an+1

1
= n.
an

Assim, obtemos uma chamada equaca


o de diferenca. Variando o valor de n de forma
decrescente de 2010 a 1, chegaremos a
1
a2011
1
a2011

..
.

1
a2010
1
a2010

1
a3
1
a2

1
a2
1
a1

= 2010
= 2009
..
=
.
=
=

2
1

Somando essas 2010 equac


oes membro a membro, obtemos
1
a2011

1
2010 2011
= 1 + 2 + ... + 2009 + 2010 =
= 2021055
a1
2

Portanto, a2011 =

1
a2011

= 2021056.

1
2021056 .


POT 2012 - Algebra
- Nvel 2 - Aula 3 - Prof. Marcelo Mendes

3 Produtos Telesc
opicos
A ideia e semelhante a das somas telesc
opicas, mas o cancelamento ocorre pelo produto
e n
ao por soma.
Problema 20. No ano 1 Papai Noel viajou sozinho para entregar seus presentes na noite
de Natal. No ano seguinte, ele percebeu que precisava de um ajundante e contratou um
Matesito (tpico habitante do P
olo Norte). A cada ano, ele sempre precisava dobrar a quantidade de Matesitos e contratava mais Matesitos para guiar as renas. Quantos Matesitos
Papai Noel vai precisar contratar no ano de 2012?
Soluc
ao. Seja Ln o n
umero de Matesitos em cada ano. O problema no diz que Ln+1 =
2Ln + 1. Somando 1 aos dois lados obtemos Ln+1 + 1 = 2(Ln + 1). Vamos escrever varias
equacoes seguidas:
Ln+1 + 1
Ln + 1
Ln1 + 1

=
=
=
...
L2 + 1 =

2(Ln + 1)
2(Ln1 + 1)
2(Ln2 + 1)
2(L1 + 1)

Multiplicando tudo, obtemos um cancelamento de varios termos (produto telesc


opico),
sobrando:
Ln+1 + 1 = |2 2 {z
2 . . . 2} = 2n+1 Ln+1 = 2n+1 1.
n+1vezes

Em particular, L2012 = 22012 1.

Problema 21. Uma sequencia e definida por a1 = 2 e an = 3an1 + 1. Determine a soma


a1 + a2 + . . . + an .
2
Problema 22. Considere
p a sequencia recorrente definida por a1 = 14 e an+1 = an 2.
2
Prove que o n
umero 3 (an 4) e divisvel por 4, n Z, n 1.
q

Soluc
ao. Primeiro, veja que 3 a21 4 = 24. Observe que

an+1 2 = a2n 4 = (an + 2)(an 2).


Reduzindo os ndices, obtemos tambem
an 2 = (an1 + 2)(an1 2)
:
a3 2 = (a2 + 2)(a2 2)
a2 2 = (a1 + 2)(a1 2)
7


POT 2012 - Algebra
- Nvel 2 - Aula 3 - Prof. Marcelo Mendes

Multiplicando todas essas equac


oes telescopicamente, obtemos
an+1 2 = (an + 2)(an1 + 2)...(a1 + 2)(a1 2)

que e m
ultiplo de 4.

a2n 4 = a2n1 a2n2 ... 16 12



3 a2n 4 = a2n1 a2n2 ... 16 36.
p
3 (a2n 4) = an1 an2 ... 4 6,

2
2, n 2. Se sn = rn 2 para n 1, prove que
Problema 23. Sejam r1 = 3 e rn = rn1
j2
sj tem, no mnimo, 2 3
divisores positivos, j 2.

Problema 24. (EUA) Defina uma sequencia de n


umeros reais a1 , a2 , a3 , ... por a1 = 1 e
a3n+1 = 99a3n , n 1. Determine o valor de a100 .





104 + 324 224 + 324 344 + 324 464 + 324 584 + 324
Problema 25. Calcule o valor de
.
(44 + 324) (164 + 324) (284 + 324) (404 + 324) (524 + 324)
Problema 26. Qual e o valor do produto

4n + 4
2008
8 12 16

...
...
?
4 8 12
4n
2004


POT 2012 - Algebra
- Nvel 2 - Aula 3 - Prof. Marcelo Mendes

Dicas

5. Agrupe os n
umeros aos pares.


1 1
1
1
=

8. Use
.
k (k + 2)
2 k k+2
9. Pense numa ideia semelhante `
a sugestao do problema 8.
13. Use mais uma vez uma ideia parecida com a do problema 8 e veja o problema 12.
14. Calcule os primeiros termos ate chegar `a resposta.
1
1
n2 + 1
=1+

.
(n 1)(n + 1)
n1 n+1

18. Fatore o denominador pondo k k + 1 em evidencia.
racionalize o denomi Depois,
nador multiplicando numerador e denominador por k + 1 k e surgira uma soma
telesc
opica.

17. Use

21. Subtraindo as equac


oes an = 3an1 + 1 e an1 = 3an2 + 1, obtemos an an1 =
3 (an1 an2 ). Depois, multiplique varias dessas equacoes seguidas (produto telesc
opico).
23. Veja problema 21.
24. Multiplique varias dessas equac
oes seguidas (produto telesc
opico).


2
25. Use a4 +182 = a4 +2a2 18+182 36a2 = a2 + 18 (6a)2 = a2 + 6a + 18 a2 6a + 18 .


POT 2012 - Algebra
- Nvel 2 - Aula 3 - Prof. Marcelo Mendes

Respostas

4. Nao
5. 50
8.

128
257

9.

1000
3001

13. 2002

99

14. 6
15. 6
18.

n+11

n+1

21.

53n 2n5
4

24. 9933
25. 373
26. 502

10

Polos Olmpicos de Treinamento


Aula

Curso de lgebra - Nvel 2


Prof. Marcelo Mendes

12

Fun
c
oes Definidas Implicitamente - Parte II
Nesta segunda aula sobre func
oes definidas implicitamente, aprofundaremos alguns
exerccios vistos na aula anterior e veremos exemplos envolvendo funcoes compostas, ou
seja, funcoes de outras func
oes.
Iniciaremos resolvendo o problema 1 da aula anterior mas, agora, com domnio racional.
Problema 1. Determine todas as funcoes f : Q Q tais que f (1) = c e
f (x + y) = f (x) + f (y), x, y Q.
Soluc
ao. J
a vimos na aula anterior que f (x) = cx quando x e inteiro n
ao-negativo.
Agora, suponha x inteiro negativo. Fazendo y receber o valor contr
ario de x (assim,
y = x > 0), obtemos
0 = f (0) = f (x) + f (x)()
f (x) = f (x) = (c(x)) = cx,
e o resultado tambem e valido para inteiros negativos.
Suponha, agora, x inteiro positivo. Podemos escrever

 
 
 
1
1
1
1
1
+ ... + f
= xf
f (1) = f + ... + = f
x
x
x
x
x
|
{z
}
|
{z
}
x

 
1
1
f
=c ,
x
x

lembrando que no problema 3 da aula anterior ja mostramos por inducao para uma equacao
funcional semelhante que podemos aplicar a regra, dada no problema para 2 n
umeros, para


POT 2012 - Algebra
- Nvel 2 - Aula 12 - Prof. Marcelo Mendes
 
1
1
= c , x Z .
qualquer quantidade finita de n
umeros. Por (*), segue que f
x
x
p
Seja agora x racional n
ao-nulo, isto e, x = , com p e q inteiros, sendo p > 0. Temos
q

 
1
p
1

f (x) = f
= f + ... +
q
q
q
| {z }
p

 
 
 
1
1
1
1
=f
= pf
+ ... + f
= p c = cx,
q
q
q
q
|
{z
}
p

finalizando a soluc
ao.

Problema 2. Determine todas as funcoes f : Q Q tais que f (1) = a e


f (x + y) = f (x) f (y), x, y Q.
Problema 3. Determine todas as funcoes f : R R satisfazendo

f (x) + f (2x + y) + 5xy = f (3x y) + 2x2 + 1, x, y R.

Soluc
ao. Fazendo x = 2y, obtemos 2x + y = 3x y e
5x2
x2
= 2x2 + 1 f (x) = 1 .
2
2
Problema 4. Seja f : R R uma func
ao satisfazendo a equacao funcional

2
f (x y) = (f (x))2 2x f (y) + y 2 .
f (x) +

Determine todos os possveis valores de f (0).

Problema 5. (ITA) Sejam f, g : R R funcoes tais que g(x) = 1 x e


f (x) + 2f (2 x) = (x 1)3 , x R.

Determine f (g(x)).
Soluc
ao. A equac
ao funcional em questao possui uma composicao de funcoes em f (2 x).
Sendo h(x) = 2 x, temos f (2 x) = f (h(x)). Essa funcao h e a chave do problema.
Reescreva a equac
ao substituindo x por h(x) = 2 x, ou seja, x 2 x (le-se: x recebe
2 x):
f (2 x) + 2f (x) = (1 x)3 .

Dessa e da equac
ao dada, obtemos f (x) = (1 x)3 e, portanto,
f (g(x)) = (1 (1 x))3 = x3 .

Obs: h(h(x)) = 2 h(x) = x. Por isso s


ao suficientes apenas 2 equacoes para encontrarmos
f (x).
2


POT 2012 - Algebra
- Nvel 2 - Aula 12 - Prof. Marcelo Mendes


Problema 6. Se 2f x2 + 3f

1
x2


= x2 1, determine f x2 .

Problema 7. Se f (x) + 2f (1 x) = x2 + 2, x R, determine f (x).




1
Problema 8. Determine f sabendo que f (x) + f
= x, x 6= 1.
1x
1
. Nesse caso, temos
Soluc
ao. Novamente, a chave do problema est
a em fazer x
1x
x1
1
1
=

. Assim:
1
1x
x
1
1x




1
x
1
f
.
+f
=
1x
x1
1x

Mas diferentemente dos problemas mais simples, envolvendo funcoes mais simples, n
ao
podemos combinar apenas essa u
ltima equacao com a equacao dada porque ainda n
ao obtivemos outro f (x).
Assim, vamos novamente promover a substituicao x

(verifique!). Logo:

x
x1

+ f (x) =

1
x1
, que nos d
a
x
1x
x

x1
.
x

Combinando as equac
oes, obtemos

2f (x) = x
f (x) =

x1
1
+
1x
x
x3 x + 1
.
2x(x 1)

x1
1
h(h(x)) =
e h(h(h(x))) = x. Por isso s
ao necessarias 3
1x
x
equacoes para encontrarmos f (x).

Obs: h(x) =

Problema 9. Determine a func


ao f : R R satisfazendo `a equacao funcional
x2 f (x) + f (1 x) = 2x x4 .
Problema 10. (Ibero) Ache todas as f tais que

(f (x))2

1x
1+x

= 64x, para todo x

distinto de 0, 1 e -1.
Problema 11. (EUA) A func
ao fn
ao est
a definida para x = 0, mas para todos os valores de
1
= 3x. Determine as solucoes da equacao f (x) = f (x).
x n
ao-nulos temos f (x) + 2f
x


POT 2012 - Algebra
- Nvel 2 - Aula 12 - Prof. Marcelo Mendes

Problema 12. Seja f uma func


ao cujo domnio e o conjunto de todos os n
umeros reais. Se


x + 2011
f (x) + 2f
= 4033 x
x1
para todo x 6= 1, encontre o valor de f (2013).
Problema 13. Encontre todas as funcoes f : R {1, 1} R satisfazendo f


x+3
f
= x, x 6= 1.
x1

x3
x+1

Soluc
ao. Faca x x:



x + 3
x 3
+f
= x
f
x + 1
x 1




x3
x+3
+f
= x, x 6= 1.
f
x1
x+1


Portanto, x = x, x 6= 1, absurdo. Logo, n


ao existe funcao f cumprindo a equacao
funcional dada.
Problema 14. Determine
todas
as funcoes f reais, de vari
avel real positiva, satisfazendo a


2002
= 3x.
condicao f (x) + 2f
x
Problema 15. Determine todas as funcoes f : R R tais que 2f (x) + f (1 x) = 1 + x.


POT 2012 - Algebra
- Nvel 2 - Aula 12 - Prof. Marcelo Mendes

Dicas

2. Repita os procedimentos do problema 1.


4. Faca x = y = 0.
6. Faca x

1
.
x

7. Faca x 1 x.
9. Faca x 1 x.
10. Faca x

1x
.
1+x

11. Faca x

1
.
x

12. Faca x

x + 2011
.
x1

14. Faca x

2002
.
x

15. Faca x 1 x.


POT 2012 - Algebra
- Nvel 2 - Aula 12 - Prof. Marcelo Mendes

Respostas e Soluc
oes

2. f (x) = ax .
4. f (0) = 0 ou f (0) = 1.


1 x2 3 + 2x2
6. f (x) =
.
5x2
7. f (x) =

(x 2)2
3

9. Fazendo x 1 x e combinando com a equacao dada, chegamos a f (x) = 1 x2 .


r
2
3 x (1 + x)
.
10. f (x) = 4
1x

1
e combinando o resultado com a equacao inicial, obtemos f (x) =
11. Fazendo x
x

2
x. Assim, as soluc
oes de f (x) = f (x) s
ao 2.
x

12. 2014.
14. Fazendo x

4004
2002
e combinando com a equacao inicial, obtemos f (x) =
x.
x
x

15. f (x) = x.

Polos Olmpicos de Treinamento


Curso de lgebra - Nvel 2
Prof. Marcelo Mendes

Aula

13

Revis
ao - Parte I
Como o ttulo indica, faremos uma breve revisao de temas j
a abordados em nosso treinamento, a fim de consolidar conceitos e ideias importantes para a fase final da OBM.
Iniciaremos resolvendo os problemas das OBMs passadas, propostos na 1a aula sobre
produtos not
aveis, mas n
ao resolvidos.
Ao final, est
ao propostos mais alguns problemas semelhantes aos tratados ate agora.
Problema 1. (OBM 1a fase/2002) Se xy = 2 e x2 + y 2 = 5, entao
a)

5
2

b)

25
4

c)

5
4

d)

1
2

y2
x2
+
+ 2 vale:
y2
x2

e) 1
Soluc
ao. Veja que

x2
y2
x4 + y 4 + 2x2 y 2
(x2 + y 2 )2
25
+
+
2
=
=
= . Letra B.
2
2
2
2
2
2
y
x
x y
x y
4

Problema 2. (OBM 3a fase/2003) Mostre que x2 + 4y 2 4xy + 2x 4y + 2 > 0 quaisquer


que sejam os reais x e y.
Soluc
ao. Procurando agrupar os termos para obter uma fatoracao, podemos escrever
x2 + 4y 2 4xy + 2x 4y + 2 = x2 4xy + 4y 2 + 2(x 2y) + 2


POT 2012 - Algebra
- Nvel 2 - Aula 13 - Prof. Marcelo Mendes
= (x 2y)2 + 2(x 2y) + 1 + 1 = [(x 2y) + 1]2 + 1 1 > 0,
pois [(x 2y) + 1]2 e n
ao-negativo ja que e o quadrado de um n
umero real.
Problema 3. (OBM 2a fase/2005)
a) Fatore a express
ao x2 9xy + 8y 2 .
b) Determine todos os pares de inteiros (x; y) tais que 9xy x2 8y 2 = 2005.
Soluc
ao.
a) x2 9xy + 8y 2 = x2 xy 8xy + 8y 2 = x(x y) 8y(x y) = (x y)(x 8y).
b) Pelo item a, podemos escrever
(x y)(8y x) = 2005 = 5 401,
e 401 e primo.

xy
i.
8y x

xy
ii.
8y x

xy
iii.
8y x

xy
iv.
8y x

Agora vejamos as possibilidades:


= 5
x = 63, y = 58.
= 401
= 5
x = 63, y = 58.
= 401

= 401
x = 459, y = 58.
= 5

= 401
x = 459, y = 58,
= 5

que s
ao todos os casos.
Problema 4. (OBM 1a fase/2005) Os inteiros positivos x e y satisfazem a equacao
r
r
1
1
x+
y x
y = 1.
2
2
Qual das alternativas apresenta um possvel valor de y?
a) 5
b) 6
c) 7
d) 8
e) 9
2


POT 2012 - Algebra
- Nvel 2 - Aula 13 - Prof. Marcelo Mendes
q
q

Soluc
ao. Talvez fiquem interessantes as substituicoes x + 21 y = A e x 12 y = B.

y+1

2
2
e,
Assim, temos A B = 1 e A B = y, que implica A + B = y. Da, A =
2
portanto:

y+2 y+1
1
y + 1 1
x+
y=
y
=
+
2
4
4
2
y = 4x 1.
Dentre as opc
oes, a u
nica que deixa resto -1 na divisao por 4 e 7. Letra C.
Problema 5. (OBM 3a fase/2006) Encontre todos os pares ordenados (x; y) de inteiros tais
que x3 y 3 = 3(x2 y 2 ).
Soluc
ao. Fatorando, obtemos (x y)(x2 + xy + y 2 ) = 3(x y)(x + y).
Assim, temos soluc
ao quando x = y, x R.
Se x 6= y, ent
ao x2 +xy +y 2 = 3(x+y) x2 +x(y 3)+y 2 3y = 0, cujo discriminante e
= 3y 2 + 6y + 9. Para que as razes sejam reais, devemos ter 0, ou seja, 1 y 3.
Testando, apenas y = 1 n
ao gera quadrado perfeito. Os demais valores d
ao as seguintes
solucoes:
y = 1, = 0, x = 2;
y = 0, = 9, x = 0; x = 6;
y = 2, = 9, x = 2; x = 1;
y = 3, = 0, x = 0.
S
Resposta: {(2, 0); (0, 0); (6, 0); (2, 2); (2, 1); (3, 0)} {(x, x), x R}.
Problema 6. (OBM 2a fase/2006) Sejam a e b n
umeros reais distintos tais que a2 = 6b+5ab
e b2 = 6a + 5ab.
a) Determine o valor de a + b.
b) Determine o valor de ab.
Soluc
ao.
a) Subtraindo as equac
oes membro a membro, obtemos a2 b2 = 6(b a). Como a e b s
ao
distintos, chegamos a a + b = 6.
b) Agora, somando as equac
oes membro a membro, temos a2 + b2 = 6(a + b) + 10ab
2
(a + b) = 6(a + b) + 12ab, o que d
a ab = 0.
3


POT 2012 - Algebra
- Nvel 2 - Aula 13 - Prof. Marcelo Mendes

Problema 7. (OBM 2a fase/2008) Sejam x e y n


umeros reais positivos satisfazendo as
1
17
2
2
4
4
.
equacoes x + y = 1 e x + y = 18 . Calcule o valor de xy
Soluc
ao. De x2 + y 2 = 1, obtemos
1
1
17
=
x2 y 2 =
.
18
18
36
1
1
Como x e y s
ao positivos, segue que xy = e
= 6.
6 xy
x4 + y 4 + 2x2 y 2 = 1 2x2 y 2 = 1

Problema 8. (OBM 1a fase/2010) Quantos s


ao os pares (x, y) de inteiros positivos tais que
x2 y 2 = 22010 ?
a) 1000
b) 1001
c) 1002
d) 1003
e) 1004
Soluc
ao. Temos (x + y)(x y) = 22010 . Como x + y > 0, entao x y > 0. Alem disso,
x + y > x y (pois y > 0) e x + y deve ter a mesma paridade de x y.
Assim, as possibilidades s
ao

x + y = 22009 , 22008 , ... , 21006
,
xy = 2
, 22
, ... , 21004
que geram 1004 pares ordenados. Letra E.
Problema 9. (OBM 3a fase/2010) Sejam a, b e c reais tais que a 6= b e a2 (b+c) = b2 (c+a) =
2010. Calcule c2 (a + b).
Soluc
ao. De a2 (b + c) = b2 (c + a) = 2010, obtemos
a2 b b2 a + a2 c b2 c = 0
ab(a b) + c(a + b)(a b) = 0
(a b)(ab + bc + ca) = 0.
Como a 6= b, conclumos que ab + bc + ca = 0.
Agora, vamos fazer o mesmo com b2 (c + a) = 2010 e c2 (a + b) = k. Subtraindo obtemos
b2 c c2 b + b2 a c2 a = 2010 k
4


POT 2012 - Algebra
- Nvel 2 - Aula 13 - Prof. Marcelo Mendes

bc(b c) + a(b + c)(b c) = 2010 k


(b c)(ab + bc + ca) = 2010 k.
Mas ab + bc + ca = 0. Assim, k = 2010.
Problema 10. (OBM 1a fase/2011) Qual e o valor da express
ao 201120112 + 201120032
16 20112007?
a) 2 201120072
b) 2 201120032
c) 2 20112007
d) 2 20112003
e) 2 201120112
Soluc
ao. Para simplificar os c
alculos, vamos escrever a = 20112007. Assim, a express
ao
desejada ser
a
(a + 4)2 + (a 4)2 16a = 2(a2 8a + 16) = 2(a 4)2 = 2 201120072 .
Letra A.
Problema 11. (IMO-Adaptado) Sejam k, m, n n
umeros naturais. Defina cs = s(s + 1).
a) Fatore cj ck , sendo j N.
b) Mostre que (cm+1 ck )(cm+2 ck )...(cm+n ck ) e divisvel por

c1 c2 ... cn
.
n+1

Problema 12. Determine todos os valores de x para os quais (1999x 99)3 = (1234x
56)3 + (765x 43)3 .
Problema 13. Simplifique a express
ao
S=

22 1 32 1 42 1
20122 1

...

.
22
32
42
20122

Problema 14. Mostre que

p
3

3
20 + 14 2 + 20 14 2 = 4.

Problema 15. Determine todas as solucoes inteiras da equacao


2(x + y) = xy + 7.


POT 2012 - Algebra
- Nvel 2 - Aula 13 - Prof. Marcelo Mendes

Dicas

11. Use o fato de que o produto de n inteiros consecutivos e divisvel por n!.
12. Resolva a equac
ao (a + b)3 = a3 + b3 .
13. Fatore os numeradores, que s
ao diferencas de quadrados.
14. Eleve ao cubo a express
ao no lado esquerdo.
15. Encontre k tal que xy 2x 2y + k possa ser fatorado.

Respostas

12.

99 43 56
, ,
.
1999 65 1234

13.

2013
.
4024

15. {(x, y) = (3, 1), (1, 5), (5, 1), (1, 3)}.

Polos Olmpicos de Treinamento


Aula

Curso de lgebra - Nvel 2


Prof. Marcelo Mendes

14

Revis
ao - Parte II
Continuando nossa breve revis
ao de temas ja abordados, propomos mais problemas de
equacoes e sistemas de equac
oes.
Problema 1. Se x R e 4y 2 + 4xy + x + 6 = 0, determine:
a) o conjunto de todos os valores de x para os quais y R;
b) y em func
ao dos valores de x encontrados no item anterior.
Problema 2. Encontre todas as soluc
oes reais da equacao

Problema 3. Determine o conjunto solucao da equacao

13 + x +

x2 + 3

Problema 4. Sejam x, y, z n
umeros reais tais que
1
y
2
=
=
.
xy
zx+1
z+1
Prove que um desses n
umeros e a media aritmetica dos outros dois.
Problema 5. Prove que a equac
ao
an + 2012 bn = cn+1
tem infinitas soluc
oes naturais a, b, c para todo inteiro positivo n.
Problema 6. (OBM) Mostre que a equacao
x3 + 1990y 3 = z 4
possui infinitas soluc
oes inteiras com x > 0, y > 0, z > 0.

4 x = 3.

x2

3
x
= , x R.
+3
2


POT 2012 - Algebra
- Nvel 2 - Aula 14 - Prof. Marcelo Mendes

Problema 7. Determine o conjunto solucao da equacao x = 1 x, onde a representa a


parte inteira de a.
p 2
px y = z 1
Problema 8. Quantas soluc
oes reais possui o sistema
y2 z = x 1 ?
2
z x = y1
Problema 9. (Banco IMO) Encontre todas as triplas de inteiros positivos x, y, z satisfazendo
4
1 1 1
+ + = .
x y z
5
Soluc
ao. Suponha, sem perda de generalidade, que x y z. Da,

4
1
1 1
3
= + + ,
5
x y z
x

4
1
< x 2. Assim, precisamos analisar os casos x = 2 e
ou seja, x 3. Tambem
x
5
x = 3.
i) x = 2:

1 1
4 1
3
10z
+ = =
y=
y z
5 2
10
3z 10

30z 100 + 100


100
30z
=
= 10 +
.
3z 10
3z 10
3z 10

Como 3y Z, segue que (3z 10) 100 (essa notacao significa 3z 10 divide 100) e
3z 10 > 0. Assim, temos as possibilidades
3y =

3z 10 = 1, 2, 4, 5, 10, 20, 25, 50, 100.


Mas z Z. Logo, restam
3z 10 = 2, 5, 20, 50
z = 4, 5, 10, 20
y = 20, 10, 5, 4,
que geram as soluc
oes (2, 4, 20), (2, 5, 10), (2, 10, 5), (2, 20, 4).
ii) x = 3:

1 1
4 1
7
15z
+ = =
y=
y z
5 3
15
7z 15
7y =

105z 225 + 225


225
105z
=
= 15 +
.
7z 15
7z 15
7z 15
2


POT 2012 - Algebra
- Nvel 2 - Aula 14 - Prof. Marcelo Mendes


Como 7y Z, segue que (7z 15) 225 e 7z 15 > 0. Assim, temos as possibilidades
7z 15 = 1, 3, 5, 15, 45, 75, 225,
7z = 16, 18, 20, 30, 60, 90, 240,
que n
ao produzem soluc
ao inteira.
Assim, as u
nicas soluc
oes s
ao (2, 4, 20), (2, 5, 10), (2, 10, 5), (2, 20, 4).
Coment
ario. Existe um problema muito parecido com esse, proposto no livro das Olimpadas
Brasileiras de Matematica - 1a `
a 8a , que possui o seguinte enunciado:
Mostre que o n
umero de soluc
oes x, y, z de inteiros positivos da equacao
1 1 1
1
+ + =
x y z
1983
e finito.
Tente resolve-lo a partir das ideias do problema 9!
Problema 10. Determine todas as solucoes em n
umeros reais x, y, z, w do sistema de
equacoes

x + y + z = w
1
1
1 .
1
+
+
=

x
y
z
w
Problema 11. (Romenia) Os n
umeros reais n
ao-nulos
dades

x + y + z =

1
1
1
+
+
=

x
y
z

x3 + y 3 + z 3 =

x, y, z, t verificam as seguintes igualt


1
.
t
10003

Determine o valor da soma x + y + z + t.

Problema 12. Seja n um dado inteiro positivo. Quantas solucoes existem em pares ordenados (x, y) de inteiros positivos para a equacao
xy
= n?
x+y
Soluc
ao. Primeiro reescrevemos a equacao como
xy nx ny + n2 = n2 (x n)(y n) = n2 .


POT 2012 - Algebra
- Nvel 2 - Aula 14 - Prof. Marcelo Mendes

Portanto, a cada divisor positivo de n2 corresponde um valor de x n e uma solucao.


Os divisores negativos sempre geram x ou y n
ao-positivo.
2ak
2a2
1
Sendo n = pa11 pa22 ... pakk , temos n2 = p2a
1 p2 ... pk , que possui

(2a1 + 1)(2a2 + 1)...(2ak + 1)


divisores positivos, que e a quantidade de solucoes da equacao inicial.
Problema 13. Determine todos os pares ordenados (m, n) de n
umeros inteiros positivos
2
4
+ = 1.
que s
ao soluc
oes da equac
ao
m n
Problema 14. Os tres n
umeros distintos a, b, c verificam as igualdades
3
a + pa + q = 0
b3 + pb + q = 0 .
3
c + pc + q = 0

Prove que a + b + c = 0.

Problema 15. (Czech and Slovak) Encontre todos os pares de inteiros a, b tais que a soma
a + b seja uma raiz da equac
ao x2 + ax + b = 0.


POT 2012 - Algebra
- Nvel 2 - Aula 14 - Prof. Marcelo Mendes

Dicas

1. Calcule o da equac
ao com vari
avel em y e resolva a inequacao 0.
2. Eleve ao cubo a equac
ao membro a membro.
r
x2 + 3
= a.
3. Use a substituic
ao
x
4. Combine os resultados das equac
oes

1
y
1
2
=
e
=
.
xy
z x + 1 xy
z+1

5. Comece procurando soluc


oes em que a = b.
5. Repita a ideia do problema 5.
7. Observe que, se a equac
ao possui solucao, entao x Z.
8. Eleve as equac
oes ao quadrado e lembre que razes quadradas s
ao n
ao-negativas.
1
10. Passe z e para o lado direito da respectiva equacao, equilibrando a quantidade de
z
termos em cada membro das equacoes.
11. Mesma dica do problema 10.
13. Tome como base as ideias da questao 10.
14. Subtraia as equac
oes 2 a 2.
15. Substitua a + b na equac
ao, calcule e iguale-o a um quadrado perfeito.


POT 2012 - Algebra
- Nvel 2 - Aula 14 - Prof. Marcelo Mendes

Soluc
oes e Respostas

1. a) (, 2] [3, +); b) y =

x2 x 6
.
2

2. 3 e -12.
3. Nao h
a soluc
ao real.
4.

1
y
1
2
=
xy 2 = z x + 1 e
=
z + 1 = 2xy.
xy
zx+1
xy
z+1
Da, xy 2 + x = 2xy x(y 1)2 = 0. Como x 6= 0, entao y = 1. Portanto,
z + y = z + 1 = 2x x =

z+y
.
2

5. Entenda inicialmente que o problema n


ao exige que se encontre todas as solucoes,
mas apenas (pode n
ao parecer a palavra mais adequada, mas pode ser ela sim!) que
existem infinitas soluc
oes.
Comecemos buscando soluc
oes em que a = b. A equacao se tornaria 2013an = cn+1 .
Nesse caso, a = c = 2013 e soluc
ao, porem ainda n
ao as conseguimos em quantidade
infinita.
Mas isso, agora, e facil. Escolha a = b = 2013kn+1 e c = 2013kn . Variando k sobre
N, geramos as infinitas soluc
oes pedidas.
7. Suponha que a equac
ao possua solucao. O lado direito da equacao e um n
umero
1
ao e um n
umero
inteiro. Assim, x Z e, portanto, x = 1 x, o que d
a x = , que n
2
inteiro, absurdo. Ou seja, n
ao h
a solucao para a equacao.
8. Naturalmente, vamos elevar ao quadrado as 3 equacoes envolvidas. Somando os resultados, obtemos x + y + z = 3.
Por outro lado, a partir da primeira equacao, obtemos z 1. Analogamente, x 1
e y 1 das demais equac
oes, que geram x + y + z 3.
Portanto, as igualdades devem ocorrer e a u
nica solucao e x = y = z = 1.
10. As soluc
oes s
ao x = y, y = z ou z = x.
11. 2000.
6


POT 2012 - Algebra
- Nvel 2 - Aula 14 - Prof. Marcelo Mendes

13.

4 2
+ = 1 (m4)(n2) = 8. As solucoes inteiras positivas vem de m4 = 1, 2, 4, 8
m n
e n 2 = 8, 4, 2, 1, respectivamente, ou seja, (m, n) = (5, 10), (6, 6), (8, 4), (12, 3).

14. Subtraindo as 2 primeiras equac


oes membro a membro, temos
(a b)(a2 + ab + b2 + q) = 0.
Como a 6= b, obtemos a2 + ab + b2 + q = 0. Analogamente, b2 + bc + c2 + q = 0.

Combinando esses 2 resultados (pela subtracao), chegamos a (a c)(a + b + c) = 0.


Mas a 6= c. Portanto, a + b + c = 0.
15. Se a + b e raiz, ent
ao
(a + b)2 + a(a + b) + b = 0 2a2 + 3ab + b2 + b = 0,
cujo discriminante e
= b2 8b = (b 4)2 16,
que deve ser um quadrado perfeito, digamos k2 , k Z+ , pois a Z, ou seja,
(b 4)2 k2 = 16 (b 4 + k)(b 4 k) = 16.
Lembrando que soma e diferenca dos mesmos n
umeros inteiros tem a mesma paridade
e que b 4 + k b 4 k pois assumimos, sem perda de generalidade, que k 0,
temos as seguintes possibilidades:


b 4 + k = 4, 8, 2, 4
,
b 4 k = 4, 2, 8, 4

que produzem as soluc


oes (a, b) = (6, 8), (6, 9), (0, 1), (0, 0).

Polos Olmpicos de Treinamento


Curso de lgebra - Nvel 2
Prof. Marcelo Mendes

Aula

15

A Ideia de Continuidade
Quando dizemos que um processo funciona de forma contnua, estamos dizendo que ele
ocorre sem interrupc
ao.
Da mesmo forma, em matem
atica, o conceito de continuidade em funcoes, por exemplo,
significa que seu gr
afico n
ao tem interrupcao. Os primeiros exemplos de funcoes contnuas
s
ao os polin
omios, que s
ao somas de parcelas do tipo ak xk , sendo k um n
umero inteiro
n
ao-negativo, ou seja, func
oes do tipo
P (x) = an xn + an1 xn1 + ... + a1 x + a0 , an 6= 0,
sendo que, no ensino fundamental, sempre s
ao estudadas as funcoes polinomiais de 1o e 2o
graus, cujos gr
aficos s
ao, respectivamente, retas e par
abolas, ambos sem interrupcao, isto
e, contnuos, da essas func
oes serem ditas contnuas.
A formalizac
ao do conceito de continuidade envolve nocoes de C
alculo - limite, especificamente - mas ja e possvel fazer muito sem formalidades. Por outro lado, precisamos
assumir detalhes que n
ao provaremos aqui nessas aulas. Por exemplo, a soma (e a diferenca,
portanto) e o produto de func
oes contnuas tambem s
ao funcoes contnuas.
` vezes, tambem, podemos considerar uma funcao contnua apenas em um determiAs
1
possui uma descontinuidade em x = 0,
nado intervalo. Por exemplo, a func
ao f (x) =
x
mas e contnua para reais positivos.
Vejamos alguns exemplos.
Problema 1. Mostre que a equac
ao x3 5x + 2 = 0 possui uma raiz real positiva.
Soluc
ao. O lado esquerdo dessa equacao e um polin
omio, digamos P (x), ou seja, podemos
escrever P (x) = x3 5x + 2.


POT 2012 - Algebra
- Nvel 2 - Aula 15 - Prof. Marcelo Mendes

Agora, veja que P (0) = 2 > 0 e P (1) = 2 < 0. Como o gr


afico de P e contnuo, pois
e um polin
omio, ele une os pontos (0,2) e (1, -2) e, necessariamente, intercepta o eixo x
entre 0 e 1, isto e, P possui uma raiz real entre 0 e 1.
Problema 2. Prove que a equac
ao x5 + 8x4 3x2 4x 2 = 0 possui ao menos duas razes
reais negativas e uma positiva.
Problema 3. Seja h : R R uma funcao polinomial. Sabe-se que
h(1) = 4, h(0) = 0, h(1) = 8.
Definimos g por g(x) = h(x) 2. Mostre que a equacao g(x) = 0 admite pelo menos 2
solucoes distintas.
Problema 4. A equac
ao
n4 2n3 + 3n2 + n 33 = 0
possui solucao real positiva?
Problema 5. De uma func
ao g contnua em R, sabe-se que:
i. 1 e raiz de g;
ii. g(3) > 0.
Prove que a equac
ao g(x) =

g(3)
tem, pelo menos, uma solucao no intervalo ]1; 3[.
2

Problema 6. Prove que o gr


afico de f (x) = x3 3x2 +1 intersecta o gr
afico de g(x) = 2x2
em pelo menos um ponto do intervalo ]0, 1[.
Problema 7. (Paulista) Demonstre que, no conjunto dos n
umeros reais, a equacao
(x b)(x c) + (x a)(x c) + (x a)(x b) = 0
sempre tem soluc
ao, quaisquer que sejam os n
umeros reais a, b, c dados.
Soluc
ao. Seja
P (x) = (x b)(x c) + (x a)(x c) + (x a)(x b).
Inicialmente, observe que se a = b ou b = c ou c = a, entao P possui uma raiz (a, b ou c).


POT 2012 - Algebra
- Nvel 2 - Aula 15 - Prof. Marcelo Mendes

Caso a, b, c sejam distintos 2 a 2, podemos supor, sem perda de generalidade (devido `a


simetria da equac
ao), a < b < c.
Assim, temos
P (a) = (a b)(a c) > 0,
P (b) = (b a)(b c) < 0,
P (c) = (c a)(c b) > 0,
que garantem uma raiz entre a e b e outra entre b e c.
Obs: Esta e uma soluc
ao envolvendo o conceito de continuidade dos polin
omios. Tente
resolve-la calculando o da equac
ao do 2o grau.
Problema 8. (IME) Considere a, b, c R tais que a < b < c. Prove que a equacao abaixo
possui exatamente duas razes x1 e x2 que satisfazem a condicao a < x1 < b < x2 < c.
1
1
1
+
+
= 0.
xa xb xc
Problema 9. Seja f uma func
ao quadr
atica (funcao polinomial do 2o grau) tal que a equacao
f (x) = x n
ao tem soluc
oes. Prove que a equacao f (f (x)) = x tambem n
ao tem solucoes.
Soluc
ao. Primeiramente, veja que a condicao de f (x) = x, que e uma equacao do 2o grau,
n
ao possuir razes poderia nos levar a pensar em usar seu discriminante . Porem, seria
complicado aplicar tal resultado na equacao f (f (x)) = x, que e do 4o grau.
Pensemos, ent
ao, com nossos argumentos de continuidade, ja que f e contnua pois seu
gr
afico (uma par
abola) n
ao possui interrupcao. Para simplificar um pouco nossa solucao,
vamos criar a func
ao g(x) = f (x) x, que tambem e contnua utilizando um resultado
(que n
ao iremos provar) que garante que soma de funcoes contnuas tambem e uma funcao
contnua.
Assim, a condic
ao do problema que garante que f (x) = x n
ao possui solucao e equivalente a dizer que g n
ao possui raiz real, ou seja, seu gr
afico est
a todo acima do eixo x ou
todo abaixo, devido `
a continuidade.
Sem perda de generalidade (o outro caso e an
alogo), vamos supor que o gr
afico de g
esteja todo acima do eixo x, isto e, g(x) > 0, x R. Portanto, f (x) > x, x R.
Da mesma forma, tambem temos f (f (x)) > f (x), x R.


POT 2012 - Algebra
- Nvel 2 - Aula 15 - Prof. Marcelo Mendes

Finalmente, combinando esses 2 resultados, conclumos que f (f (x)) > x, x R (ou


f (f (x)) < x, x R), garantindo que f (f (x)) = x n
ao possui solucao real.
Obs. A func
ao em quest
ao poderia ser qualquer funcao contnua.
Problema 10. Seja f uma func
ao contnua para todo x real tal que a equacao f (x) = x
ao possui solucao, n N.
n
ao possui soluc
ao. Prove que f (f...f (x)...) = x tambem n
| {z }
n

Problema 11. A func


ao f : R R e contnua e

f (x) f (f (x)) = 1, x R.
Sendo f (2004) = 2003, determine f (1999).
Soluc
ao. Varios alunos iniciam esse problema substituindo f (x) por y, obtem f (y) =
1
, que e a resposta correta.
1999
1
Mas como explicar que f (2004) n
ao e
e, sim, 2003?
2004

1
e,
y

portanto, f (1999) =

A resposta est
a no fato de nem todo n
umero (y, no caso) poder ser escrito como f (x).
Isso s
o e verdade se y est
a no conjunto imagem da funcao. Por exemplo, 2003 e imagem de
1
1
2004 e isso e suficiente para garantir que f (2003) =
e, portanto,
tambem est
a
2003
2003


1
no conjunto imagem e f
= 2003.
2003
Sendo assim, nosso objetivo e conseguir mostrar que 1999 est
a no conjunto
imagem dessa

1
funcao. No par
agrafo anterior, vimos que o gr
afico de f passa pelos pontos 2003,
2003


1
e
, 2003 . A continuidade de f garante que seu gr
afico percorrera pontos com or2003
1
denadas (coordenadas y) desde
ate 2003, ou seja, vai passar por um com ordenada
2003
1
1999, o que significa dizer que 1999 est
a no conjunto imagem de f . Logo, f (1999) =
.
1999
Obs: Para uma melhor compreens
ao, faca um esboco no plano cartesiano dos dados encontrados nessa soluc
ao.
Problema 12. A func
ao f : R R e contnua e
f (x) f (f (x)) = 1, x R.
Se f (2011) = 2010, mostre que 2012 n
ao pertence ao conjunto imagem de f , ou seja, n
ao
existe x real tal que f (x) = 2012.


POT 2012 - Algebra
- Nvel 2 - Aula 15 - Prof. Marcelo Mendes

Problema 13. Sejam f, g : R R funcoes contnuas tais que f (a) < g(a) e f (b) > g(b).
Mostre que existe c (a, b) tal que f (c) = g(c).
Soluc
ao. Vamos criar a func
ao h(x) = f (x) g(x), tambem contnua. Pelo problema,
h(a) = f (a) g(a) < 0,
h(b) = f (b) g(b) > 0.
A continuidade de h garante que existe c (a, b) tal que h(c) = 0, ou seja, f (c) = g(c).
Problema 14. Seja f : [a, b] [a, b] uma funcao contnua (pois e a diferenca entre funcoes
contnuas). Prove que f tem um ponto fixo, isto e, existe c [a, b] tal que f (c) = c.
(Sugestao: considere a func
ao g(x) = f (x) x.)


POT 2012 - Algebra
- Nvel 2 - Aula 15 - Prof. Marcelo Mendes

Dicas

2. Sendo P (x) = x5 + 8x4 3x2 4x 2, calcule P (0), P (1). Mostre que P assume
apenas valores negativos quando x diminui suficientemente. Depois, calcule P (1).
3. Calcule g(1), g(0), g(1).
4. Sendo P (n) = n4 2n3 + 3n2 + n 33, calcule P (0), P (1), P (2), P (3).
5. Escreva f (x) = 2g(x) g(3). Calcule f (1) e f (3).
6. Escreva h(x) = f (x) g(x) e calcule h(0) e h(1).
8. Veja soluc
ao do problema 7.
10. Veja soluc
ao do problema 9.
12. Veja soluc
ao do problema 11.
14. Veja soluc
ao do problema 13.


POT 2012 - Algebra
- Nvel 2 - Aula 15 - Prof. Marcelo Mendes

Soluc
oes

2. Sendo P (x) = x5 + 8x4 3x2 4x 2, entao P (0) = 2 e P (1) = 6. Assim, ja


existe uma raiz negativa no intervalo (1, 0). Quem controla o sinal de P quando
x caminha para e o termo x5 , que e negativo nesse caso. Da, partindo de um
valor positivo P (1) = 6, o gr
afico, quando x caminha para , cortar
a novamente
o eixo x gerando uma nova raiz negativa. Alem disso, P (1) = 0, ou seja, 1 e uma raiz
positiva.
3. g(1) = h(1) 2 = 2, g(0) = h(0) 2 = 2, g(1) = h(1) 2 = 6, ou seja, existe
uma raiz no intervalo (1, 0) e outra no intervalo (0, 1).
4. P (2) = 19, P (3) = 24 e continuidade garantem que existe uma raiz no intervalo
(2, 3).
5. Sendo f (x) = 2g(x) g(3), temos f (1) = 2g(1) g(3) = g(3) < 0 e f (3) =
2g(3) g(3) = g(3) > 0, ou seja, existe uma raiz no intervalo (1, 3).
6. Seja h(x) = f (x) g(x) = x3 3x2 2x + 3. Como h(0) = 3 e h(1) = 1 e h
e contnua, segue que h possui uma raiz em (0, 1), isto e, os gr
aficos de f e g se
intersectam em pelo menos um ponto desse intervalo.


1
1
ef
= 2010, ou seja,
12. Pela soluc
ao do problema 11, temos que f (2010) =
2010
2010
2010 est
a no conjunto imagem de f . Se 2012 tambem estivesse, entao a continuidade
1
, absurdo.
de f garantiria que 2011 tambem estaria e, portanto, f (2011) =
2011
14. Se f (a) = a ou f (b) = b, ent
ao n
ao h
a nada a fazer. Caso contr
ario, temos f (a) > a
e f (b) < b. O resultado segue an
alogo ao encontrado na solucao do problema 13
criando a func
ao h(x) = f (x) x.

Polos Olmpicos de Treinamento


Aula

Curso de lgebra - Nvel 2


Prof. Marcelo Mendes

16

N
umeros Complexos - Parte I
Introdu
c
ao e Forma Alg
ebrica

S
ao as express
oes da forma a + bi, em que a e b s
ao n
umeros reais e i = 1 (i e a
primeira letra da palavra imagin
ario, sin
onimo de n
umero complexo) ou i2 = 1. Dizemos
que z = a + bi e forma algebrica do n
umero complexo z.
Os n
umeros complexos da forma a + 0i s
ao chamados n
umeros reais. Assim, R C.
Os n
umeros complexos da forma 0 + bi s
ao chamados n
umeros imagin
arios puros. Em
particular, 0 + 1i = i e chamado de unidade imagin
aria.

Interpreta
c
ao Geom
etrica dos N
umeros Complexos
Im

z = a + bi

bi
|z|

A figura acima mostra um n


umero complexo no plano, que chamaremos de plano complexo. Nele, o eixo horizontal contem n
umeros reais e o eixo vertical, n
umeros imagin
arios
puros. A dist
ancia de z `
a origem e o m
odulo de z (assim como acontece com os n
umeros
reais) e representamos da maneira usual, ou seja, |z|. Observe que existe uma associacao


POT 2012 - Algebra
- Nvel 2 - Aula 16 - Prof. Marcelo Mendes

entre a notac
ao cartesiana de um ponto (x, y) e a notacao complexa do n
umero x + yi.

Opera
c
oes
1. Igualdade: a + bi = c + di a = c e b = d. (Observacao: Nao h
a comparacao dos tipos
>, <, , )
2. Soma: a + bi + c + di = (a + c) + (b + d)i.
3. Produto: (a + bi)(c + di) = (ac bd) + (ad + bc)i.
ac + bd
bc ad
a + bi
= 2
+i 2
.
c + di
c + d2
c + d2

5. Modulo: |z| = a2 + b2 .
4. Quociente:

Pot
encias do i
As potencias de i s
ao peri
odicas. De fato, i1 = i i2 = 1, i3 = i, i4 = 1, i5 = i,
i6 = 1, ... A repetic
ao ocorre a cada 4 potencias (o perodo da repeticao e 4.) Tambem e
s
comum precisar calcular potencias de 1+i ou 1i. E
o usar que (1i)2 = 12i1 = 2i.

O Conjugado e suas Propriedades


Im

z = a + bi

bi
|z|

bi

|z|
z = a bi

umero complexo z = a + bi e, por definicao, o n


umero complexo
O conjugado z n
z = a bi. Vejamos algumas propriedades u
teis.
ao mutuamente conjugados.
1. O conjugado do n
umero complexo z e z e, por isso, z e z s


POT 2012 - Algebra
- Nvel 2 - Aula 16 - Prof. Marcelo Mendes

2. |z| = |z|, pois cada lado da igualdade e

a 2 + b2 .

3. z = z z R, ja que a + bi = a bi implica b = 0. Esse fato deve ser usado quando o


objetivo for provar que um determinado n
umero e real.
4. z = z z e um n
umero imagin
ario puro, pois a + bi = (a bi) implica a = 0.
Esse fato deve ser usado quando o objetivo for provar que um determinado n
umero e
imagin
ario puro.
UMA PROPRIEDADES
5. |z|2 = z z, pois z z = (a + bi)(a bi) = a2 + b2 . ESSA E

MAIS UTEIS, pois consegue eliminar o m


odulo dos calculos, algo bom mesmo que os
n
umeros envolvidos sejam reais.
6. A soma ((a + bi) + (a bi) = 2a) e o produto (visto no item anterior) de n
umeros
mutuamente conjugados e um n
umero real.
7. z1 + z2 = z1 + z2 . De fato, z1 + z2 = (a1 + ib1 ) + (a2 + ib2 ) = (a1 + a2 ) + i(b1 + b2 ) =
(a1 + a2 ) i(b1 + b2 ) = (a1 ib1 ) + (a2 ib2 ) = z1 + z2 (Esse fato ja foi uma questao
proposta pelo IME).
8. z1 z2 = z1 z2 .
 
z1
z1
= .
9.
z2
z2
Tente verificar esses dois u
ltimos itens fazendo z1 = a + bi e z2 = c + di. Vejamos agora
alguns exemplos.
Problema 1. Calcule i2011 , i2012 , i2013 .
Problema 2. Calcule o valor de i8n+3 + i4n+1 .
Problema 3. Calcule (1 + i)2011 , (1 i)2012 , (1 + i)2013 .
Problema 4. Encontre todas as razes da equacao z 3 = 1.
Soluc
ao. A equac
ao pode ser reescrita como
z 3 1 = 0 (z 1)(z 2 + z + 1) = 0.
A primeira raiz z = 1 vem de z 1 = 0. As demais vem dez 2 + z + 1 =0, cujo
1 3
1 i 3
discriminante e = 3. Logo, essas u
ltimas duas razes s
ao
=
.
2
2


POT 2012 - Algebra
- Nvel 2 - Aula 16 - Prof. Marcelo Mendes

Problema 5. Encontre as razes das equacoes


a) z 3 = 8;
b) z 4 = 81.
Problema 6. Encontre n
umeros reais x, y, u, v satisfazendo
z = x + i, w = 3 + iy,
z + w = u i, zw = 14 + iv.
Problema 7. Seja z = a + bi, em que a, b R. Encontre condicoes sobre a e b para que:
a) z 3 seja real;
b) z 3 seja imagin
ario puro.
Soluc
ao. Veja:
z = a + bi z 3 = (a + bi)3 = a3 + 3a2 bi + 3a(bi)2 + (bi)3
z 3 = (a3 3ab2 ) + i(3a2 b b3 ).
a) z 3 e real se, e somente se, sua
parte imagin
aria e nula, ou seja, 3a2 b b3 = 0, o que
ocorre quando b = 0 ou b = a 3.
b) z 3 e imagin
ario puro se, e somente
se, sua parte real e nula, ou seja, a3 3ab2 = 0, o
que ocorre quando a = 0 ou a = b 3.
Problema 8. Para z C, prove que
|z| = 1 z =

1
.
z

Problema 9. Prove que |1 + iz| = |1 iz| se, e somente se, z e um n


umero real.
Soluc
ao. Sabendo que |z|2 = z z, temos
|1 + iz| = |1 iz| |1 + iz|2 = |1 iz|2
(1 + iz)(1 + iz) = (1 iz)(1 iz)
(1 + iz)(1 iz) = (1 iz)(1 + iz)

1 + iz iz + |z|2 = 1 iz + iz + |z|2
iz = iz z = z,
que e a condic
ao necessaria e suficiente para z ser real.
4


POT 2012 - Algebra
- Nvel 2 - Aula 16 - Prof. Marcelo Mendes

Problema 10. Sejam a e b n


umeros reais. Se a + bi 6= 0, determine a forma algebrica do
1
.
n
umero
a + bi
Soluc
ao. A ideia de tornar o denominador real e sempre utilizar o conjugado, parecido
com o que fazemos quando queremos racionalizar um denominador irracional:
a bi
a bi
a
b
1
=
= 2
= 2
+i 2
,
2
2
a + bi
(a + bi)(a bi)
a +b
a +b
a + b2

que e a forma algebrica desejada.

Problema 11. (ITA) Seja z = a + bi um n


umero complexo. Se z +
entao mostre que b = 0 ou |z| = 1.

1
e um n
umero real,
z

Problema 12. (ITA) Se z1 e z2 s


ao n
umeros complexos e z1 + z2 e z1 z2 s
ao ambos reais,
entao mostre que z1 e z2 s
ao ambos reais ou z1 = z.


POT 2012 - Algebra
- Nvel 2 - Aula 16 - Prof. Marcelo Mendes

Dicas

5. Veja a soluc
ao da quest
ao 3.
6. Use que a igualdade entre n
umeros complexos, ou seja,a + bi = c + di a = c e
b = d, se a, b, c, d s
ao reais.
8. Use |z|2 = z z.
11. Use o problema 10.
12. Escreva z1 e z2 em suas formas algebricas, ou seja, z1 = a + bi e z2 + c + di, sendo
a, b, c, d n
umeros reais.


POT 2012 - Algebra
- Nvel 2 - Aula 16 - Prof. Marcelo Mendes

Respostas e Soluc
oes

1. i, 1, i.
2. 0.
3. (1 + i)2011 = (1 + i)2010 (1 + i) = (2i)1005 (1 + i) = 21005 i1004 i(1 + i) = 21005 (i 1).
(1 i)2012 = (2i)1006 = (2)1006 i1004 i2 = 21006 .

(1 + i)2013 = (1 + i)2011 (1 + i)2 = 21005 (i 1)2i = 21006 (1 + i).

1. a) 2, 1 3.
b) 3, 3i.

6. x = 4, y = 2, u = 7 e v = 5.
8. Sabendo que |z|2 = z z, temos
|z| = 1 |z|2 = 1 zz = 1 z =

1
.
z

a
b
b
b(a2 + b2 1)
1
= a + bi + 2
+
i

ser
a
real
se
b

=
= 0,
z
a + b2
a 2 + b2
a2 + b2
a 2 + b2
ou seja, b = 0 ou |z| = a2 + b2 = 1.

11. z +

12. Escrevendo z1 = a + bi e z2 + c + di, temos z1 + z2 = (a + c) + i(b + d) e z1 z2 =


(ac bd) + i(ad + bc). Como esses n
umeros s
ao reais, devemos ter:
1. b + d = 0 b = d;

2. ad + bc = 0 ad = bc.
Se d = 0, ent
ao b = 0. Da, z1 e z2 s
ao reais. Se d 6= 0, entao podemos fazer
o cancelamento na equac
ao do item 1 e achar a = c. Isso mostra que z1 e z2 s
ao
conjugados complexos.

Polos Olmpicos de Treinamento


Aula

Curso de lgebra - Nvel 2


Prof. Marcelo Mendes

17

N
umeros Complexos - Parte II
Vamos finalizar nosso estudo dos n
umeros complexos apresentando a forma de escrevelos com o auxlio da Trigonometria, que dar
a suporte a mais teoria posterior, e mais
exerccios.

Forma Trigonom
etrica
Im

z = a + bi

bi
|z|

A figura acima nos permite escrever cos =

b
a
e sen =
. Assim, temos
|z|
|z|

z = a + ib = |z|cos + i|z|sen,
ou seja,
z = |z|(cos + isen).
O angulo e chamado de argumento do n
umero complexo z e o denotamos por arg z.
Completando as propriedades do conjugado, temos arg z = 360o arg z.
Problema 1. Escreva os seguintes n
umeros na forma trigonometrica.


POT 2012 - Algebra
- Nvel 2 - Aula 17 - Prof. Marcelo Mendes

a) 2.
b) 3i.
c) 1 + i.

d) 1 + i 3.
Soluc
ao.
a) 2 = 2(cos0 + isen0).
b) 3i = 3(cos90o + isen90o ).
!

2
2
c) 1 + i = 2
= 2(cos45o + isen45o ).
+i
2
2

d) 1 + i 3 = 2

!
3
1
+i
= 2(cos60o + isen60o ).
2
2

Para ilustrar o item c) dessa quest


ao, observe a figura a seguir.
Im

z = 1+i
i

| 2|
45o

Apos localizar o n
umero 1 + i no plano complexo, visualizamos um quadrado
de lado
o
1. Assi, fica mais facil enxergar que o argumento de z e 45 e que o m
odulo e 2.
Problema 2. Determine o polin
omio de menor grau e com coeficientes reais que possui um
2
como raiz.
n
umero complexo com m
odulo 1 e argumento
3
Problema 3. Sejam x = a + b, y = a + b 2 , z = a 2 + b, onde 2 + + 1 = 0. Calcule
x + y + z e expresse x3 + y 3 + z 3 em termos de a e b.


POT 2012 - Algebra
- Nvel 2 - Aula 17 - Prof. Marcelo Mendes

Problema 4. (EUA) O n
umero complexo z satisfaz z + |z| = 2 + 8i. Calcule |z|2 .
Soluc
ao. Supondo z = a + bi, a equac
ao fica
p
a + bi + a2 + b2 = 2 + 8i.

A igualdade entre n
umeros complexosnos garante que b = 8 (comparando as partes
imagin
arias dos lados da equac
ao) e a + a2 + b2 = 2 (comparando as partes reais dos
lados da equac
ao), ou seja,
p
a2 + b2 = 2 a 0 a = 15.
Portanto, z = 15 + 8i e a u
nica solucao e |z|2 = 152 + 82 = 289.

Problema 5. (IME) Dois n


umeros complexos z1 e z2 , n
ao-nulos, s
ao tais que |z1 + z2 | =
z2
e imagin
ario puro.
|z1 z2 |. Mostre que
z1
Problema 6. (IME) Sendo a, b e c n
umeros naturais em progressao aritmetica e z um
n
umero complexo de m
odulo unit
ario, determine um valor para cada um dos n
umeros
a, b, c e z de forma que eles satisfacam a igualdade
1
1
1
+ b + c = z9.
a
z
z
z
Problema 7. (ITA) Determine todos os n
umeros complexos z, que s
ao razes da equacao
|z| z = 1 + 2i, sendo i a unidade imagin
aria.
Problema 8. (ITA) Considerando z e w n
umeros complexos arbitrarios e u = z w + z w,
mostre que o conjugado de u e igual ao dobro da parte real do n
umero z w.
Problema 9. (ITA) Determine o valor da express
ao |1 z|2 + |1 + z|2 , sendo z um n
umero
complexo unit
ario.
Problema 10. (ITA)
Determine o produto dos n
umeros complexos z = x + yi que tem

m
odulo igual a 2 e tais que y = 2x 1.
Soluc
ao. |z|2 = 2 x2 + (2x 1)2 = 2 5x2 4x 1 = 0, cujas razes s
ao x = 1 e
1
x= .
5
1 7
6 8
Assim, os n
umeros s
ao 1 + i e i, cujo produto e i.
5 5
5 5


POT 2012 - Algebra
- Nvel 2 - Aula 17 - Prof. Marcelo Mendes

Problema 11. (ITA) Mostre que, resolvendo a equacao z 2 = 2 + z no conjunto dos n


umeros
complexos, todas as razes s
ao n
umeros inteiros.
Problema 12. (ITA) Sejam x e y n
umeros reais, com x 6= 0, satisfazendo (x+iy)2 = (x+y)i.
Mostre que x e uma raiz da equac
ao x3 + 3x2 + 2x 6 = 0.
Soluc
ao. (x + iy)2 = (x + y)i x2 + 2xyi y 2 = (x + y)i. Pela igualdade entre n
umeros
complexos, temos x2 y 2 = 0, o que d
a x = y ou x = y.
Se x = y, ent
ao x + y = 0 e 2xy = 0, absurdo pois x 6= 0.
Logo, x = y e 2xy = x + y equivale a x2 = x. Assim, x = 1, que e uma raiz da equacao
dada.
Problema 13. Resolva a equac
ao (z + i)2 + (z i)2 = 2.
Problema 14. (ITA) Escreva as formas algebrica e trigonometrica da potencia

!93
2
.
1+i


POT 2012 - Algebra
- Nvel 2 - Aula 17 - Prof. Marcelo Mendes

Dicas

2. Esse n
umero e z =

cos120o

isen120o

3
1
.
= +i
2
2

3. Mostre que 3 = 1 e use esse resultado.


5. Use |z|2 = z z.
6. Tome z = i, que tem m
odulo unit
ario e encontre valores para a, b e c.
7. Escreva z = a + bi, com a e b reais.
8. Use z w = z w e escreva zw = a + bi, com a e b reais.
9. Use |w|2 = w w.
11. Escreva z = a + bi, com a e b reais.
14. Use (1 + i)2 = 2i.


POT 2012 - Algebra
- Nvel 2 - Aula 17 - Prof. Marcelo Mendes

Respostas e Soluc
oes

2. P (z) = z 2 + z + 1.
3. x + y + z = a(1 + + 2 ) + b(1 + + 2 ) = 0. Assim, x3 + y 3 + z 3 = 3xyz = 3(a3 + b3 )
pois
xyz = (a + b)(a + b 2 )(a 2 + b)
= (a + b)(a2 + b2 + ab( 4 + 2 )
= (a + b)(a2 + b2 ab) = a3 + b3 ,
visto que 4 = pois 3 1 = ( 1)( 2 + + 1) = 0.

Outra forma de calcular x3 + y 3 + z 3 e elevar ao cubo as express


oes de x, y, z e,
depois, somar os resultados.
5. |z1 + z2 | = |z1 z2 | |z1 + z2 |2 = |z1 z2 |2 (z1 + z2 )(z1 + z2 ) = (z1 z2 )(z1 z2 )
(z1 + z2 )(z1 + z2 ) = (z1 z2 )(z1 z2 ) z1 z1 + z1 z2 + z2 z1 + z2 z2 = z1 z1 z1 z2
z2 z1 + z2 z2 z1 z2 + z2 z1 = 0
6. Tomando z = i, que tem m
odulo 1, uma possvel solucao e a = 2, b = 3 e c = 4
(P.A.), como pedido no enunciado (n
ao foi pedido encontrar todas as solucoes).

umeros
7. Fazendo z = a + bi, temos a2 + b2 a bi = 1 + 2i. A igualdade entre n
complexos nos d
a b = 2 e
p

Logo, z =

3
a2 + 4 = a + 1 a = .
2

3
2i e a u
nica solucao.
2

8. u = u. Se zw = a + bi, ent
ao u = 2a.
9. |1 z|2 + |1 + z|2 = (1 z)(1 z) + (1 + z)(1 + z) = 2 + 2|z|2 = 4.
11. Fazendo z = a + bi, temos a2 + b2 = 2 + a bi. A igualdade entre n
umeros complexos
garante que b = 0 e que a2 = 2 + a, cujas razes s
ao 2 e -1.

13. z = 2.

93 92
2 = 246 2.
14. 2 = 2
(1 + i)93 = (1 + i)92 (1 + i) = (2i)46 (1 + i) = 246 (1 + i)
!93

2
2
2(1 i)
2
2
Assim,
=
=
=
+
i = cos315o + isen315o .
1+i
1+i
(1 + i)(1 i)
2
2

Polos Olmpicos de Treinamento


Aula

Curso de lgebra - Nvel 2


Prof. Marcelo Mendes

18

Relac
oes de Girard - Parte I
Ao estudar as equac
oes (polinomiais) do 2o grau, voce deve ter aprendido que e possvel
calcular a soma e o produto das razes, mesmo sem conhece-las. Vamos recordar essas
formulas.
Sejam x1 e x2 as razes da equac
ao ax2 + bx + c = 0. Assim, podemos escrever
ax2 + bx + c = a(x x1 )(x x2 )
ax2 + bx + c = ax2 a(x1 + x2 )x + ax1 x2 .
Igualando os coeficientes de termos de mesmo grau, obtemos b = a(x1 + x2 ) e c =
ax1 x2 , ou seja,
b
x1 + x2 = ,
a
c
x1 x2 = .
a
Agora, n
ao pense mais em soma e produto das razes e, sim, que s
ao somas das razes
separadamente e, depois, aos pares. Para um conjunto de 3 razes, por exemplo, a soma
delas separadamente seria x1 + x2 + x3 e, aos pares, x1 x2 + x2 x3 + x3 x1 . Alem disso, ainda
haveria a soma delas de 3 em 3: x1 x2 x3 , que, nesse caso, coincide com o produto.
Com essas express
oes e repetindo os calculos acima para uma equacao ax3 +bx2 +cx+d =
0, cujas razes s
ao x1 , x2 e x3 , obtemos
b
x1 + x2 + x3 = ,
a
x1 x2 + x2 x3 + x3 x1 =

c
,
a

d
x1 x2 x3 = .
a
E isso pode ser extendido para equacoes polinomiais de qualquer grau, sempre dividindo
pelo primeiro coeficiente, que e sempre diferente de zero, e fazendo altern
ancia de sinais.


POT 2012 - Algebra
- Nvel 2 - Aula 18 - Prof. Marcelo Mendes

Tais resultados s
ao conhecidos como Relaco
es de Girard (Albert Girard (1590 - 1639)).
Por exemplo, se a equac
ao for 2x4 + 3x2 + 8x + 1 = 0, entao
x1 + x2 + x3 + x4 = 0,
3
x1 x2 + x1 x3 + x1 x4 + x2 x3 + x2 x4 + x3 x4 = ,
2
8
x1 x2 x3 + x1 x2 x4 + x1 x3 x4 + x2 x3 x4 = = 4,
2
1
x1 x2 x3 x4 = .
2
o
Problema 1. Determine uma equac
ao do 3 grau cujas razes sejam 1, 2 e 3.
Soluc
ao. O resultado e imediato escrevendo
(x 1)(x 2)(x 3) = 0 x3 6x2 + 11x 6 = 0.
Mas tambem podemos criar os coeficientes da equacao atraves das Relacoes de Girard.
Sendo x3 ax2 + bx c = 0 uma equacao (pois podemos multiplic
a-la por qualquer valor
n
ao-nulo sem que suas razes sejam alteradas) procurada (os sinais nos coeficientes est
ao
alternados pelo padrao das Relac
oes de Girard). Assim:
a = 1 + 2 + 3 = 6,
b = 1 2 + 2 3 + 3 1 = 11,
c = 1 2 3 = 6,
obtendo assim a equac
ao x3 6x2 + 11x 6 = 0.
Problema 2. Resolva a equac
ao polinomial x3 + 4x2 + x 6 = 0 sabendo que uma de suas
razes e 1.
Soluc
ao. Sejam 1, r, s as razes dessa equacao. Assim, por Girard, temos:
1 + r + s = 4,
1 r s = 6,
ou seja,
r + s = 5,
rs = 6,
donde r = 2 e s = 3.
Problema 3. (EUA) Para quantos inteiros positivos n entre 1 e 100 e possvel fatorar
x2 + x n como produto de dois fatores lineares com coeficientes inteiros?


POT 2012 - Algebra
- Nvel 2 - Aula 18 - Prof. Marcelo Mendes

Problema 4. (ITA) Se a, b, c s
ao as razes da equacao x3 2x3 + x 4 = 0, determine o
1 1 1
valor de + + .
a b
c
Problema 5. (ITA) As razes da equac
ao x4 + qx3 + rx2 + sx + t = 0, com q, r, s, t Q+ ,
s
ao L, M, N, P . Determine o valor de
M
N
P
L
+
+
+
.
MNP
LN P
LM P
LM N
Problema 6. (IME) Sejam x1 e x2 as razes da equacao x2 + (m 15)x + m = 0. Sabendo
que x1 e x2 s
ao n
umeros inteiros, determine o conjunto de valores possveis para m.
Problema 7. Os tres n
umeros distintos a, b, c verificam as igualdades
3
a + pa + q = 0
b3 + pb + q = 0 .
3
c + pc + q = 0
Prove que a + b + c = 0.

Soluc
ao. As relac
oes dadas significam que a, b e c s
ao as razes da equacao polinomial do
3o grau x3 + px + q = 0, que, por Girard, tem soma das razes igual a 0, isto e, a + b + c = 0.
Problema 8. Sejam m, n, k Q as razes de t3 + at + b. Prove que as razes de mt2 + nt + k
tambem s
ao racionais.
Soluc
ao. Observe que n
ao existe o termo do 2o grau em t3 + at + b. Assim, por Girard,
temos que m + n + k = 0. Ora, mas isso mostra que 1 e uma raiz de mt2 + nt + k.
k
Novamente, Girard, atraves do produto das razes, nos d
a que a outra raiz e , que
m
e racional por ser quociente de racionais. Isso mostra que as 2 razes de mt2 + nt + k s
ao
racionais.
Problema 9. (ITA) As razes da equacao de coeficientes reais x3 + ax2 + bx + c = 0 s
ao
inteiros positivos consecutivos. A soma dos quadrados dessas razes e igual a 14. Determine
o valor de a2 + b2 + c2 .
Problema 10. Determine o valor da soma a + b para que as razes do polin
omio
4x4 20x3 + ax2 25x + b
estejam em progress
ao aritmetica de raz
ao

1
.
2


POT 2012 - Algebra
- Nvel 2 - Aula 18 - Prof. Marcelo Mendes
1
3
Soluc
ao. Sejam r, r + , r + 1, r + as razes da equacao. Da,
2
2
3
20
1
1
=5r= ,
r+r+ +r+1+r+ =
2
2
4
2
3
1
ou seja, as razes s
ao , 1, e 2. Por Girard, temos:
2
2
a
1
1 3 1
1
3
35
= 1+ + 2+1 +12+ 2 =
a = 35,
4
2
2 2 2
2
2
4
1
3
6
b
= 1 2 = b = 6.
4
2
2
4
Portanto, a + b = 41.


POT 2012 - Algebra
- Nvel 2 - Aula 18 - Prof. Marcelo Mendes

Dicas

3. Chame as razes de a e b. Em seguida, utilize Relacoes de Girard.


4. Reduza a soma das frac
oes a um denominador comum (o que se chama comumente
de tirar o mnimo. Mas nem sempre o mmc e o produto, alem de s
o ser definido
para naturais). Em seguida, use Girard.
5. Reduza a soma das frac
oes a um denominador comum. Em seguida, use Girard.
6. Alem das Relac
oes de Girard, e interessante conseguir utilizar a identidade
xy + x + y + 1 = (x + 1)(y + 1).
9. Denote as razes por n 1, n, n + 1. Em seguida, use a dica do enunciado e Girard.


POT 2012 - Algebra
- Nvel 2 - Aula 18 - Prof. Marcelo Mendes

Respostas e Soluc
oes

3. Ao expressarmos x2 + x n como produto de dois fatores lineares com coeficientes


inteiros, os fatores ser
ao (x a) e (x b), com a e b inteiros. Por Girard, temos
a + b = 1,
ab = n.
Assim, precisamos encontrar n tal que a(a + 1) = n, 1 < n < 100. As possibilidades
s
ao a = 1, 2, 3, 4, 5, 6, 7, 8, 9, que d
ao, respectivamente, n = 2, 6, 12, 20, 30, 42, 56, 72, 90.
Portanto, o resultado e possvel para 9 valores de n.
4. Por Girard, temos que ab + bc + ca = 1 e abc = 4. Logo,
ab + bc + ca
1
1 1 1
+ + =
= .
a b
c
abc
4
5. Manipulando a equac
ao e utilizando as Relacoes de Girard, temos
M
N
P
L2 + M 2 + N 2 + P 2
L
+
+
+
=
MNP
LN P
LM P
LM N
LM N P
=

(L + M + N + P )2 2(LM + LN + LP + M N + M P + N P )
LM N P
=

q 2 2r
.
t

6. As Relac
oes de Girard nos d
ao x1 + x2 = 15 m e x1 x2 = m. Portanto, solucoes
(x1 , x2 ) e (x2 , x1 ) d
ao o mesmo resultado (valor de m) e
x1 x2 + x1 + x2 + 1 = 16 (x1 + 1)(x2 + 1) = 16.
Da, temos as possibilidades


x1 + 1 = 1, 2, 4, 4, 2, 1
x2 + 1 = 16, 8, 4, 4, 8, 16

x1 = 0, 1, 3, 5, 3, 2
,
x2 = 15, 7, 3, 5, 9, 17

que d
ao os seguintes possveis valores para m = x1 x2 : 0, 7, 9, 25, 27, 34.


POT 2012 - Algebra
- Nvel 2 - Aula 18 - Prof. Marcelo Mendes

9. Sejam n 1, n e n + 1 as razes dessa equacao. Assim,


(n 1)2 + n2 + (n + 1)2 = 14 3n2 + 2 = 14 n = 2.
Pelas Relac
oes de Girard, temos
1 + 2 + 3 = a a = 6,
1 2 + 2 3 + 3 1 = b 11,
1 2 3 = c c = 6.
Logo,
a2 + b2 + c2 = 62 + 122 + 62 = 36 + 121 + 36 = 193.

Polos Olmpicos de Treinamento


Aula

Curso de lgebra - Nvel 2


Prof. Marcelo Mendes

19

Relac
oes de Girard - Parte II
Vamos continuar vendo mais exemplos das Relacoes de Girard. Veremos tambem um
resultado novo que relaciona esse assunto com n
umeros complexos.
Problema 1. (ITA) Seja k R tal que a equacao 2x3 + 7x2 + 4x + k = 0 possua uma
raiz dupla e inteira x1 e uma raiz x2 (ou seja, as razes s
ao x1 , x1 e x2 ), distinta de x1 .
Determine o valor de (k + x1 )x2 .
Soluc
ao. Vamos utilizar as Relac
oes de Girard para soma e soma aos pares:
7
x1 + x1 + x2 = 2x1 + x2 = ,
2
4
x1 x1 + x1 x2 + x1 x2 = x21 + 2x1 x2 = = 2.
2
Eliminando x2 , obtemos:
3x21 + 7x1 + 2 = 0,
3
e, portanto,
2
3
k
x1 x1 x2 = = k = 3.
2
2

cuja raiz inteira e x1 = 1. Assim, x2 =

Problema 2. Mostrar que f (x) = x3 +x2 10x+8 e divisvel por (x1) mas n
ao e divisvel
por (x 1)2 .
Soluc
ao. Veja que f (1) = 1+ 1 10+ 8 = 0, o que mostra que f (x) possui um fator (x 1).
Agora, suponha que f (x) seja divisvel por (x 1)2 . Isto significaria que f possui 1
como raiz dupla. Suponha, ent
ao, que as razes sejam 1, 1 e r. Por Girard, temos


POT 2012 - Algebra
- Nvel 2 - Aula 19 - Prof. Marcelo Mendes

1 + 1 + r = 1 r = 3,
1 1 r = 8 r = 8,
um absurdo. Logo, f n
ao e divisvel por (x 1)2 .
Problema 3. Verifique se a equac
ao x3 3x + 8 = 0 tem razes iguais.
Problema 4. Determinar m para que a equacao x3 7x + m = 0 tenha uma raiz igual ao
dobro de uma outra.
Problema 5. (IME) Seja
p(x) = x5 + bx4 + cx3 + dx2 + ex + f
um polin
omio com coeficientes inteiros. Sabe-se que as cinco razes de p(x) s
ao n
umeros
inteiros positivos, sendo quatro deles pares e um mpar. Determine o n
umero de coeficientes
pares de p(x).
Problema 6. (OCM) Considere todas as retas que encontram o gr
afico da funcao
f (x) = 2x4 + 7x3 + 3x 5
em quatro pontos distintos, digamos (x1 , y1 ), (x2 , y2 ), (x3 , y3 ), (x4 , y4 ). Mostre que o valor
x1 + x2 + x3 + x4
e independente da reta e ache esse valor.
de
4
Soluc
ao. Seja y = ax + b a equac
ao de uma dessas retas que cortam o gr
afico de f em 4
pontos distintos. Queremos resolver a equacao f (x) = y, ou seja:
2x4 + 7x3 + 3x 5 = ax + b 2x4 + 7x3 + (3 a)x b 5 = 0.
Por Girard, x1 + x2 + x3 + x4 =

7
e, portanto,
2

x1 + x2 + x3 + x4
7
= ,
4
8
que independe da reta pois n
ao varia com os valores de a ou b. A chave dessa ideia funcionar e que os coeficientes de f que d
ao a soma (os dois primeiros) n
ao foram afetados por
a ou b.
Problema 7. (IME) Determine o valor da soma das razes da equacao
1

y 2 + 5y + 2y 2 + 8 = 0.
2


POT 2012 - Algebra
- Nvel 2 - Aula 19 - Prof. Marcelo Mendes

Problema 8. S
ao dados a, b, c R. Sabe-se que
a + b + c > 0,
bc + ca + ab > 0,
abc > 0.
Prove que a > 0, b > 0, c > 0.
Soluc
ao. Seja x3 Ax2 + Bx C = 0 a equacao cuja razes s
ao a, b, c. Por Girard, temos
A = a + b + c A > 0,
B = ab + bc + ca B > 0,
C = abc C > 0.
Suponha que a > 0, b > 0, c > 0 n
ao ocorra, ou seja, existe uma raiz r 0. Mas
r 3 |{z}
Ar 2 + |{z}
Br |{z}
C
|{z}
0

>0

seria negativo, contrariando o fato de r ser raiz. Portanto, a > 0, b > 0, c > 0.
Problema 9. Suponha que t3 + pt + q = 0 tenha uma raiz n
ao real a + bi, sendo a, b, p, q
todos reais e q 6= 0. Mostre que aq > 0.
Soluc
ao. Vamos iniciar com o seguinte
Teorema (das Razes Complexas). Se uma equacao polinomial com coeficientes reais
possui uma raiz complexa z = a + bi (b =
6 0), entao z = a bi tambem e uma raiz dessa
equacao.
Demonstraca
o. Uma equac
ao polinomial e da forma
an xn + an1 xn1 + ... + a1 x + a0 = 0.()
Se z e uma raiz dessa equac
ao, significa que
an z n + an1 z n1 + ... + a1 z + a0 = 0.
A igualdade acima garante que tambem vale a igualdade entre os conjugados. Utilizando
as propriedades vistas a respeito dos n
umeros complexos, temos
an z n + an1 z n1 + ... + a1 z + a0 = 0
an z n + an1 z n1 + ... + a1 z + a0 = 0
3


POT 2012 - Algebra
- Nvel 2 - Aula 19 - Prof. Marcelo Mendes

an z n + an1 z n1 + ... + a1 z + a0 = 0
an z n + an1 z n1 + ... + a1 z + a0 = 0,
onde a condic
ao de os coeficientes serem reais foi usada pois o conjugado de um n
umero
real e esse pr
oprio n
umero. A u
ltima equacao garante que z tambem e raiz de (*).
Voltando ao problema, temos as condicoes do teorema acima, uma vez que 1, 0, p, q
s
ao reais. Logo, a bi tambem e uma raiz. Seja r a terceira raiz. Por Girard, temos soma
0, ou seja
a + bi + a bi + r = 0,
que mostra que r = 2a e a terceira raiz. Agora, pelo produto
(a + bi)(a bi)(2a) = q 2a(a2 + b2 ) = q,
temos que a e q tem o mesmo sinal. Alem disso, q 6= 0 a 6= 0. Portanto, aq > 0.
Problema 10. (OCM) Mostre que 1 e a u
nica raiz real da equacao x3 + x2 = 2.
Problema 11. (ITA) A equac
ao 4x3 3x2 + 4x 3 = 0 admite i (unidade imagin
aria) como
raiz. Determine as demais razes.


POT 2012 - Algebra
- Nvel 2 - Aula 19 - Prof. Marcelo Mendes

Dicas

3. Use Girard para analisar os casos:


i) as 3 razes s
ao iguais.
ii) as razes s
ao r, r, s (r 6= s).
4. Denote as razes por a, 2a, b e utilize Girard.
5. Utilize Girard para analisar a paridade das razes.
1

7. Faca a substituic
ao y 2 = x e use Girard. Tenha cuidado a soma pedida e em relacao
`a vari
avel y.
10. Denote as razes diferentes de 1 por a + bi e a bi (o Teorema das Razes Complexas
garante que as u
ltimas 2 razes, de fato, s
ao n
umeros complexos conjugados). Depois,
use Girard.
11. Use o Teorema das Razes Complexas para obter que i tambem e raiz. Depois, use
Girard.


POT 2012 - Algebra
- Nvel 2 - Aula 19 - Prof. Marcelo Mendes

Respostas e Soluc
oes
3. Se suposermos que h
a 3 razes iguais, entao, pela formula de Girard para a soma,
obteramos 0 como raiz, um absurdo.
Assim, s
o nos resta analisar o caso em que as razes s
ao r, r e s. Por Girard, teramos:
2r + s = 0 s = 2r,
2

r s = 8 2r 3 = 8 r 3 = 4.
Para concluirmos o absurdo desta parte, podemos utilizar que, se r e uma raiz, entao
r 3 3r + 8 = 0 3r = 8, que contradiz a igualdade r 3 = 4.
4. Sejam a, 2a, b as razes. Por Girard, temos
a + 2a + b = 0 3a + b = 0,
a 2a + a b + 2a b = 3 2a2 + 3ab = 3.
Eliminando b, temos a = 1. Assim:
i) a = 1 b = 3. Logo, m = 6.
ii) a = 1 b = 3. Logo, m = 6.
5. Sejam p1 , p2 , p3 , p4 as 4 razes pares e i a raz mpar. Por Girard, b e a soma de
quatro n
umero pares e um mpar, ou seja, b e mpar; os demais coeficientes ser
ao
somas de produtos em que pelo menos um fator e pk (k = 1, 2, 3 ou 4) e, portanto,
s
ao todos pares. Logo, p possui 4 coeficientes pares.
1

7. Vamos comecar com a substituic


ao y 2 = x. A equacao se torna
x3 + 5x2 + 2x + 8 = 0.
Todavia, devemos ficar atentos que n
ao nos interessa o valor de x1 + x2 + x3 , uma
vez que a letra x n
ao e a inc
ognita inicial.
Nosso objetivo e calcular
y1 + y2 + y3 = x21 + x22 + x23 ,
ou seja,
(x1 + x2 + x3 )2 2(x1 x2 + x2 x3 + x3 x1 ) = (5)2 2(2) = 21.
6


POT 2012 - Algebra
- Nvel 2 - Aula 19 - Prof. Marcelo Mendes

10. Inicialmente, veja que 1 e raiz de x3 + x2 = 2, pois 13 + 12 = 2, e que essa equacao


pode ser reescrita como x3 + x2 2 = 0.
Suponha que, alem da raiz 1, essa equacao possua uma raiz complexa e n
ao-real a+bi.
Como temos a condic
ao do teorema visto no problema 20 (que e termos coeficientes
reais), a bi tambem deve ser uma raiz. Por Girard, temos
1 + a + bi + a bi = 1 a = 1,
1 (a + bi) (a bi) = 2 1 + b2 = 2 b = 1.
Temos, assim, razes 1, 1 i, que verificam a relacao de Girard restante
1(1 + i) + 1(1 i) + (1 + i)(1 i) = 0.
Assim, 1 e, de fato, a u
nica raiz real.
11. Observe, inicialmente, que todos os coeficientes dessa equacao s
ao reais. Pelo teorema visto no problema 20, podemos concluir que i (que e o conjugado do n
umero
complexo i) tambem e raiz da equacao dada.
Por Girard, sendo r a terceira raiz, entao
i + (i) + r =

3
3
r= .
4
4

Outra maneira, talvez ate mais natural de se resolver esse problema, e atraves de
fatorac
ao:
4x3 3x2 + 4x 3 = 0
x2 (4x 3) + (4x 3) = 0
(4x 3)(x2 + 1) = 0,
cujas razes s
ao, de fato,

3
, i e i.
4

Potrebbero piacerti anche